ࡱ>  @ hbjbjFF t,,e`DDDDt""222VtXtXtXtXtXtXt$vRKyl|tk"|t229tm5m5m5J)22Vtm5Vtm5m5J&*1&t2 `D2KVtt0tM2&y4y&ty&t0m5|t|tK5"1. 24-year-old G2P1 has a Pap smear performed on her 1st prenatal clinic visit at 10 weeks gestation. The results return in 2 weeks as "high grade squamous intraepithelial lesion suggestive of severe dysplasia". What is the next step in the management of this patient? a. Cold knife conization of the cervix b. Colposcopy with biopsy of any suspicious lesions c. Colposcopy with biopsy of any suspicious lesions with endocervical curettage d. Repeat Pap smear at 28 weeks e. Termination of the pregnancy followed by cold knife conization 2. An 18-year-old G1PO presents at 10 weeks gestation stating she received the rubella vaccine at 8 weeks. She tells you she is concerned about fetal anomalies and would like to terminate the pregnancy. How should you counsel this patient? a. Termination of the pregnancy is recommended since congenital rubella is likely following vaccination. b. Termination of the pregnancy is not indicated because the vaccine does not contain live virus. c. Pregnancy termination is not recommended because there are no reports of congenital rubella follow ing maternal rubella vaccination. d. Termination of the pregnancy is recommended since congenital rubella cannot be detected by ultra sound. e. Termination of the pregnancy is recommended only if the she develops rubella antibodies indicating infection occurred. 3. What constitutes a positive contraction stress test? a. The occurrence of any late decelerations in a 10 minute period b. Late decelerations occurring with greater than 50% of contractions c. Late decelerations occurring with every contraction in a 10 minute period d. Decreased beat to beat variability with variable decelerations e. Decreased beat to beat variability with or without decelerations 4. What constitutes a suspicious contraction stress test? a. Late decelerations occurring with less than 50% of contractions b. Late decelerations occurring with greater than 50% of contractions c. Late decelerations occurring with every contraction in a 10 minute period d. Decreased beat to beat variability without decelerations e. Late deceleration occurring with greater than 50% of contractions but with good beat to beat variabili ty 5. Which is considered the most accurate method to predict the expected date of delivery? a. Accurate last menstrual period b. 1st trimester pelvic examination c. 1st trimester crown-rump length ultrasound d. Early 2nd trimester biparietal diameter e. Early 2nd trimester femur length 6. How is iron transferred across the placenta to the fetus? a. simple diffusion b. facilitated diffusion c. active transport d. protein bound e. iron does not cross the placenta to the fetus 7. How is insulin transferred across the placenta to the fetus? a. simple diffusion b. facilitated diffusion c. active transport d. protein bound e. Insulin does not cross the placenta to the fetus 8. What are maternal fasting glucose levels compared with non-pregnant women? a. Not different b. Increased c. Decreased d. Twice increased e. Three times increased 9. Where is the first site of hemoglobin formation in the fetus? a. bone marrow b. spleen c. liver d. placenta e. yolk sac 10. The concentration of fetal serum a fetoprotein peaks at what gestational age? a. 8 weeks b. 15 weeks c. 22 weeks d. 30 weeks e. term 11. The concentration of maternal serum alpha-fetoprotein peaks at what gestational age? a. 8 weeks b. 15 weeks c. 22 weeks d. 30 weeks e. term 12. What mechanism is responsible for the increased risk of Down syndrome with advanced maternal age? a. Balanced translocation b. Robertsonian translocation c. Primary mitotic nondisjunction d. Primary meiotic nondisjunction e. Secondary meiotic nondisjunction 13. A patient's maternal serum alpha-fetoprotein (MSAFP) concentration is elevated. Which of the following options is recommended as the next step in the management of this? a. Repeat MSAFP levels b. Referral to maternal fetal medicine specialist for comprehensive ultrasound. c. Amniocentesis with amniotic fluid AFP concentration d. Maternal serum acetylcholinesterase determination e. Maternal serum pregnancy associated plasma protein A (PAPP-A) determination. 14. What is the next step in the management of a patient with a low maternal serum alpha-fetoprotein ? a. Repeat MSAFP determination b. Ultrasound to confirm dates c. Targeted ultrasound to determine if there is any ultrasound evidence of Down syndrome present. d. Amniocentesis for fetal karyotyping e. a or b are correct 15. Which of the following ultrasound findings would NOT increase the risk of fetal aneuploidy? a. "Double bubble" sign in the fetal abdomen b. Abnormal 4 chamber view of the heart c. Omphalocoele d. Gastroschisis e. Mid line cleft lip and palate 16. Why is the resin T3 uptake (rT3) test decreased in pregnancy? a. The normal hyperthyroid state of pregnancy b. Increased thyroid binding globulin c. Increased basal metabolic rate in pregnancy d. Human placenta! lactogen e. Fetal T3 productio 17. Total serum thyroxin levels in pregnancy compared to non-pregnant women are: a. not different b. decreased c. increased d. undetectable 18. 18.A 24 year old gravida one patient at 14 weeks presents with an elevated free thyroxin (T4) level? What is the diagnosis? a. Hyperthyroidism b. Elevated free T4 is normal in pregnancy c. Cannot be determined without knowing the TSH level d. Cannot be determined without confirmation with the resin T-3 uptake test 19. Possible consequences of Grave's disease in pregnancy include: a. Thyroid storm b. Increased perinatal morbidity and mortality c. Newborn Graves disease d. Poor maternal weight gain e. All of the above 20. You have just delivered 32 year old woman who was diagnosed with Graves disease 10 years earlier. At that time she was successfully treated with 1131 , but developed hypothyroid as a consequence of treatment. She took levothyroxine 200 meg daily.throughout the entire pregnancy. The baby is found to have a pulse of 200 bpm, exopthalamus, and a resting tremor. The pediatrician suspects hyperthyroidism. Which of the following is the most likely cause of this newbom's signs and symptoms? a. Maternal thyroxine ingestion caused symptoms of hyperthyroidism in the neonate b. Thyroid stimulating hormone (TSH) crossed the placenta causing neonatal hyperthyroidism c. Thyroid stimulating antibodies crossed the placenta d. Congenital Graves disease since Graves disease is an autosomal dominantly inherited disease and the baby accquired the abnormal gene from the mother. e. All of the above 21. Consequences of maternal hypothyroidism include all of the following EXCEPT: a. infertility b. spontaneous abortion c. maternal bradycardia d. fetal macrosomia e. increased maternal weight gain 22. How does low dose aspirin decrease the incidence of thrombosis in at risk patients? a. the direct vasodilatory effect of aspirin b. decreased thromboxane c. decreased prostaglandin E2 d. decreased prostacyclin e. increased nitric oxide production 23. All of the following factors is LEAST commonly associated with shoulder dystocia? a. Maternal diabetes b. Fetal macrosomia c. Post term pregnancies d. Maternal obesity e. Nulliparity 24. What is the term for two cell lines with different chromosome numbers in the same individual? a. aneuploidy b. triploidy c. chimeras d. mosicism e. translocation 25. Which of the following characteristics are typical of anthropoid pelves? a. the anterior-posterior diameter of the pelvic inlet is shorter than the transverse b. most commonly seen in Orientals c. associated with occiput anterior presentations d. wide pubic arch e. long sacrospinous ligament 26. What is the most common position of the fetal vertex with a true anthropoid pelvis? a. Occiput anterior b. Occiput posterior c. Occiput transverse d. Brow presentation 27. What maternal pelvic type has the worst prognosis for vaginal delivery? a. Gynecoid b. Anthropoid c. Android d. Platyploid 28. When does the first stage of labor begin? a. the onset of regular uterine contractions b. the onset of the active phase of labor c. the onset of Braxton-Hicks contractions d. when the cervix is completely dilated e. when the fetus delivers 29. What is the most common position of the fetal vertex with a true platyploid pelvis? a. Occiput anterior b. Occiput posterior c. Occiput transverse d. Brow presentation e. Face presentation 30. Which forceps is designed to deliver the after coming head in breech presentations? a. Simpson b. Tucker-Mclane c. Piper d. Elliot e. Kielland 31. What is the immediate precursor for the production of estriol produced? a. maternal liver b. maternal adrenal gland c. fetal adrenal gland d. fetal liver e. placenta 32. How is engagement of the fetal vertex defined? a. a fetal head is no longer floating b. a fetal head whose leading edge has entered the pelvic inlet c. a fetal head whose widest part has traversed the pelvic inlet d. a fetal head that has internally rotated to an occiput anterior position e. a fetal scalp that is visible at the introitus 33. Valproic acid use in the 1st trimester is associated with what fetal anomaly? A. Neural tube defects B. Epstein's anomaly C. duodenal atresia D. Renal agenesis (Potter's syndrome) E. Cystic hygroma 34. Lithium use in the 1st trimester is associated with what fetal anomaly? a. Neural tube defects b. Epstein's anomaly c. Duodenal atresia d. Renal agenesis (Potter's syndrome) e. Cystic hygroma 35. What is the most common cause of death in live bom infants with renal agenesis? a. azothemia (renal failure) b. prematurity c. hypertension d. pulmonary hypoplasia e. associated cardiac defects 36. What is necessary for a drug to cause a fetal anomaly? a. ingestion of the drug when the organ is developing b. fetal susceptibility to the adverse effects of the drug c. the ability of the drug to cross the placenta d. the concentration of the drug in the maternal circulatory system e. all of the above 37. What is the most common first trimester ultrasound finding in a fetus with Turners syndrome? a. short extremities b. coartation of the aorta c. cystic hygroma d. webbed neck e. omphalocoele 38. Which of the following is the most common infectious cause of neonatal death? a. Herpes simplex virus b. Group B beta-hemolytic streptococcus c. Toxoplasmosis d. Varicella (chicken pox) e. Cytomegalovirus 39. Which of the following carries the highest risk for the development of neonatal herpes simplex virus (HSV) infections? a. recurrent outbreak at the onset of labor b. prolonged rupture of the membranes c. positive HSV culture 1 day prior to he onset of labor d. primary maternal infection e. positive maternal IgM antibodies to HSV 40. What is the mechanism of action of oxytocin's ability to cause uterine contractions? a. beta 2 receptor antagonism b. beta 2 receptor stimulation c. causes extrusion of calcium from the myometrial cells via calcium channels d. decreased production prostaglandin p2a e. causes release of calcium from the sarcoplasmic reticulum 41. What is the mechanism of action of ritadrine's ability to inhibit uterine contractions? a. phosphorylation of myosin b. blockade of uterine muscle alpha receptors c. increases intracellular adenosine triphosphate (ATP) levels d. increases intracellular calcium concentrations e. increases intracellular cyclic adenosine monophosphate (cAMP) levels 42. What is the most reliable method to diagnosis premature rupture of the membranes? a. vaginal pooling of fluid b. positive nitrazine test (alkaline vaginal fluid) c. positive fern test d. ultrasound evidence of oligohydramnios e. negative wet prep for vaginal infection 43. What is the karyotype of a complete molar pregnancy? a. 46XX b. 47XX +21 c. 47XX+18 d. 45X e. triploidy (69XXX or 69XXY) 44. What is the karyotype of a partial molar pregnancy? a. 46XX b. 47XX +21 c. 47XX+18 d. 45X e. triploidy (69XXX or 69XXY) 45. A 34-year-old G2P1 has a pelvic examination performed on her 1st prenatal visit at 6 weeks gestation in which a 6cm right adnexal mass is palpated. An ultrasound confirms the presence of an intrauterine pregnancy with a complex cystic and solid mass most likely originating from the right ovary. Malignancy cannot be ruled out. What is the next step in evaluating or treating this patient? a. immediate laparoscopy or laparotomy b. obtain tumor markers, and if normal observe c. laparoscopy at 14 weeks gestation d. laparotomy at 14 weeks gestation e. repeat pelvic examination and ultrasound at 14 weeks 46. If the above findings in question 45 were discovered at 16 weeks gestation, what would be the most appro priate course of action? a. immediate laparoscopy or laparotomy b. obtain tumor marker, and if normal observe c. repeat the pelvic examination and ultrasound in 4 weeks, and if the mass is still present proceed with laparotomy d. cesarean section at term with a right oophorectomy e. repeat pelvic examination and ultrasound 4 weeks post partum, and if the mass is still present pro ceed with laparotomy 47. What hormone triggers the LH surge prior to ovulation? a. GnRH b. FSH c. 17-3- estradiol d. prolactin e. testosterone 48. What is thought to cause the amenorrhea seen in female athletes? a. polycystic ovarian disease b. hyperprolactinemia c. hypothyroidism d. androgen excess e. hypothalamic dysfunction 49. Why is epidural anesthesia discouraged in patients with Eisenmenger's syndrome? a. preload of intravenous fluids can lead to pulmonary edema b. high spinal is more likely c. decreased blood pressure increases right to left shunting d. cardiac toxicity of the local anesthetic e. venous pooling can lead to decreased cardiac output Matching (50-63) Match the following histological and clinical findings with the pathologic diagnosis. Mature cystic teratoma a. Lipid cell ovarian neoplasms b. Serous tumors c. Granulosa cell tumor d. Hypersecretory endometrium e. Brenner tumor f. Kukenberg tumor Immature teratoma h. Struma ovarii i. Endodermal sinus tumor j. Clear cell adenocarcinoma k. Heterologous mixed mesodermal sarcoma I. Pseudomyxoma peritonei m. Fallopian tube carcinoma n. Walthard rests crystals of Reinke benign ovarian tumor with thyroid gland tissue hobnail nuclei psammoma bodies signet ring cells g. Cari-Exner bodies Shiller-Duval bodies ovarian tumor with primitive neural tissue sarcoma with malignant cartilage Arias Stella reaction peritoneal mucinous cell implants causing ascites hydrops tubae profluens Rokitanski protuberance 64. What condition is associated with primary amenorrhea, lack of development of secondary sexual characteris tics, and anosmia? a. Kallman's syndrome b. Mayer-Rokitansky-Kuster-Hauser Syndrome (mullerian agenesis) c. Androgen insensitivity syndrome d. Swyer's syndrome (XY gonadal dysgenesis) e. Turners syndrome (45X) 65. You have just delivered a baby whose sex cannot be easily determined because the genitalia are ambigu ous. Which of the following is the most likely cause? a. Androgen secreting tumor b. Androgen insensitivity syndrome c. Maternal 21-cc-hydroxylase deficiency d. Fetal 21- a -hydroxylase deficiency e. Fetal 17-hydroxylase deficiency 66. A 25-year-old woman presents with a 6-month history of amenorrhea and galactorrhea. A prolactin level returns 50ng/ml (normal < 20ng/ml). A lateral coned down view of the sella tursica shows enlargement of the sella tursica. Computerized tomography (CT) scan shows pituitary enlargement. What is the next step in the management of this case? a. Begin bromocriptine b. Transphenoidal hypophysectomy with removal of pituitary adenoma c. Craniotomy with removal of pituitary adenoma d. Obtain TSH levels e. Obtain oxytocin levels 67-71. Matching: Match the ovarian tumor with the hormone it secretes. Estrogen Human gonadotroping Testosteron Alpha-fetoprotein Progesterone a. Endodermal sinus tumor b. Sertoli-leydig cell chorionic tumor c. Choriocarcinoma d. Granulosa cell tumor e. none of the above 72. A15 year old woman presents with primary amenorrhea. She is 54 inches tall (4'-6"), with Tanner stage I breast and pubic hair development. Speculum and bimanual examinations confirm the presence of a normal vaginal length, and a normal cervix and uterus. Ovaries cannot be palpated. A karyotype is obtained and returns 46.XY. What is the diagnosis? a. Androgen insensitivity syndrome (testicular feminization) b. Swyer syndrome c. Mayer-Rokitansky-Kuster-Hauser Syndrome (mullerian agenesis) d. Turner syndrome (45X) e. Kallman's syndrome 73. A 25-year-old marathon runner presents with secondary amenorrhea. She failed to menstruate when given a progesterone challenge, and her FSH level returned elevated. What is the diagnosis? a. Polycystic ovarian disease b. Asherman's syndrome c. Hypothalamic amenorrhea d. Premature ovarian failure e. Gonadal dysgenesis 74. What is the next step in the management of the case described in question 73? a. begin oral contraceptive pills b. hysteroscopy c. begin bromocriptine d. advise your patient to stop long distance running e. obtain a karyotype 75. By definition, what time period is required to diagnosis a couple with infertility? a. 6 months b. 12 months c. 18 months d. 24 months e. 30 months 76. An infertility patient is found to have extensive scaring, dilation, and occlusion of both fallopian tubes. She gives no history of pelvic pain, unexplained fevers, dyspareunia, or pelvic inflammatory disease. What is the most likely diagnosis? a. Chlamydia salpingitis b. Gonococcal salpingitis c. Endometriosis d. Tuberculosis salpingitis e. In utero exposure to diethylstilbesterol (DES) 77. Condyloma lata is caused by what organism? a. Chlamydia trachomatous b. human papilloma virus c. syphilis d. gonorrhea e. Condyloma lata is not caused by an infectious agent 78. Lymphogranuloma venereum is caused by what organism? a. Chlamydia trachomatous b. human papilloma virus c. Cytomegalovirus d. calymmatobacterium granulomatis e. haemophilis ducreyi 79. What is the most important first step in the management of a newborn infant with ambiguous genitalia? a. serum electrolyte levels b. karyotype c. estradiol levels d. buccal smear e. testosterone level 80. A patient complaining of urinary incontinence is administered a cystometrogram. If the patient has genuine urinary stress incontinence, what finding would be expected? a. elevated resting bladder tone b. reduced bladder capacity c. involuntary bladder contractions d. leakage of water from the urethra 2 seconds following coughing e. none of the above 81. A 52-year-old patient presents with an abnormal papaniculau smear. Colposcopic examination shows an ascetic acid stained lesion whose biopsy confirms the presence of a squamous intraepithelial lesion. The patient requests hysterectomy because of symptomatic uterine descendsus. When would this NOT be appropriate? a. the entire lesion is visible with colposcopy b. the endocervical curettage agrees with the Pap findings c. the transitional zone is entirely visualized d. there is DNA-probe evidence of the presence of human papilloma virus in the biopsy specimen e. the pap smear agrees with tissue diagnosis 82. What is the treatment of choice for Lymphogranuloma venereum? a. penicillin G b. amoxicillin c. metronidazole d. doxycycline e. sulfamethoxizole/trimethoprin 83. Which of the following is an indication for Swanz-Ganz catherization with severe preeclampsia? a. In order to safely administer a fluid challenge if oligouria develops. b. A blood pressure of greater than 160/110 mmHg 20 minutes following a 5 mg intravenous bolus of hydralazine. c. Preeclampsia associated with severe pulmonary edema d. Preeclampsia associated with HELLP syndrome e. All are indications for Swanz-Ganz catherization 84. A patient with carcinoma of the cervix presents with a pelvic examination that is suggestive of right parame- trial involvement, but does not extend to the pelvic sidewalk Cystoscopy and proctoscopy are negative for blad der or rectal involvement. IVP shows bilateral hydronephrosis. What is the stage of this disease? a. MA b. MB c. 1MB d. Ill (urinary) e. IVA 85. What ovarian neoplasm is most commonly associated with acute hemorrhage? a. Brenner tumor b. Endodermal sinus tumor c. Benign cystic teratoma d. Serous cystadenocarcinoma e. Granulosa cell tumor 86. What is the most common malignant ovarian tumor seen in pregnancy? a. Serous cystadenocarcinoma b. Mucinous cystadenocarcinoma c. Clear cell adenocarcinoma d. Dysgerminoma e. Immature teratoma 87. Which of the following is the major drawback to low-molecular-weight heparin use in pregnant women at risk of thrombosis, or during treatment for an active thrombotic event. a. Unlike conventional heparin therapy, low-molecular-weight heparin freely crosses the placenta and places the fetus at risk for intraventricular hemorrhage during delivery. b. Low-molecular-weight heparin has a longer half-life and bioavailabilty c. Low-molecular-weight heparin has a more predictable dose-response d. Partial thromboplastin time (PIT) levels need to be followed more frequently with low- molecular-weight heparin. e. Epidural anesthesia cannot be used with patients taking low-molecular-weight heparin because of reports of epidural hematomas 88. What ovarian tumor is most sensitive to radiation therapy? a. Serous cystadenocarcinoma b. Mucinous cystadenocarcinoma c. Clear cell adenocarcinoma d. Dysgerminoma e. Granulosa cell tumor 89. What is the most common primary carcinoma metastatic to the ovary? a. breast b. malignant melanoma c. stomach d. lung e. leukemia 90. What is the most common primary carcinoma that can metastasize to the fetus? a. breast b. malignant melanoma c. stomach d. lung e. cervical 91. What is the most common genital tract malignancy? a. vulva b. vagina c. cervix d. endometrial e. ovary . 92. What genital tract malignancy is responsible for the most annual deaths? a. vulva b. vagina c. cervix d. endometrial e. ovary 93. Which is NOT a reason why radical hysterectomy is recommended over radiation therapy for cervical can cer? a. higher 5 year survival rate b. preservation of vaginal function c. preservation of ovarian function d. less long term morbidity e. all are reasons why radical hysterectomy and pelvic lymphadectomy is preferred 94. What is the most common cause of precocious puberty? a. constitutional b. craniopharyngioma c. ovarian carcinoma d. adrenal tumors e. McCune- Albright syndrome 95. What is the preferred treatment of constitutional precocious puberty? a. danocrine b. depoprovera c. GnRH agonist therapy d. continuous oral contraceptive pills e. clomiphine citrate 96. Which is a true statement concerning osteitis pubis? a. It is more commonly seen following a Burch procedure than a Marshall-Marchetti-Krantz procedure b. It can lead to vessicovaginal fistula c. Corticosteroids are not effective d. It is thought to be caused by bacterial infection e. Presents with intense pain over the symphysis pubis 97. Chancroid is caused by what organism? a. Chlamydia trachomatous b. human papilloma virus c. Cytomegalovirus d. calymmatobacterium granulomatis e. haemophilis ducreyi 98. Which of the following statements is true concerning scheduled cesarean section to prevent the vertical transmission of the human immunodeficiency virus (HIV) from the mother to the fetus. a. The ACOG committee on Obstetrical Practice concluded that HIV-infected women should be offered a scheduled cesarean section to decrease the rate of vertical transmission. b. No benefit from cesarean section is found if the mother's viral load is < 10,000 copies/ml of plasma. c. Vertical transmission rates are decreased from 5-8% to 0.5% with cesarean section d. The benefits of cesarean section are NOT affected by the presence of labor or rupture of the mem branes. e. Amniocentesis can be routinely employed to document fetal lung maturity, prior to cesarian birth. 99. Granuloma inguinal! is caused by what organism? a. Chlamydia trachomatous b. human papilloma virus c. Cytomegalovirus d. calymmatobacterium granulomatis e. haemophilis ducreyi 100. How is the diagnosis of lichen sclerosis of the vulva made? a. inspection b. culture c. wet prep d. radioimmunoassay e. biopsy 101. An 18 year old woman presents with primary amenormea. She is 68" tall (5'8") and has Tanner stage 5 (Adult contour) breast development, and Stage 2 (presexual) pubic hair growth. She is found to have a short vagina and no visible cervix. The uterus cannot be palpated on bimanual examination. Her total serum testos terone level returns 400 ng/dl (normal female < 80 ng/dl). What is the likely diagnosis? a. Androgen insensitivity syndrome (testicular feminization) b. Swyer's syndrome c. Mayer-Rokitansky-Kuster-Hauser Syndrome (mullerian agenesis) d. Turner syndrome (45X) e. Kallman's syndrome 102. An 18 year old woman presents with primary amenorrhea. She is 68" tall (5'8") and has Tanner stage 5 (Adult contour) breast development, and Stage 5 (adult) pubic hair growth. She is found to have a short vagina and no visible cervix. The uterus cannot be palpated on bimanual examination. Her total serum testosterone level returns 70 ng/dl (normal female < 80 ng/dl). What is the likely diagnosis? a. Androgen insensitivity syndrome (testicular feminization) b. Swyer's syndrome c. Mayer-Rokitansky-Kuster-Hauser Syndrome (mullerian agenesis) d. Turner syndrome e. Kallman's syndrome 103. Why do patients with Swyer syndrome (46.XY gonadal dysgenesis) have a uterus? a. Lack of androgen production b. Lack of mullerian inhibitory hormone production c. Absent testes determining region on the X-chromosome d. Absence of androgen receptors on target cells e. Swyer syndrome patients do have a uterus 104. A woman 6-weeks postpartum is evaluated for diabetes mellitus. She was diagnosed with gestational dia betes during her pregnancy. Her fasting serum glucose returns 135mg/dl. An oral 75gm glucose tolerance test is administered and the 1-hour value returns 210mg/dl (NL < 200mg/dl). The other glucose values are normal. What is the diagnosis? a. The patient does not diabetes mellitus b. The patient has impaired glucose tolerance c. The patient has diabetes mellitus type I d. The patient has diabetes mellitus type II e. Further evaluation is required to establish a diagnosis. Questions 105-106: An 18-year-old woman presents with a one-year history of increasing hair growth on her face, chest, and lower abdomen. Her testosterone returns within normal limits, but the dehydroepiandrosterone-sulfate (DHEA-sulfate) level returns significantly elevated. 105. Where is the excess androgen production being secreted to explain her hirsuitism? a. pituitary b. lung carcinoma c. adrenal gland d. ovary e. endometrium 106. She is found to have an elevated 17-hydroxyprogesterone level. What is the diagnosis? a. pituitary adenoma b. carcinoid tumor c. adult onset 21- a -hydroxylase deficiency d. ovarian carcinoma e. complete 21- a -hydroxylase deficiency 107. When ligation of the hypogastric artery is required because of massive pelvic hemorrhage, how should the right angle clamp be passed under the hypogastric artery? a. medial to lateral b. lateral to medial c. superior to inferior d. inferior to superior e. any method is appropriate 108. What is the most common symptom of ureteral injury in a post operative hysterectomy patient? a. unexplained fever b. flank pain c. ascites d. decreased urinary output e. asymptomatic 109. What is the most common cause of death from cervical cancer? a. hemorrhage b. sepsis c. renal failure d. bowel obstruction e. malnutrition 110. What is the most common cause of death from ovarian cancer? a. hemorrhage b. sepsis c. renal failure d. bowel obstruction e. malnutrition 111. Which of the following genital tract malignancies has the highest 5-year survival rate? a. squamous cell cervical carcinoma b. adenocarcinoma of the endometrium c. squamous cell carcinoma of the vulva d. ovarian carcinoma e. choriocarcinoma 112. A patient with polycystic ovarian disease presents with hirsuitism but has NORMAL testosterone and DHEA- S levels. How can this be explained? a. other androgens are elevated b. decreased sex hormone binding globulin c. decreased FSH levels d. elevated luteal hormone (LH) levels e. decreased progesterone 113. Which of the following can be used to treat hirsuitism? a. oral contraceptive pills b. dexamethasone c. cimetadine (Tagamet) d. spironolactone e. all the above 114. A patient presents with a painless vulvar lesion in which you suspect primary syphilis. What is the best method to confirm this? a. VDRL (venereal disease research laboratory) slide test b. RPR (rapid plasma reagin) c. FTA-ABS (fluorescent-labeled Treponema antibody absorption) d. Dark field examination e. TPI (Treponema immobilization test) 115. A patient who is 20 weeks pregnant presents for prenatal care. Routine laboratory studies return with a pos itive VDRL and TPHA. A review of her records indicates she had a negative VDRL 7 months earlier. She is aller gic to penicillin. What is the recommended treatment? a. Desensitization followed by treatment with 2.4 million units of Benzathine penicillin G intramuscularly. b. Desensitization followed by treatment with 2.4 million units of Benzathine penicillin G intramuscularly once a week for 3 treatments. c. Tetracycline 500mg per os 4 times a day for 15 days d. Erythromycin SOOmg per os 4 times a day for 15 days e. Azythromycin one gram per os as a single dose 116. Which medication is associated with neonatal thrombocytopenia when given prenatally to the mother? a. hydrochlorothiazide b. methyldopa (Aldomet) c. prednisone d. catapress e. diltiazem (Cardizem) 117. Which medication has long term studies to show both safety and efficacy when used to treat maternal essential hypertension in pregnancy? a. hydrochlorothiazide b. methyldopa c. propanolol d. nifedipine e. apresoline 118. Which maternal antibodies are associated with congenital heartblock? a. antinuclear antibody (ANA) b. anticardiolipin c. lupus anticoagulant d. anti-SSA(rho) antibodies e. all of the above 119. Which of the following is NOT associated with the antiphospholipid syndrome? a. prolonged activated partial thromboplastin time (aPTT) b. increased risk of hemorrhage c. habitual abortion d. intrauterine growth retardation e. stillbirth 120. A patient with chronic renal disease presents for preconceptional counseling. Which of the following findings has the worst prognosis for a successful pregnancy? a. creatinine clearance less than 50cc/min b. serum creatinine greater than 1.2mg/dl c. renal disease secondary to diabetes mellitus d. diastolic blood pressure >110mmHg e. greater than 5 grams of proteinuria in a 24 hours urine collection 121. A 30-year-old patient with systemic lupus erythematosus presents for preconceptional counseling. Which factor would best predict a successful pregnancy outcome? a. antinuclear antibody (ANA) less that 1:64 b. high C3 and C4 levels c. negative anti-DNA liter d. negative Smith antibody liter e. quiescent disease for 6 months prior to conception 122. Which of following vaccinations is contraindicated in pregnancy? a. Rabies b. Hepatitis B c. Flu d. Pneumococcal e. Mumps 123. A 32-year-old term pregnant primigravid woman presents in active labor, 5cm dilated, and 90% effaced, and zero station. Three hours later she is 6cm dilated, 90% effaced, and zero station. What is the diagnosis? a. normal progress of labor b. arrest of descent c. prolongation disorder d. protraction disorder e. cephalopelvic disproportion 124. How does a Burch procedure help improve the symptoms of urinary stress incontinence? a. restores the normal anatomical position of the urethra b. restores the normal vessico-urethral angle c. prevents cystocoele formation d. retains the intra-abdominal position of the upper urethra during valsalva maneuvers e. all of the above 125. Which drug is contraindicated in pregnancy for the treatment of asthma? a. epinephrine b. terbutaline c. aminophyllin d. corticosteroids e. azythromycin 126. What drug is contraindicated in pregnancy for the treatment of essential hypertension? a. methyldopa b. apresoline c. clonidine d. captopril e. nifedipine 127-131. Matching: Match the following hormones with their biochemical structure OH HO' Estradiol Testosterone Progesterone Estrone Estriol HO' 132. In women, when is meiosis II completed? a. the onset of puberty b. during recruitment for ovulation c. during ovulation d. following ovulation but before fertilization e. following fertilization 133. A 31-year-old woman with a history of systemic lupus erythematosus presents at 26 weeks gestation with elevated blood pressures, edema of the hands and face, and increased proteinuria. Which of the following find ings would indicate that these findings are secondary to a flare-up of systemic lupus erythematosus and not preeclampsia? a. positive ANA b. elevated serum creatinine c. decreased compliment levels d. no elevation in liver function studies e. the presence of thrombocytopenia 134. Which of the following is NOT associated with detrussor instability a. multiple sclerosis b. spinal cord injuries c. decreased bladder capacity d. urgency incontinence e. increased post void residual volumes 135. Which of the following medical conditions improves in a majority of patients during pregnancy? a. epilepsy b. systemic lupus erythematosis c. ulcerative colitis d. rheumatoid arthritis e. Crohn disease 136. What is the best method to diagnosis the presence of an enterocoele? a. speculum examination with inspection of the vaginal floor b. digital examination of the vaginal floor c. simultaneous digital examination of the vagina and rectum d. rectal examination e. vaginal probe ultrasound 137. What is the most common cause of an abnormal post coital test? a. decreased FSH b. increased estradiol c. decreased estradiol d. decreased progesterone e. the test was performed on the wrong day of the menstrual cycle 138. What is the first histological sign of ovulation? a. subnuclear vacuoles b. glandular secretions c. stromal edema d. cilia formation e. loss of nuclear mitoses 139. Which of the following would best suggest the diagnosis of the inadequate corpus luteum syndrome? a. endometrial biopsy returns dated day 22, two days before the patient's actual period b. 21-day serum progesterone of 16 ng/ml c. abnormal post coital test d. decreased temperature elevation on basal body temperature assessment e. short follicular phase 140. Which of the following can be used to treat the inadequate corpus luteum syndrome? a. oral micronized progesterone b. progesterone vaginal suppositories c. intramuscular 17-hydroxyprogesterone d. clomiphene citrate e. all of the above 141. What drug has been shown to prevent diabetic nephropathy in patients with diabetes mellitus? a. thalidamide b. diethelstilbesterol c. angiotension converting enzyme inhibitors d. beta-blockers e. calcium channel blockers 142. A woman who has a brother with cystic fibrosis is now pregnant at 8 weeks. Her husband has no family his tory of cystic fibrosis. The incidence of the carrier state in the general population in 1 in 20. What is the risk of this fetus having cystic fibrosis? a. 1 in 50 b. 1 in 120 c. 1 in 160 d. 1 in 240 e. 1 in 420 Questions 143-144: A woman 10-weeks pregnant presents with a low-grade temperature, malaise, and a "slap cheek" rash over her face. 143. What is the most likely diagnosis? a. systemic lupus erythematosus b. rubella c. rubeola d. parvovirus B-19 e. Cytomegalovirus 144. what is the risk to the fetus? a. hydrocephaly b. cardiac defects C. hydrops fetalis d. intracranial calcifications e. neural tube defects 145. What is the immediate precursor for the production of estriol? a. 17-hydroxyprogesterone b. dihydroepiandrosterone (DHEA) c. dihydroepiandrosterone - sulfate (DHEA-S) d. 16-hydroxy-dihydroepiandrosterone (16-OH-DHEA) e. 16-hydroxy-dihydroepiandrosterone - sulfate (16-OH-DHEA-S) 146. Which maternal congenital heart lesion carries the highest maternal mortality risk? a. Eisenmenger's syndrome b. Tetralogy of Fallot c. Mitral stenosis d. Patent ductus arteriosus e. Aortic stenosis 147. A 42-year-old patient presents with a one-month history of menstrual irregularities, increasing hair growth on her face, chest, and lower abdomen, deepening of her voice, clitoral enlargement, and male pattern bald ness. Laboratory data shows a serum testosterone level of 400ng/dl (NL = <60ng/dl), normal DHEA-S levels, and normal 17-hydroxyprogesterone levels. What is the most likely diagnosis? a. Polycystic ovarian disease b. Adult onset adrenal hyperplasia c. Androgen secreting ovarian neoplasia d. Androgen secreting adrenal tumor e. Gushing syndrome 148. Which statement below best describes the partial pressure of oxygen (pO2) in the right compared with the left ventricles of the fetus in utero? a. The pO2 on the right is greater than the left b. The pO2 on the left is greater than the right c. The pO2 on the left is equal to the right d. During uterine contractions the pO2 on the right is greater than the left 149. What is the treatment of choice for labial agglutination? a. Topical estrogen cream b. Topical steroid cream c. Oral estrogen d. Oral progesterone e. GnRH agonists 150. What is the mechanism for the onset of follicular development at puberty? a. The growth spurt increases FSH levels b. The maturation of the positive feedback of estradiol to trigger the LH surge c. The level of estrogen required to inhibit FSH increases d. The level of growth hormone increases e. Increased androgens from the adrenal gland 151-155: Matching Match the homologous structures in the male and female Phallus a. vagina Prostate gland b. clitoris Cowpers (bulbo-urethral) gland c. Labia majora Scrotum d. Skene's ducts None e. Bartholin glands 156. .A 25-year-old-gravida 1 para 1 presents for preconceptional counseling. Her first pregnancy was compli cated by fetal anencephaly. Which of the following options should be offered to this woman? a. Folic acid 4 mg per day beginning one month prior to conception until 12 weeks gestation b. Maternal serum alpha-fetoprotein screening at 15-weeks gestation c. Amniocentesis for amniotic fluid alpha-fetoprotein levels d. Amniotic fluid acetylcholinesterase screening if the amniotic fluid alpha-fetoprotein level is elevated. e. all of the above can be offered 157. A 41- year-old gravida 2 para 1 presents at 15 weeks for genetic counseling and possible amniocentesis. Her first pregnancy at age 36 was complicated by fetal trisomy 18. Which of the following best describes the risk of this fetus having aneuploidy compared to women her same age, with no history of aneuploidy? a. The risk for aneuploidy is not different b. The risk for aneuploidy is doubled c. The risk is for aneuploidy is tripled d. The risk for aneuploidy is slightly increased e. The risk for aneuploidy is slightly decreased 158. A 30-year-old woman presents with primary infertility. During the taking of her history and physical examina tion you discover she has experienced a rapid weight gain and lack of energy over the past 6 months. Her rest ing pulse is 55 beats per minute. You suspect hypothyroidism. Which of the following tests should be ordered? a. Total T4 b. Total T3 c. Resin T3 uptake d. TSH e. All the above 159. A woman with adult onset adrenal hyperplasia being treated with dexamethasone, presents 5 weeks from her last menstrual period. Her urine pregnancy test is positive. Her husband has a sister who was born with ambiguous genitalia. Which of the following should be offered to this patient? a. Discontinue the dexamethasone immediately b. Taper the dexamethasone gradually and then discontinue c. Perform a ACTH stimulation test on her husband d. Double her dose of dexamethasone e. Wait for fetal sex determination to determine if treatment is needed. 160. What is the final hormone event of puberty that brings about the onset of ovulation? a. The set point to inhibit FSH secretion by estrogens is increased b. Maturation of the positive feedback of estradiol to induce the LH surge c. Epiphyseal plate closure triggers ovulation d. The adrenal gland begins secretion of androstenedione e. Androgen production by the ovary begins. 161. Who should NOT receive the annual flu vaccine? a. Sickle cell anemia patients b. Pregnant patients c. Patients over 50-years-old d. Immuno-compromised patients e. All of the above patients should receive the flu vaccine 162. Risk groups for invasive group B streptococcal disease includes all EXCEPT: a. Neonates b. Diabetics c. Pregnant women d. Patients positive for the human immunosuppressive virus (HIV) e. All of the above are at risk 163. Risk factors for early onset streptococcal newborn disease include which of the following? a. Low maternal CD4 count b. Prior low birth weight infant c. Maternal GBS bacteruria d. Post-term delivery e. Fetal macrosomia 164. A 24-year-old gravida 2 para 1 has a Pap smear performed on her 1st prenatal clinic visit at 10 weeks ges tation. The results return in 2 weeks as "high grade squamous intraepithelial lesion indicating possible invasive cancer". Colposcopically directed biopsies return "microinvasive squamous cell carcinoma" What is the next step in the management of this patient? a. Cold knife conization of the cervix b. Termination of the pregnancy followed by cold knife conization c. Termination of the pregnancy followed by intra-fascial hysterectomy d. Termination of the pregnancy followed by extra-fascial hysterectomy e. Allow the pregnancy to go to term, followed by an extra-fascial hysterectomy 8 weeks post partum. 165. How soon after initiating chemoprophylaxis for GBS is the primary benefit found? a. <1 hour b. >1-2 hours c. >2-4 hours d. >4-8 hours f. >8-12 hours 166. Common pathogens for amnionitis include all of the following EXCEPT: a. E coli b. Klebsiella c. Group B streptococci d. Listeria monocytogenes e. Bacteriodes 167. Which of the following is not associated with Chorioamnionitis? a. Abnormal labor b. Newborn sepsis c. Abruption d. Fetal tachycardia e. Uterine atony 168. Which of the following occurs more frequently in multiple gestation when compared with singleton pregnan cies? a. Pyelonephritis b. Maternal acidosis c. Placenta! abruption d. Fetal macrosomia e. Chronic hypertension 169. Division of the blastocyst between the 4-8th day is associated with what type of twinning? a. Diamniotic, dichorionic b. Diamniotic, monochorionic c. Monoamniotic, dichorionic d. Monoamniotic, monochorionic e. Conjoined 170. The mean gestational age for delivery of triplets is: a. 38 weeks b. 36 weeks c. 34 weeks d. 32 weeks e. 30 weeks 171. Which of the following is not true concerning abortions in the United States? a. There are approximately 1.25 million abortions in the United States yearly. b. Women having abortion are predominately white, young and single. c. Catholics are less likely to have an abortion d. More that half of women seeking abortion was using contraception in the month of their pregnancy. e. One third have condom failure as source of their pregnancy. 172. Complications of first trimester vacuum curettage include which of the following? a. infection b. Ectopic pregnancy c. Incomplete abortion d. Allergic reaction to medications e. All of the above 173. Which of the following is not a part of the "Post Abortion Triad"? a. Pain b. vasovagal reaction c. Bleeding d. infection 174-179. Matching: Match the following procedures with the common complications. High dose Oxytocin Saline termination Hysterotomy & Hysterectomy Methotrexate & Misoprostol Mifepristone & Misoprostol Misoprostol a. Highest morbidity and mortality. b. High success rate for first trimester termination. c. Used in all trimesters for cervical ripening. d. Highest success rate in first trimester abortions worldwide e. Increased morbidity from incomplete abortions and DIG. f. Less effective with increased risk of water intoxication. 180. Support of level I and II includes all of the following structures except: a. Arcus tendineus attachment of the pubocervical fascia. b. Uterosacral cardinale complex. c. Denonvilliers fascia and its attachments d. Round ligament e. Levator ani 181. When Hyskon is absorbed systemically, which of the following is the least common complication? a. Hyponatremia b. Pulmonary Edema c. Encephalopathy d. adult respiratory distress syndrome e. Disseminated intravascular coagulation 182. For the following POPQ values select the appropriate answer. Aa=+1, Ba=+1, C=-4, Ap=+2, Ap=+2, GH=5, PB=4, TVL=9 a. Stage II cystocele, Stage I uterine prolapse, Stage III rectocele b. Stage II cystocele, Stage II vault prolapse, Stage II rectocele c. Stage II cystocele, Stage I vault prolapse, Stage III rectocele d. Stage III cystocele, Stage II vault prolapse, Stage III rectocele 183. Which of the following procedures gives NO apical support of the vagina? a. Sacrospinous ligament fixation b. McCall culdopasty c. Perineorrhaphy d. Sacrocolpopexy e. Paravaginal repair 184. The rectovaginal septum (Denonvilliers) attachments include all of the following except a. The cul de sac b. Perineal body c. The levator ani posterior to the arcus tendineus d. The sacrospinous ligament e. The uterosacral ligaments 185. Which of the following statements are not true concerning an enterocele. a. An enterocele has pelvic peritonium in direct contact with vaginal epithelium. b. Enterocele exists when the pubocervical fascia and/or rectovaginal septum is not in continuity with the uterosacral ligament complex. c. Ligation of the enterocele sac will prevent enterocele. d. It is difficult to distinguish between a high rectocele and an enterocele clinically 186. Costs are lower with which type of hysterectomy? a. Abdominal hysterectomy b. Laparoscopically assisted c. Supracervical hysterectomy d. Pelviscopic intrafascial hysterectomy e. Vaginal hysterectomy 187. Postoperative hemorrhage is most common with which operative procedure? a. Endometrial Ablation b. Abdominal Hysterectomy c. Supracervical Hysterectomy d. LAVH e. Vaginal Hysterectomy 188. Advanced paternal age increases which of the following autosomal dominant disorders? a. Neurofibromatosis b. Achondroplasia c. Apert syndrome d. Marfan syndrome e. All of the above 189. Advanced paternal age increases the risk of X-linked disorders in which of the following cases? a. Transmission to affected male off-spring b. Transmission to affected female offspring. c. Transmission to his daughter (becomes a carrier) and then to an affected grandson ("grandfather effect") d. Transmission to his daughter (becomes a carrier) and then to an affected granddaughter ("grand daughter effect") e. None of the above. 190. According to the National Institutes of Health Consensus Panel Recommendations, which of the following describes their recommendations concerning antenatal steroids in the presence of premature ruptured mem branes (PROM)? a. antenatal steroids should be offered if less than 32-34 weeks of pregnancy b. antenatal steroids should be offered if less than 30-32 weeks of pregnancy c. the presence of chorioamnionitis is NOT an absolute contraindication for antenatal steroids with PROM d. antenatal steroids are offered in this setting to decrease the incidence and severity of respiratory dis tress syndrome of the newborn. 191. Which of the following is a true statement concerning bacterial vaginosis (BV) infections in pregnancy? a. Large clinical trials have failed to find an association between BV and preterm birth. b. Oral and topical metronidazole therapies are equally effective for the treatment of BV in pregnancy. c. ACOG currently recommends screening all pregnant patients for BV. d. Screening for BV should only be considered in patients at risk for preterm birth. e. BV has been identified as a causative factor for preterm birth. 192. What is the risk of hepatitis C transmission to the baby of an infected pregnant woman? a. There is no significant transmission risk b. 5-6% c. 10-16% d. 25-31% e. 35-41% 193. Which statement is true concerning recommendations about breastfeeding and maternal hepatitis C infec tions? a. Infected women can breastfeed their newborn infants. b. There is a slightly, but statistically significant higher rate of transmission in breastfed infants com pared with bottle-fed infants of hepatitis C infected mothers. c. Unlike HIV infections, the viral load for hepatitis C plays no role in the risk of transmission. d. There are no studies concerning breastfeeding Vs bottle-feeding, and thus breastfeeding should be avoided until more is known. 194. What is the estimated percentage of breast and ovarian cancers that are attributable to the BRCA 1 or BRCA 2 mutation? a. 5-7% b. 5-17% c. 25-27% d. 40% e. 50% 195. How are the BRCA mutations transmitted to offspring? a. Autosomal dominant b. Autosomal recessive c. X-linked recessive d. X-linked dominant e. Multi-factorial 196. What is the lifetime risk of a woman with the BRCA 1 mutation developing breast cancer? a. 20% b. 40% c. 60% d. 80% e. 100% 197. A 30-year-old woman presents with a family history of breast cancer in her maternal grandmother and mother who are both deceased from their disease. Her 33-year-old sister was recently diagnosed with breast cancer. She requests BRCA screening. Which of the following statements is MOST correct concerning BRCA screening in this patient? a. She is a candidate for screening immediately, but only after genetic counseling and informed consent. b. She is a candidate for screening, but the affected sister should be screened first, to determine which, if any mutation is present in this family. c. This patient should be discouraged from screening because her insurance carrier would consider this a preexisting condition if she tests positive and not cover future expenses if she develops breast can cer. d. This patient should be discouraged from screening because therapeutic options for carriers have not been established. e. BRCA screening has not yet been proven to be beneficial or cost-effective, and currently should not be offered to any patient. 198. What is the lifetime risk of a woman with the BRCA 1 mutation developing ovarian cancer? a. 25% b. 45% c. 65% d. 85% e. 100% 199. Which of the following conditions cannot be reliable diagnosed by chorionic villus sampling? a. Fragile X syndrome b. Down syndrome c. Turner syndrome d. Cystic fibres is e. Sickle cell disease 200. Which of the following statements is true concerning cystic fibrosis (CF) carrier screening? a. The National Institutes of Health Workshop on Population Screening for CF recommended screening all ethnic groups at risk for CF even without a family history. b. If one or both of the parents screening test returns negative, there is no risk of delivering a child with CF. c. Screening can be offered to patients with a family history of CF d. The most common mutation associated with CF is the AF208 mutation. e. Screening has not been proven reliable and should not be offered to any patient at this time. 201. Which of the following conditions is (are) associated with an increased risk for thrombosis. a. The antiphospholipid antibody syndrome (Lupus anticoagulant syndrome) b. Factor V Leiden mutation c. Protein C deficiency d. Protein S deficiency e. All are associated with an increased risk for thrombosis. 202. Which of the following is the main benefit for obtaining the fetal fibronectin test? a. It identifies asymptomatic patients who will experience a preterm birth. b. It identifies symptomatic patients at risk for preterm birth with high sensitivity and specificity. c. It can identify patients who will not deliver within the next 2 weeks, even if they are symptomatic. d. Identifies which patients require long-term oral or subcutaneous tocolytic therapy. e. None of the above statements are correct. 203. Fragile X syndrome is an X-linked disorder and only males are affected. a. True b. False Questions 204-206: Nearly all cases of the fragile X syndrome involve expansion of a triplet repeat consisting of the bases cytosine-guanine-guanine on the X chromosome. The number of repeats determines the severity of the disease. 204. Below which number of triplet repeats will the individual be unaffected and not at risk for transmission to their off spring? a. 25 b. 50 c. 100 d. 200 e. 300 205. What range of triplet repeats needs to be present for a woman to be considered to have a premutation (unaffected, but capable of transmission)? a. 25-50 b. 50-200 c. 200-300 d. 300-400 e. 400-600 206. Above which number of triplet repeats will the individual be affected by the fragile X syndrome? a. 25 b. 50 c. 100 d. 200 e. 300 207. Which of the following radiation exposures is the threshold for the increased risk for abortions, congenital anomalies, or growth restriction with x-ray or nuclear medicine studies? a. 5 mrad b. 1 rad c. 5 rad d. 10 rad e. 15 rad 208. Which of the following statements is correct about the use and safety of magnetic resonance imaging (MRI) in pregnancy? a. The National Radiological Protection Board advises against MRI use in all pregnant women regard less of their gestational age even though there have been no adverse reports. b. Adverse fetal effects have been reported with MRI use in pregnancy and should not be used. c. The adverse fetal effects are caused by the high ionizing radiation doses with MRI. d. MRI has not been shown to cause fetal harm, and has been shown to be useful in diagnosing fetal central nervous system anomalies, and IUGR. e. The National Radiological Protection Board states MRI can be used in the first trimester 209. Which nuclear medicine diagnostic study or treatment is contraindicated in pregnancy? a. Iodine 131 for the treatment of Graves disease. b. Diagnostic Iodine 123 in a patient with a thyroid nodule suspicious for thyroid cancer. c. Ventilation-perfusion scans for the diagnosis of pulmonary embolism. d. Technetium Tc 99m for brain, bone, renal, or cardiovascular scans. e. All can be given in pregnancy without fetal consequences. 210. Which of the following is a true statement concerning the ACOG committee opinion concerning home uter ine activity monitoring (HUAM). a. HUAM may be useful, but only in patients with the highest risks for preterm birth. b. HUAM has never been shown to be a benefit in prolonging pregnancy or preventing preterm birth, and therefore cannot be recommended. c. ACOG does not recommend HUAM primarily because of the high cost. d. HUAM may be useful in patients with documented contractions that cannot be perceived by the patient. 211. Which of the following patients may be considered for incidental appendectomy (when no pathology is seen) at the time of laparotomy or laparoscopy? a. Women between 10-30 years being evaluated for right lower quadrant pain. b. Women with a previous history of Crohn disease. c. Women with prior radiation therapy. d. Women with the presence of vascular grafts. e. Incidental appendectomy should not be considered in any patient. 212. Which of the following statements are true concerning ACOG's committee opinion concerning labor induc tion with misoprostol (Cytotec). a. Misoprostol may be used with caution in patients with a history of cesarian section, if they qualify as a candidate for VBAC. b. Misoprostol is less effective than Prostaglandin E2 preparations for cervical ripening. c. Misoprostol has proven to decrease the cesarean delivery rate. d. Misoprostol given 25mcg every 3 hours reduces the rate of uterine hyperstimulation compared with a 50 meg every 4 hour dosing schedule. e. The committee recommends the 25-mcg dose initially. 213. Which is an improper use of laparoscopically assisted vaginal hysterectomy (LAVH). a. LAVH may be used when vaginal hysterectomy would otherwise be impossible. b. LAVH may be used for lysis of adhesions c. LAVH may be used for the treatment of pelvic endometriosis. d. LAVH can be used when removal the ovaries are also desired. e. Physicians inexperienced with vaginal hysterectomy can use LAVH. 214. A pregnant woman presents at term for induction of labor with oxytocin because of a history of a T-4 spinal cord transection and an inability to perceive labor. While on 10 mu/min of oxytocin she suddenly develops a severe headache, sweating, flushing, and nasal congestion. Her BP, which was normal until now is markedly elevated. She is also found to have an irregular pulse of 45 beats per minute and the fetal monitor shows new onset of late decelerations. What is the most likely diagnosis? a. Preeclampsia with impending eclampsia b. Hypersensitivity reaction to oxytocin, which is commonly seen in spinal cord injury patients. c. Autonomic dysreflexia d. Undiagnosed pheochromocytoma e. None of the above. 215. What is the most important aid to diagnosis premenstrual syndrome? a. The initial history showing symptoms only in the second half of the menstrual cycle. b. Evaluation a daily symptom diary filled out prospectively by the patient. c. Basal body temperature charts d. Endometrial biopsy e. Menstrual cycle day 21 serum progesterone. 216. Which of the following treatments has no proven efficacy for the treatment of premenstrual syndrome? a. Exercise b. Fluoxetine (Prozac) c. Progesterone d. Spironolactone e. Vitamin B6 217. A 15-year-old patient presents for a routine examination. During your history taking she exhibits signs and symptoms of depression. After some hesitation, she admits that she has recently thought of suicide. She asks you not to tell her parents about this. What is the most appropriate way to manage this case? a. Accept her wish and not tell her parents, so your relationship with her will not be compromised, but begin antidepressant therapy. b. Do not tell her parents, but make an appointment for her to see a child psychiatrist the following day. c. Insist that she tell her parents herself. d. Inform her that this information cannot be kept secret, and inform her parents so that they can take steps to remove firearms, medications and other potentially harmful objects from the home. e. Only inform the parents if you feel she was really serious about committing suicide. 218. In a non-pregnant woman with a high risk factor for the development of diabetes [history of gestational DM, obesity, hypertension, 1st degree relative with DM, member of a high-risk ethnic group (African-American, Hispanic, Native American)], how often should you obtain a fasting glucose to screen for DM? Yearly a. Every other year b. Every 3 years c. Every 5 years d. Every 10 years 219-222: Matching: Match the most common cause of death in women for each age group. Age 13-19 A. Heart Disease Age 19-39 B. Cancer Age 40-64 C. Accidents Ages 65 and greater D. Cerbrovascular accidents E. Diabetes mellitus 223. After how many consecutive adequate and normal Pap tests can reduced frequency of screening be offered in low-risk patients? a. 2 b. 3 c.4 d. 5 e. One should never recommend reducing the frequency of Pap tests 224. What are the current recommendations for lung cancer screening in high-risk patients? a. Yearly chest x-rays b. Yearly chest examinations (auscultation and percussion) c. Yearly sputum cytology screenings d. There are no effective screening methods to diagnosis lung cancer 225. What are the current recommendations for colorectal cancer screening and prevention? a. Low-fat diet b. Yearly digital rectal examinations after age 50. c. Sigmoidoscopy every 3-5 years after age 50. d. Yearly stools for occult blood after age 50. e. All of the above. 226. Mammography has been proven to reduce the death rate from breast cancer above what age? a. 35 b. 40 c. 50 d. 60 e. Mammography has never been shown to lower death rates in any age group. 227. Which medication has been shown to reduce the rate of breast cancer in high-risk patients? a. tomoxifen b. GnRH agonist therapy c. raloxifene d. medroxyprogesterone acetate e. None of the above 228. Which of the following statements is correct concerning Apgar scores, perinatal asphyxia, and cerebral palsy? a. Low initial Apgar scores (1 and 5 minutes) increase the risk for cerebral palsy and this proves perina tal asphyxia was the cause. b. Low Apgar scores are usually caused by perinatal asphyxia. c. The Apgar score was developed as a method to quickly evaluate the clinical status of the newborn. d. Cerebral palsy has never been linked with perinatal asphyxia. e. Low Apgar scores (0-3) even after 20 minutes does not increase the risk of poor future neurological development. 229. What level best defines clinically significant acidemia from an umbilical artery pH assessment? a. <7.25 b. <7.20 c. <7.15 d. <7.10 e. <7.00 230. Normal umbilical artery pH values exclude intrapartum hypoxemia as a proximate cause of neonatal depression. a. True b. False 231. What is the vertical transmission rate of human immunodeficiency virus (HIV) from mother to fetus without zidovudine prophylaxis? a. 5-8% b. 25% c. 40% d. 60% e. 90% 232. What is the rate of vertical transmission of human immunodeficiency virus (HIV) from mother to fetus with zidovudine prophylaxis? a. 1-3% b. 5-8% c. 25% d. 40% e. 60% 233. Which of the following findings would mandate in-patient intravenous antibiotic treatment for suspected pelvic inflammatory disease. a. Multiparous patient b. Rebound tenderness in the lower abdomen c. Allergy to penicillins d. Presence of an IUD e. The presence of cervical motion tenderness 234. Which of the following presents with a painful lesion of the vulva? a. Chancroid b. Syphilis c. Condyloma accuminata d. Lymphogranuloma Venereum e. Granuloma Inguinal! 235. What is the preferred and most cost-effective treatment for Gonococcal cervicitis? a. Ceftriaxone 125mg IM times one b. Cefixime 400mg PO times one c. Ciprofloxacin SOOmg PO times one d. Ofloxacin 400mg times one. e. All of the above are equally preferred and cost effective. 236. Which of the following medications is NOT effective for the treatment for Chlamydia cervicitis? a. Doxycycline 100mg PO twice a day for 7 days b. Azithromycin 1gm PO times one c. Erythromycin SOOmg PO 4 times a day d. Ofloxacin SOOmg PO twice a day for 7 days e. All of the above are effective treatments. 237. Which of the following is NOT effective for the treatment of recurrent genital herpes simplex virus infections of the vulva? a. Oral acyclovir b. Oral famcoclovir c. Oral famciclovir d. Topical acyclovir e. All of the above are effective treatments. 238. A non-pregnant patient presents with a palmar rash, which you suspect represents syphilis. You successful ly treated her for syphilis 3 years earlier. Which of the following findings would confirm your suspicion? a. Positive rapid plasma reagin (RPR) b. Rising quantitative VDRL liters c. Positive FTA-ABS d. Positive MHA-TP e. All of the above findings indicate a repeat infection 239. How is microinvasive carcinoma of the vulva defined? a. Depth of invasion < 1mm b. Depth of invasion < 3mm c. Depth of invasion < 5mm d. The lesion is < 2cm in diameter e. There is no microinvasive category for cancer of the vulva. Questions 240-241: A 53-year-old woman has a 3cm lesion of the right vulva. It is located in the mid-labia majora, 4cm from the clitoris. The biopsy returns invasive squamous cell carcinoma. There are no palpable inguinal nodes. 240. What is the stage of this cancer? a. Her stage cannot be determined since vulvar cancer is surgically staged. b. Stage I. c. Stage II. d. Stage III. e. Stage IV. 241. What is the preferred treatment for this lesion? a. Radical vulvectomy and bilateral superficial and deep inguinal node dissection. b. Radical local excision, with no inguinal node dissection. c. Radical local excision with bilateral inguinal node dissection. d. Radical local excision with ipsilateral inguinal node dissection. e. Hemi-vulvectomy with ipsilateral inguinal node dissection. 242. An exploratory laparotomy is performed for suspected ovarian cancer. Ascitic fluid is sent for cytology, a total abdominal hysterectomy, bilateral salpingoophorectomy, omentectomy, retroperitoneal lymph node sam pling, and removal of a 1cm nodule from the surface of the liver. All specimens return positive for serous cys- tadenocarcinoma of the ovary. What is the stage of her disease? a. IVA. b. IVB. c. IMA d. 1MB. e. IMC. 243. Which is NOT a risk factor for endometrial cancer? a. Tamoxifen b. Smoking c. Early menarche d. Nulliparity e. Estrogen replacement therapy Questions 244-245. Following a staging laparotomy the following pathology results are obtained. Peritoneal washings = negative; cervix = no evidence of malignancy ; uterine corpus = adenocarcinoma grade I invading over half the myometrium, no serosal involvement, ovaries = no evidence of malignancy; pelvic and paraaortic lymph node sampling = no evidence of malignancy. 244. What is the stage of her disease? a. IAG1 b. IB G1 c. 1C G1 d. IIAG1 e. IIB G2 245. What is the next step in the management of this patient? a. No further treatment is required. b. External radiation therapy c. Single agent chemotherapy d. Multi-agent chemotherapy e. Tamoxifen Questions 246-247. A patient has a cold-knife conization pathology specimen return invasive squamous cell carcinoma. An examination under anesthesia is negative for parametrial involvement. There is no evidence of spread to the vagina. Cystoscopy and protoscopy are negative. The intravenous pyelogram is normal. She is staged as a Stage IB cervical carcinoma. The patient elects for radical surgery. However, at surgery, a suspicious paraaortic node is palpated prior to performing the radical hysterectomy. A frozen section confirms the node to be positive for squamous cell carcinoma. 246. How should this case be managed? a. Radical hysterectomy, pelvic and paraaortic node dissection. b. Radical hysterectomy, pelvic and paraaortic node dissection, with post-operative radiation therapy. c. Cancel the planned surgery and perform external beam radiation therapy. d. Cancel the planned surgery and perform intracavitary brachytherapy plus external beam radiation therapy. e. Simple hysterectomy followed by external beam radiation therapy. 247. What is the correct stage of her disease? a. IB. b. MIA. c. IIIB. d. IVA. e. IVB. Questions 248-255. Matching: Match the chemotherapeutic agent with its common adverse effect. Tamoxifen (Nolvadex) a. Peripheral neuropathy Paclitaxel (Taxol) b. mucosal ulceration Methotrexate c. Cystitis Bleomycin (Blenoxane) d. Cardiac toxicity Vincristine (Oncovin) e. renal toxicity Cis-dichlorodiammine (Cisplatin) f. pulmonary fibrosis Cyclophosphamide (Cytoxan) g. retinopathy Doxorubicin (Adriamycin) h. hypersensitivity reactions All of the following conditions are associated with a heavy vaginal discharge containing numerous WBC's except: a. Candidiasis b. Desquamative Vaginitis c. Atrophic Vaginitis d. Foreign body vaginitis e. Trichomoniasis 257. All of the following organisms may be found with bacterial Vaginosis except: a. Mobiluncus species b. Listeria monocytogenes c. Bacteroides species d. Peptococcus e. peptostreptococcus 258. Sex partners of females affected by which of the following conditions should be treated : a. Bacterial Vaginosis b. Vaginitis emphysematosa c. Candidiasis d. Trichomoniasis e. Foreign body Vaginitis 259. Which of the following vaginal entities increases the incidence of vaginal and cervical dysplasia: a. Epithelial polyps b. Leiomyoma c. Gartner's duct cyst d. Endometriosis e. Vaginal adenosis 260. The least common symptom associated with suburethral diverticulum is: a. Dysuria b. Urinary frequency c. Urinary urgency d. Dyspareunia e. Post-mictural dribbling 261. Which of the following disease entities may be associated with desquamative vaginitis: a. Lichen sclerosus b. Crohn's disease c. Behcet's syndrome d Lichen planus e. Lichen simplex chronicus 262. Which of the following conditions affects the apocrine glands of the vulva? a. Psoriasis b. Erythrasma c. Fox-Fordyce disease d. Seborrheic dermatitis e. Lichen planus 263. Crohn's disease of the vulva may be helped by which of the following medications? a. Erythromycin b. Metronidazole c. Clindamycin d. Augmentin e. Doxycycline 264. Which of the following disease entities may cause oral and vulvar ulcerations and inflammatory vasculitis of the eye? a. Hailey-Hailey disease b. Pemphigus vulgaris c. Behcet's disease d. Darier disease e. Tuberculosis 265. Which of the following vulvar lesions may be confused with adenocarcinoma: a. Syringoma b. Ectopic breast tissue c. Endometriosis d. Neurofibroma e. Hidradenoma 266. Which of the following is not an indication for pelvic ultrasound? a. Evaluation of uterine anomalies b. To determine who requires an endometrial biopsy in a post-menopausal patient with bleeding sus pected of having endometrial atrophy. c. Guidance for interventional procedure d. Location of IUD 267. A hemorrhagic cyst can persist for as long as six months: a. True b. False 268. Which of the following is seen with ultrasound imaging in the presence of ovarian torsion? a. The ovary is normal sized b. Color Doppler shows increased flow c. The right ovary is more commonly involved d. A pre-existing mass is more often malignant 269. A 66 year old female presents in November for a general check-up, lacking any known immunizations and otherwise healthy. She should be offered which of the following? a. Flu vaccine b. Pneumovax c. Td toxoid d. Hepatitis A vaccine e. Choices A, B, and C 270. Hepatitis B vaccine is indicated for which of the following conditions: a. Hepatitis C patients b. Partners of those with Hepatitis B c. End stage Renal Disease patients d. Healthcare workers e. All of the above 271. During fetal life, serum concentrations of FSH and LH reach adult levels at mid-gestation. a. True b. False 272. By mid-to-late puberty the maturations of the positive feedback relationship between estradiol and FSH is established. a. True b. False 273. Sexual precocity is associated with premature menopause and adult short stature. a. True b. False 274. The most common form of sexual precocity in females is idiopathic. a. True b. False 275. GnRH agonist treatment is effective for McCune-Albright syndrome or noncentral forms of precocious puberty. a. True b. False 276. The highest incidence of ectopic pregnancies occurs in women aged: a. 15-19 b. 20 - 24 c. 25 - 29 d. 30 - 34 e. 35 and older 277. All of the following are contraindications to medical treatment of ectopic pregnancy with methotrexate except: a. Renal Disease b. Immunodeiciency c. Ectopic mass of 3 cm d. Hepatic Disease e. Breast feeding 278. Recurrent ectopic pregnancy rates are highest for which of the following surgical managements of ectopic pregnancy: a. Fimbrial Evacuation b. Salpingostomy c. Salpingectomy d. Unilateral Salpingoophorectomy e. Segmental Resection of Fallopian Tube 279. Which of the following statements is NOT true with regards to ovarian cancer: a. Ovarian cancer is the most common malignancy affecting women worldwide. b. The majority of ovarian cancer patients are postmenopausal. c. Ovarian cancer cause more deaths than any other malignancy affecting the female pelvis. d. 30% of ovarian masses in postmenopausal women are malignant. 280. Which of the following factors, when present, does NOT increase a woman's risk of developing ovarian cancer. a. low parity b. Caucasian race c. A family history of ovarian cancer d. Chronically elevated gonadotropins e. Rare or infrequent ovulation (oligo-ovulation) 281. The majority of ovarian cancer patients have pathology consistent with: a. Mucinous tumors b. Serous tumors c. Endometriod tumors d. Clear cell tumors e. Brenner tumors 282. When a proper staging laparotomy is performed for a patient with ovarian cancer, the most likely site where metastatic disease will be found is: a. Diaphragm b. Omentum c. Para-aortic lymph nodes d. Obturator lymph nodes e. Common iliac lymph nodes 283. With proper surgical staging, approximately 30% to 40% of apparent Stage I cases will be upstaged to a higher stage. a. Jure b. False 284. Gestational trophoblastic neoplasia includes a spectrum of trophoblastic diseases, including all except: a. Hydatidiform b. Invasive mole c. Endodermal sinus tumor d. Choriocarcinoma 285. Risk factors for hydatidiform mole include all except: a. Nutritional deficiency of animal fat or fat soluble vitamin carotene b. History of prior spontaneous abortions c. Professional occupation d. Age between 20 and 30 e. History of previous molar pregnancy 286. Which of the following statements is incorrect with regard to the management of hydatiform mole? a. Suction curettage is the management method of choice b. Chemotherapy should be started immediately if the hCG rises or plateaus or if matastatic disease is detected at any time. c. Hysterectomy is never an option in the primary management of hydatiform mole. d. After a moderate amount of tissue is removed, a Pitocin drip should be started. e. Chest x-rays should be taken initially, but should only be repeated if the hCG titer plateaus or rises. 287. In order to make a diagnosis of hydratiform mole or gestational trophoblastic neoplasia, it si always manda tory to perform a biopsy. a. True b. False 288. A newborn after 37 complete weeks may be: a. small for gestational age (SGA) b. Adequate for gestational age (AGA) c. Growth restricted d. All of the above 289. Causes of growth restriction may be: a. Infection b. Genetics c. Nutritional d. None of the above e. All of the above 290. Which of the following is an environmental toxin? a. Maternal diabetes b. Oligohydramnios c. Smoking d. None of the above 291. Antepartum fetal assessments for growth retardation include: a. Biophysical profile b. Estriol (Urinary) c. TSH levels d. Fetal fibronectin studies 292. Intrauterine restricted growth can produce what problem or problems in the newborn? a. Neonatal asphasia b. Hypolglycemia c. Future learning defects d. All of the above e. None of the above 293. The prerequisite(s) for a forceps delivery is (are): a. Operative skill b. Presenting part engaged c. Cervix completely dilated d. All are correct 294. What factor increases morbidity when delivery is accomplished with forceps? a. Maternal Weight b. Abnormal position c. Fetal cardiac abnormalities d. Trisomy 21 fetus 295. Bird's Safety Rules for Vacuum delivery of a fetus include(s): a. Cup must not be applied more than twice b. Head, not just the scalp, must advance c. The head should be delivered within 15 minutes d. None of the above e. All of the above 296. A fetal complication that may cause death from a vacuum delivery is: a. Scalp laceration b. Conjunctiva! hemorrhage c. Subgaleal Hematoma d. Poor maternal cooperation 297. A 22 year old sexually active women presents to you for routine gynecologic care. She has no physical complaints and her last PAP smear was 2 years ago. She has had one sexual partner over the past 2 years and currently is on oral contraceptives. She reports infrequent use of condoms. She has no past history of abnormal PAP smears. Which of the following is appropriate management of this patient? a. Routine pelvic examination with PAP smear. b. Routine pelvic examination alone. c. Routine pelvic examination, PAP smear, and test for gonorrhea d. Routine pelvic examination, PAP smear, test for gonorrhea and Chlamydia e. Schedule colposcopy 298. Which of the following is not recommended for the treatment of gonorrhea Cervicitis? a. Penicillin G, aqueous procaine 4.8 million units b. Ceftriaxone 125 rng IM x 1 c. Cefixime 400 mg PO x 1 d. Ciprofloxacin 500 mg PO x 1 e. Ofloxacin 400 mg PO x 1 299. Which of the following statements about pelvic inflammatory disease (PID) is not true? a. Suitable outpatient treatment of PID is ceftriaxone 250 mg IM and doxycycline 100 mp PO b.i.d. for 14 day b. The incidence of infertility after PID is approximately 2% c. Gonorrhea and Chlamydia account for more than 50% of cases d. Precise diagnosis of PID is difficult and is most accurately made by invasive tests. e. In developing countries, tuberculosis is a common cause of PID 300. Which of the following statements concerning genital ulcer disease is true? a. Herpes simplex virus (HSV) is the most common cause of genital ulcers in developing countries b. The incidence of syphilis in the US is increasing over the past 5 years. c. A reasonable initial work up for a patient who presents with a genital ulcer in the United States is swabbing the lesion for darkfield examination and HSV culture d. Lymphogranuloma vemereum is caused by Hemophilus ducreyi e. Donovan bodies are found in lymph nodes biopsies from patients with chancroid. 301. Reasonable empiric treatment of postoperative gynecologic infection is all but one of the following: a. Cefoxitin b. Clindamycin/gentamicin c. Ampicillin/gentamicin d. Ticarcillin/clavulanic acid (Timentin) e. Ciprofloxacin 302. The most common cause of habitual abortion is: a. Genetic b. Idiopathic c. Anatomical d. Infectious 303. Inadequate luteal phase can be diagnosed by: a. Endometrial dating b. Luteal phase less than 11 days c. Mid luteal phase serum progesterone of less than 5 Mgan/ml d. All of the above 304. If one partner has a balanced translocation, the chance of a successful pregnancy is: a. 5% b. 25% c. 50% d.)% 305. The morbidity and mortality risk is the same regardless of the BMI value. a. True b. False 306. Obesity associated-risk is the same for men as for women. a. True b. False 307. Patients with BMI of 25 or greater should be considered candidates for drug therapy. a. True b. False 308. The composition of the diet is more important than calories for the treatment of obesity. a. True b. False 309. The most successful approach for the treatment of obesity includes diet, exercise, and behavior modifica tion. a. True b. False 310. The rate of Group B streptococcal colonization of the newborn effectively falls how soon after administration of ampicillin? a. < 1 hour b. < 2 hours c. < 4 hours d. < 8 hours e. 8 hours 311. Which of the following tests on amniotic fluid are consistent with intra-amniotic infection? a. Gram stain showing WBCs b. Glucose = 30 mg/dl c. LDH = 190 d. Gram stain showing bacteria e. Alkaline phosphatase = 30 312. Which of the following are true concerning single dose gentamicin? a. Infuse in 10 - 15 minutes b. May be used with reduced renal function c. Requires peak and trough levels d. Administer at 5 - 7 mg/kg every 24 hours e. More toxic than standard therapy 313. Which of the following is the most likely complication of multifetal gestation? a. Abruption b. Growth restriction c. Uterine atony d. Preterm delivery e. Pre-eclampsia 314. Monozygote twins that divide at 3 days post conception are most likely: a. Diamniotic, dichorionic b. Diamniotic, monochorionic c. Monoamniotic, dichorionic d. Monoamniotic, monochorionic e. Conjoined 315. Spontaneous triplets occur how often? a. 1 in 100 b. 1 in 1000 c. 1 in 8000 d. 1 in 15000 e. 1 in 25000 316. A 28 year old PO 100 presents to your office at 11 weeks gestation for a first prenatal visit. Her last preg nancy ended in an intrauterine fetal demise after an abruption. She required 6 units of packed RBCs 20 unites of cryoprecipitate, 6 units of platelets, and 2 units of fresh frozen plasma during the delivery. The patient's blood type is O negative. Her Hematocrit on today's visit is 39%. The patient is considering a termination of pregnan cy, as she does not want to have a similar experience in this pregnancy. She inquires about her risk of a recur rence of abruption in this pregnancy. You inform her that her risk of recurrent abruption in this pregnancy is: a. 2% b. 7% c. 17% d. 25% e. 50% 317. The patient decides to continue the pregnancy. At 24 weeks she presents to Labor and Delivery complain ing of vaginal bleeding for the past 2 hours, decreased fetal movement, and cramping. A transvaginal ultra sound report describes a placenta completely covering the internal cervical os, with a hyperchoic fluid collection between the placental edge and the choriamniotic membranes. Electronic fetal monitoring shows contractions every 5 minutes with a background of uterine irritability, and a fetal heart rate baseline of 155 bpm, without decelerations. Hematocrit today is 26%. Fibrinogen level is 425 mg/dl. Platelet count is 198,000/ml. The cor rect diagnosis is: a. Total placenta previa b. Total placenta previa with subchorionic abruption c. Partial placenta previa d. Partial placenta previa with subchorionic abruption e. Total palcenta previa with preplacental abruption 318. After discussion of treatment options with the patient, you decide to attempt conservative management to prolong the pregnancy. Which of the following measures is least likely to contribute to a good maternal and fetal outcome? a. Transfusion of 2 units of packed RBCs b. Kleihauer-Betke test c. Intravenous magnesium sulfate d. Intramuscular betamethasone e. Subcutaneous terbutaline 319. After 24 hours of treatment, the patient's vaginal bleeding subsides. After a period of observation in the hospital, she is discharged to her home on bedrest and pelvic rest. She experiences no further episodes of bleeding during her pregnancy. At 36 weeks, she undergoes a transvaginal ultrasound that describes a placenta completely covering the cervical os. The most appropriate management at this point is: a. Immediate cesarean delivery b. Double setup examination to determine appropriateness for trial of labor c. Schedule patient for cesarean delivery in 1 week d. Amniocentesis for fetal lung maturity and cesarean delivery if indices are mature. e. Corticosteroids and cesarean delivery 48 hours later. 320. A 22 year old Para O well-known to your staff as a cocaine abuser presents to labor and delivery at 29 weeks with complications of the sudden onset of contractions and abdominal pain. Maternal vital signs show a pulse of 124, blood pressure of 130/82, respiratory rate of 20. Electronic fetal monitoring shows contractions occurring every minute, lasting 30 seconds. The nurse reports increased uterine tone between contractions. The fetal heart rate is 160 with occasional variable decelerations. Ultrasound findings show a fundal placenta with no abnormalities, and appropriately-grown fetus with normal amniotic fluid index. You suspect abruption as the cause of this patient's condition. This would be classified as: a. Grade 1 abruption b. Grade 2 abruption c. Grade 3 abruption d. No abruption at all, there is no vaginal bleeding or ultrasound signs of abruption 321. Initial lab work is drawn and sent. A red-topped tube is drawn and observed for clot formation. A clot forms after 4 minutes of observation. You immediately cancel the order for the fibrinogen level as you know the probable fibrinogen level is closest to: a. 450 mg/dl b. 350 mg/dl c. 250 mg/dl d. 150 mg/dl e. Unable to determine, as the test was not performed correctly 322. The initial labs show a HCT of 22%, a fibrinogen of 120 mg/dl, platelet count of 102,000/ml, and a normal PT/PTT. The most appropriate initial blood component therapy should include: a. 2 units PRBC, 2 units fresh frozen plasma, 10 units platelets b. 4 units PRBC, 6 units cryoprecipitate c. 4 units PRBC, 4 units fresh frozen plasma, 6 units platelets d. 2 units PRBC, 6 units cryoprecipitate e. No blood component therapy is necessary, as the patient is normotensive and not actively bleeding 323. Polycystic ovary syndrome patients may be treated using the following medications EXCEPT: a. OCR b. Spironolactone c. Metformin d. Lipitor 324. Chronic anovulation may be caused by the following conditions: a. Hyperprotactinemia b. Thyroid disease c. Stress d. Hyperinsuline e. All of the above 325. Clinical consequences of polycystic ovary syndrome includes which of the following: a. Infertility b. Diabetes Mellitus c. Endometrial Cancer d. Hirsutism e. All of the above 326. All of the following is true regarding Thalassemias EXCEPT: a. Involves quantitative problem with hemoglobin chain production. b. Increased risk in Oriental and African populations c. In utero compromise never develops d. Prenatal testing is available 327. With Sickle Cell trait all of the following should be considered in the management of pregnant women EXCEPT: a. Increased perinatal mortality b. Increased risk of pyelonephritis c. monthly urine cultures d. Paternal sickle cell testing 328. Sickle cell disease in pregnancy is associated with all of the following EXCEPT: a. Increased stillbirths b. Increased pregnancy induced pre eclampsia c. Increased pyeloneshritis d. Decreased osteomyelitis 329. A person with normochromic microcytic anemia with normal iron indices, an increase in Hgb A2 would be suggestive of which disorder: a. Sickle cell disease b. Beta thalasseima c. Alpha thalasseima d. Sickle Cell trait 330. Sickle cell anemia is associated with which mutation: a. Change value for glutamic acid at postion #6 on beta chain. b. Change lysne for value at position #20 on alpha chain c. Change value for glutamic acid at position #12 on alpha chain d. None of the above 331. Normal physiologic changes in pregnancy include all EXCEPT: a. Elevation of diaphragm b. Mild compensated respiratory alkalosis c. Increased respiratory rate d. Decreased total lung capacity 332. All of the following are worrisome signs in a pregnant asthmatic EXCEPT: a. Prior hospital admissions b. Prior intubations c. Steroid dependency d. Intermittent use of 62 antagonist inhalers 333. Varicella pneumonia in pregnancy is characterized by all of the following EXCEPT: a. Complicates 0.7% all pregnancies with varicella b. Increased mortality rate if pregnant c. Pulmonary symptoms 2-3 days after peak of rash d. Slow progression to pulmonary compromise 334. Plasma volume increases by what percent in pregnancy? a. 10% b. 30% c. 50% d. 70% 335. Red blood cell mass increases by what percent in pregnancy? a. 10% b. 30% c. 50% d. 70% 336. The majority of plasma volume/cardiac output increases occur by what time in pregnancy? a. Mid first trimester b. Mid second trimester c. Mid third trimester d. Labor 337. Which of the following medications is considered safe in pregnancy for the treatment of epilepsy? a. Dilantin b. Tegretol c. Valproic acid d. Phenobarbital e. Each drug has associated increased risk 338. The best treatment of Chorioamnionitis in a pregnant patient with myasthenia gravis is: a. Ampicillin b. Ampicillin and Gentamycin c. Gentamycin and Cleocin d. Unasyn 339. Which antiepileptic agent does not stimulate the cytochrome P450 enzyme system? a. Dilantin b. Valproic acid c. Phenobarbital d. Primidone 340. Breast-feeding is contraindicated in patients on antiepileptic drugs. a. True b. False 341. Breast-feeding is contraindicated in patients with Myasthenia Gravis. a. True b. False 342. Cranial CT scans are contraindicated in pregnancy. a. True b. False 343. An otherwise healthy 35 year old woman is undergoing an intravenous pyelogram. The risk of mortality is: a. 1 in 1000 b. 1 in 10,000 c. 1 in 100,000 d. 1 in 1 million e. None in patients with no known drug allergies 344. Which of the following antihypertensive medications should be discontinued before surgery? a. Clonidine b. Beta Blockers c. Diuretics d. Calcium channel blockers e. MAO inhibitors 345. In a patient with chronic obstructive pulmonary disease which of the following tests should be obtained pre- operatively? a. Venous Po2 b. Residual capacity c. Forced expiratory volume at one second d. Cardiac echocardiogram e. Functional reserved capacity 346. The most likely sign of pulmonary emboli is: a. Crackles b. Tachacardia c. Tachypnea d. Chest pain e. Agitation 347. Wound dehiscence is associated with : a. An incision through a previous scar b. A Chemey incision c. Closure with synthetic absorbable suture d. Internal bleeding e. Air-fluid levels on abdominal x-ray prior to dehiscence 348. Pressure from a retractor on the psoas muscle can damage the nerve coursing on the anterior surface of this muscle with resultant anesthesia on the medial thigh and lateral labia majora. What is the nerve affected? a. Ilioninguinal b. Obturator c. Sympathetic Chain d. Genitofemoral e. Femoral 349. Lymphatic drainage from the lower vagina will track initially to which nodes? a. Superficial inguinal b. Lumbar (paraortic) c. Internal iliac d. External iliac e. None of the above 350. In performing a laparotomy for abdominal exploration, why would you not want to make a paramedian verti cal incision immediately lateral to the rectus abdominus muscle? a. Could damage inferior epigastric artery entering the rectus sheath b. Could damage the ilioinguinal nerve c. Could damage segmental nerve supply to the rectus abdominus muscle d. All of the above e. A and C only 351. Of the following structures, which is most anterior as it crosses the posterior aspect of the pelvic brim? a. Common iliac artery b. Common iliac vein c. Ureter d. Sympathetic chain ganglia e. Round ligament of the ovary Questions 352 356 Matching Match the following sonographic findings with the ovarian tumor with which they are commonly associated: 352. Smooth wall, no internal echoes a. Endometrioma 353. Fibrin strands in a mostly cystic structure b. Hemorrhagic cyst 354. Papillary excresences c. Malignant neoplasm 355. Ground glass appearance d. Cystic teratoma 356. Calcifications e. Physiologic cyst 357. Which of the following is not a type of epithelial ovarian tumor? a. Brenner cell b. Mucinous c. Serous d. Fibroma e. Clear cell f. Endometriod Questions 358 362 Matching Match the following ovarian lesions with the most appropriate therapy. 358. Brenner Cell Tumor a. Expectant with U/S follow-up 359. Premenopausal 6 cm cystic mass b. Ovarian cystectomy 360. Postmenopausal 4 cm cystic mass c. Oophorectomy 362. Cystic teratoma in adolescent PAGE  PAGE 44    3 4 6 7 g h j k     r s u v ŷśōŷŷqqcchMp*@B*CJaJphhMp*@B*CJaJphhMp*@B*CJaJphhMp*@B*CJaJphhMp*@B*CJaJphhMp*@B*CJaJphhMp*@B*CJaJphhMp*hMp*@B*CJaJphhMp*@B*CJaJphhMp*B*CJaJphhMp*@B*CJaJph& 4 h  s ^ D ii d-DM ^`gdMp* d-DM ^`gdMp* d -DM gdMp* d-DM ^gdMp* d-DM ^gdMp* d-DM gdMp* ehh < ] ^ ` a 0 C D F G { | ~  PQSTԪrdhMp*@B*CJaJphhMp*@B*CJaJphhMp*@B*CJaJphhMp*@B*CJaJphhMp*@B*CJaJphhMp*@B*CJaJphhMp*@B*CJaJphhMp*@B*CJaJphhMp*@B*CJaJphhMp*B*CJaJphhMp*@B*CJaJphhMp*'D | QT# (d-DM ^`(gdMp* d-DM ^gdMp* d-DM ^gdMp* d-DM ^gdMp* d-DM gdMp* STVW"#%&  0134^_abhMp*@B*CJaJphhMp*@B*CJaJphhMp*@B*CJaJphhMp*@B*CJaJphhMp*@B*CJaJphhMp*@B*CJaJphhMp*B*CJaJphhMp*@B*CJaJphhMp*6 1_,=ns d-DM gdMp* d-DM ^gdMp* d-DM ^gdMp* d-DM gdMp* d-DM ^gdMp* d-DM gdMp* +,./<=?@mn·עzvhzvWH:vhMp*@B*CJaJphhMp*6B*CJ]aJph!hMp*6@B*CJ]aJphhMp*@B*CJaJphhMp*hMp*@B*CJaJphhMp*B*CJaJphhMp*@B*CJaJph)hhMp*@B*CJaJmH phsH hhMp*mH sH )hhMp*@B*CJaJmH phsH %hhMp*B*CJaJmH phsH )hhMp*@B*CJaJmH phsH npq35ӾvhZLhhMp*@B*CJaJphhMp*@B*CJaJphhMp*@B*CJaJph)hhMp*@B*CJaJmH phsH hhMp*mH sH )hhMp*@B*CJaJmH phsH %hhMp*B*CJaJmH phsH )hhMp*@B*CJaJmH phsH hMp*hMp*@B*CJaJphhMp*B*CJaJphhMp*@B*CJaJph45m d-DM ^gdMp* d-DM ^gdMp* d -DM ^gdMp* d-DM gdMp* d -DM ^gdMp* d-DM ^gdMp* 578)*,-3467ӷӍӍqӷӷhMp*@B*CJaJphhMp*@B*CJaJphhMp*@B*CJaJphhMp*@B*CJaJphhMp*@B*CJaJphhMp*@B*CJaJphhMp*@B*CJaJphhMp*hMp*@B*CJaJphhMp*B*CJaJphhMp*@B*CJaJph&*4=IUkkkk d-DM ^gdMp* d-DM ^gdMp* Fd-DM ^gdMp* d-DM ^gdMp* d-DM ^gdMp* d-DM gdMp* 7<=?@HIKLTUXYөӍqcUcUhMp*@B*CJaJphhMp*@B*CJaJphhMp*@B*CJaJphhMp*@B*CJaJphhMp*@B*CJaJphhMp*@B*CJaJphhMp*@B*CJaJphhMp*@B*CJaJphhMp*@B*CJaJphhMp*B*CJaJphhMp*@B*CJaJphhMp*hMp*@B*CJaJph!679:ABDEMNPQYZ\]efjT>>+hMp*@B*CJOJQJRHO^JaJph+hMp*@B*CJOJQJRHO^JaJph+hMp*@B*CJOJQJRHO^JaJph+hMp*@B*CJOJQJRHO^JaJph+hMp*@B*CJOJQJRHO^JaJph#hMp*B*CJOJQJ^JaJph+hMp*@B*CJOJQJRHO^JaJphhMp*@B*CJaJphhMp*B*CJaJphhMp*@B*CJaJphhMp*7BNZfn .Pt d-DM ^gdMp* Fd-DM ^gdMp* d-DM ^gdMp* d-DM ^gdMp* Fd-DM ^gdMp* fhimnqr  -.01OPRyk]OyyAhMp*@B*CJaJphhMp*@B*CJaJphhMp*@B*CJaJphhMp*@B*CJaJphhMp*@B*CJaJphhMp*@B*CJaJphhMp*@B*CJaJphhMp*B*CJaJphhMp*@B*CJaJphhMp*+hMp*@B*CJOJQJRHH^JaJph#hMp*B*CJOJQJ^JaJph+hMp*@B*CJOJQJRHH^JaJphRSstwx!"$%89;<DEHIŷqq`QhMp*6B*CJ]aJph!hMp*6@B*CJ]aJphhMp*@B*CJaJphhMp*@B*CJaJphhMp*@B*CJaJphhMp*@B*CJaJphhMp*@B*CJaJphhMp*@B*CJaJphhMp*@B*CJaJphhMp*@B*CJaJphhMp*hMp*@B*CJaJphhMp*B*CJaJpht"9EKrgg d-DM ^gdMp* d-DM ^gdMp* Fd-DM ^gdMp* d-DM ^gdMp* d-DM ^gdMp* Fd-DM ^gdMp* JKMNqrtu<=?@rdhMp*@B*CJaJphhMp*@B*CJaJphhMp*@B*CJaJphhMp*@B*CJaJphhMp*@B*CJaJphhMp*@B*CJaJphhMp*@B*CJaJphhMp*@B*CJaJphhMp*@B*CJaJphhMp*B*CJaJphhMp*@B*CJaJphhMp*#r=M^o d-DM ^gdMp* Fd-DM gdMp* d-DM ^gdMp* d-DM ^gdMp* Fd-DM ^gdMp* d-DM ^gdMp* @LMOP]^`a~DEGH`acdvwz{өӍӍӍӍqӍqhMp*@B*CJaJphhMp*@B*CJaJphhMp*@B*CJaJphhMp*@B*CJaJphhMp*@B*CJaJphhMp*@B*CJaJphhMp*@B*CJaJphhMp*B*CJaJphhMp*@B*CJaJphhMp*hMp*@B*CJaJph&Eawk d-DM ^gdMp* d-DM ^gdMp* d -DM ^gdMp* Fd-DM gdMp* d-DM ^gdMp* d-DM ^gdMp* BCFGөӛӛqөqөhMp*@B*CJaJphhMp*@B*CJaJphhMp*@B*CJaJphhMp*@B*CJaJphhMp*@B*CJaJphhMp*@B*CJaJphhMp*@B*CJaJphhMp*hMp*@B*CJaJphhMp*B*CJaJphhMp*@B*CJaJph,CP s d-DM ^gdMp* Fd-DM gdMp* d-DM ^gdMp* Fd-DM gdMp* d-DM ^gdMp* Fd-DM gdMp*   O P R S E!z!{!}!!!!!!!!!!!!!"" " "ӷ婛ӍӍӍ婛ӷhMp*@B*CJaJphhMp*@B*CJaJphhMp*@B*CJaJphhMp*@B*CJaJphhMp*@B*CJaJphhMp*hMp*@B*CJaJphhMp*B*CJaJphhMp*@B*CJaJph7P {!!!!""3"U"""m d-DM ^gdMp* Fd-DM gdMp* d-DM ^gdMp* Fd-DM gdMp* -d-DM ^`-gdMp* d-DM ^gdMp* """!"""2"3"5"6"T"U"X"Y"""""""""""""####*#+#-#.#O#P#S#T###########hMp*@B*CJaJphhMp*@B*CJaJphhMp*@B*CJaJphhMp*@B*CJaJphhMp*@B*CJaJphhMp*@B*CJaJphhMp*B*CJaJphhMp*@B*CJaJphhMp*hMp*@B*CJaJph.""#+#P###### $ $n$|$q d-DM ^gdMp* ^d-DM gdMp* d-DM ^gdMp* d-DM ^gdMp* Fd-DM gdMp* d-DM ^gdMp* ########### $ $$$m$n$p$q${$|$~$$$$$$$$$$$$$$$$$$өӍөөөqөchMp*@B*CJaJphhMp*@B*CJaJphhMp*@B*CJaJphhMp*@B*CJaJphhMp*@B*CJaJphhMp*@B*CJaJphhMp*@B*CJaJphhMp*@B*CJaJphhMp*hMp*@B*CJaJphhMp*B*CJaJphhMp*@B*CJaJph&|$$$$$$U%x%%%%3&G&o d-DM ^gdMp* ^d-DM gdMp* d-DM ^gdMp* d-DM ^gdMp* ^d-DM gdMp* d-DM ^gdMp* $$$%%T%U%W%X%w%x%z%{%%%%%%%%%%%%%2&3&5&6&F&G&I&J&[&\&^&_&q&r&t&u&&&өөөөqhMp*@B*CJaJphhMp*@B*CJaJphhMp*@B*CJaJphhMp*@B*CJaJphhMp*@B*CJaJphhMp*@B*CJaJphhMp*@B*CJaJphhMp*B*CJaJphhMp*@B*CJaJphhMp*hMp*@B*CJaJph*G&\&r&&&&&&'4'a'''mmm d-DM ^gdMp* d-DM ^gdMp* d-DM ^gdMp* ^d-DM gdMp* d-DM ^gdMp* d-DM ^gdMp* &&&&&&&&&&&&&&&&&&&'' ' '3'4'6'7'`'a'c'd'''''''''''''''ӛӍqӛӍhMp*@B*CJaJphhMp*@B*CJaJphhMp*@B*CJaJphhMp*@B*CJaJphhMp*@B*CJaJphhMp*@B*CJaJphhMp*@B*CJaJphhMp*hMp*@B*CJaJphhMp*B*CJaJphhMp*@B*CJaJph,'''R(f({(((())/)8)B)qq d-DM ^gdMp* d-DM ^gdMp* ^d-DM gdMp* d-DM ^gdMp* ^d-DM gdMp* d-DM ^gdMp* '''Q(R(T(U(e(f(h(i(z({(}(~((((((((((((())))өөӍjW%hhMp*B*CJaJmH phsH )hhMp*@B*CJaJmH phsH hMp*@B*CJaJphhMp*@B*CJaJphhMp*@B*CJaJphhMp*@B*CJaJphhMp*@B*CJaJphhMp*@B*CJaJphhMp*hMp*@B*CJaJphhMp*B*CJaJphhMp*@B*CJaJph))) )!).)/)1)2)7)8):);)A)B)D)E)M)N)Q)R)ʷߍseaSsEa7shMp*@B*CJaJphhMp*@B*CJaJphhMp*@B*CJaJphhMp*hMp*@B*CJaJphhMp*B*CJaJphhMp*@B*CJaJph)hhMp*@B*CJaJmH phsH )hhMp*@B*CJaJmH phsH %hhMp*B*CJaJmH phsH )hhMp*@B*CJaJmH phsH hhMp*mH sH )hhMp*@B*CJaJmH phsH B)N)))))))+*Q***+P+Q+ d-DM ^gdMp* d-DM ^gdMp* d-DM ^gdMp* ^d-DM gdMp* d-DM ^gdMp*R)))))))))))))))))))))))))**+*ŰŰ{Ű{ŰmaSEahMp*@B*CJaJphhMp*@B*CJaJphhMp*B*CJaJphhMp*@B*CJaJph)hhMp*@B*CJaJmHphsH)hhMp*@B*CJaJmHphsHhhMp*mHsH)hhMp*@B*CJaJmHphsH%hhMp*B*CJaJmHphsH)hhMp*@B*CJaJmHphsHhMp*hMp*@B*CJaJph+*-*.*P*Q*S*T*********++ +!+O+Q+T+U+++++++++++++++++ӷөөqchMp*@B*CJaJphhMp*@B*CJaJphhMp*@B*CJaJphhMp*@B*CJaJphhMp*@B*CJaJphhMp*@B*CJaJphhMp*@B*CJaJphhMp*@B*CJaJphhMp*hMp*@B*CJaJphhMp*B*CJaJphhMp*@B*CJaJph&Q+++++ ,,g,~,,oU d-DM ^gdMp* d -DM ^gdMp* Zd-DM gdMp* d-DM ^gdMp* d-DM ^gdMp* d -DM ^gdMp* Zd-DM gdMp* +, , , ,,,,,f,g,i,j,},~,,,,,,,,,,,,,,,,,,,2-3-5-6-O-P-R-S-^-_-a-b-qqhMp*@B*CJaJphhMp*@B*CJaJphhMp*@B*CJaJphhMp*@B*CJaJphhMp*@B*CJaJphhMp*@B*CJaJphhMp*@B*CJaJphhMp*B*CJaJphhMp*@B*CJaJphhMp*hMp*@B*CJaJph,,,,,3-P-_-o----.Q... d-DM ^gdMp* Zd-DM gdMp* d -DM ^gdMp* Zd-DM gdMp* d-DM ^gdMp*b-n-o-q-r-------------....P.Q.S.T.............:/;/=/>/O/P/R/S/j/k/m/n/|/}/ӍӍӍӍӍhMp*@B*CJaJphhMp*@B*CJaJphhMp*@B*CJaJphhMp*@B*CJaJphhMp*@B*CJaJphhMp*@B*CJaJphhMp*B*CJaJphhMp*@B*CJaJphhMp*hMp*@B*CJaJph2..;/P/k/}////0.0?0Z0o d-DM ^gdMp* d-DM ^gdMp* Zd-DM gdMp* d-DM ^gdMp* Zd-DM gdMp* d-DM ^gdMp* }///////////////000 0-0.00010>0?0A0B0Y0Z0\0]0l0m0p0q000000111191:1<1=1r1s1u1ӷӷӛӷ׍ӷӛhMp*@B*CJaJphhMp*@B*CJaJphhMp*@B*CJaJphhMp*@B*CJaJphhMp*@B*CJaJphhMp*@B*CJaJphhMp*hMp*@B*CJaJphhMp*B*CJaJphhMp*@B*CJaJph2Z0m001:1s111232R2qq d-DM ^gdMp* Zd-DM gdMp* d-DM ^gdMp* d-DM ^gdMp* Zd-DM gdMp* d-DM ^gdMp* u1v111111111222222325262Q2R2T2U2222222222233n[$hMp*6@B*CJH*]aJph!hMp*6@B*CJ]aJphhMp*@B*CJaJphhMp*@B*CJaJphhMp*@B*CJaJphhMp*@B*CJaJphhMp*@B*CJaJphhMp*@B*CJaJphhMp*@B*CJaJphhMp*hMp*@B*CJaJphhMp*B*CJaJph!R22233d3333 4h4m d-DM ^gdMp* d-DM ^gdMp* d -DM ^gdMp* Pd-DM gdMp* d-DM ^gdMp* d-DM ^gdMp* 3 3 3c3d3f33333333334 4"4#4g4h4k4l44444444 555#5$5&5M5N5P5x5y5|5}55555߷߭߷ߙ#hMp*B*CJOJQJ^JaJph'hMp*@B*CJOJQJ^JaJphhMp*@B*phhMp*@B*phhMp*@B*phhMp*@B*phhMp*@B*phhMp*hMp*@B*phhMp*B*phhMp*@B*ph.h4445$5N5y5555oo d-DM ^gdMp* Pd-DM gdMp* d -DM ^gdMp* d-DM ^gdMp* d-DM ^gdMp* Pd-DM gdMp* 55555555555555555555q]PE;3hMp*B*phhMp*@B*phhMp*6B*]phhMp*6@B*]ph'hMp*@B*CJOJQJ^JaJph'hMp*@B*CJOJQJ^JaJph'hMp*@B*CJOJQJ^JaJph'hMp*@B*CJOJQJ^JaJph'hMp*@B*CJOJQJ^JaJph#hMp*B*CJOJQJ^JaJph'hMp*@B*CJOJQJ^JaJphhMp*'hMp*@B*CJOJQJ^JaJph5555.666B6M6T6r67o Pd-DM gdMp* d-DM ^gdMp* d-DM ^gdMp* Pd-DM gdMp* d-DM ^gdMp* d-DM ^gdMp* 5-6.6061656668696A6B6D6E6L6M6O6P6S6T6V6W6q6r6u6v6777"8#8%8&8Q8R8T8v8w8y8z8888888Y9Z9\99999999#:$:&:':Y:Z:\:]:::hMp*@B*phhMp*@B*phhMp*@B*phhMp*@B*phhMp*@B*phhMp*@B*phhMp*B*phhMp*@B*phhMp*hMp*@B*ph@7#8R8w888Z999$:Z::;k Pd-DM gdMp* #d-DM ^`#gdMp* d-DM ^gdMp* Pd-DM gdMp* d-DM ^gdMp* d-DM ^gdMp* :::;;;;;;;;; ;";#;2;3;5;6;?;@;B;C;O;P;U;V;;;;;;;;;;;;;;;;;߷ߟ{qqqhMp*@B*phhMp*@B*phhMp*@B*CJaJphhMp*B*CJaJphhMp*@B*CJaJphhMp*@B*phhMp*@B*phhMp*@B*phhMp*@B*phhMp*@B*phhMp*hMp*@B*phhMp*B*phhMp*@B*ph*;; ;3;@;P;Q;R;;;;;; < << d-DM ^gdMp* Pd-DM gdMp* d -DM ^gdMp* d-DM ^gdMp* d-DM ^gdMp*;; <<a<b<d<e<<<<<<<<<<<<<%=&=(=)=\=]=m=n========= >~m~m\!hThMp*@B*CJaJph!hThMp*@B*CJaJphhThMp*B*CJaJph!hThMp*@B*CJaJphhMp*5>*@B*\phhMp*@B*phhMp*@B*phhMp*@B*phhMp*@B*phhMp*@B*phhMp*@B*phhMp*hMp*B*phhMp*@B*ph$<b<<<<&=]=n====ww\ & Fz d -DM gdMp*0-DM ^0gdMp*0-DM ^0gdMp* d-DM ^gdMp* d -DM ^gdMp* d-DM ^gdMp*d-DM ^gdMp* ==>(>F>W>g>|>>>>>?#?3?F?xd-DM ^xgdMp* & Fz d-DM gdMp* & Fz d-DM gdMp* & Fz d-DM gdMp* > >>>%>'>(>C>E>F>S>T>V>W>f>g>x>y>|>>>>>>>>>>>>>>??𬽛߽y߽hW!hThMp*@B*CJaJph!hThMp*@B*CJaJph!hThMp*@B*CJaJph!hThMp*@B*CJaJph!hThMp*@B*CJaJph!hThMp*@B*CJaJph!hThMp*@B*CJaJph!hThMp*@B*CJaJph!hThMp*@B*CJaJphhThMp*B*CJaJph!???? ?$?2?3?E?F?u??????@"@7@8@j@@@@@@@ AAA A̡̽̽̽vndS!hhMp*@B*mHphsHhMp*@B*phhMp*B*phhMp*@B*phhMp*hMp*B*CJaJph!hThMp*@B*CJaJphhThMp*CJaJ!hThMp*@B*CJaJphhThMp*B*CJaJph!hThMp*@B*CJaJph!hThMp*@B*CJaJph!hThMp*@B*CJaJphF?u??????"@8@@@@A4AtA{{ d-DM ^gdMp* Zd-DM ^gdMp*5d-DM ^5gdMp*Md-DM ]M^gdMp*d-DM gdMp*d-DM UD]gdMp* A!A3A4A6A7AsAtAvAwAAAAAAAAAAAAHBkBoB}B~BBBBBBBBBBBBԨ~rhhMp*@B*phhMp*>*@B*phhMp*@B*phhMp*@B*phhMp*hMp*@B*phhMp*B*phhMp*@B*ph!hhMp*@B*mHphsH!hhMp*@B*mHphsHhhMp*mHsH!hhMp*@B*mHphsHhhMp*B*mHphsH$tAAAA~BBBB C0CDg Zd-DM ^gdMp* d-DM ^gdMp* d-DM ^gdMp* Zd-DM ^gdMp* d-DM ^gdMp* d-DM ^gdMp* BBB C CCC/C0C3C4CCqDDDDDDDDDDDDEEE(E)E+EBECEUEVEEEEEEEEE߭ߞߍ|k|!hThMp*@B*CJaJph!hThMp*@B*CJaJph!hThMp*@B*CJaJphhMp*5>*@B*\phhMp*@B*phhMp*@B*phhMp*@B*phhMp*@B*phhMp*@B*phhMp*hMp*@B*phhMp*B*phhMp*@B*ph)DDDE)ECEDEEEVEEEEEo & F{ d-DM UD3]gdMp* & F{ d-DM gdMp*-DM ^gdMp* -DM ^ gdMp* d-DM ^gdMp* d-DM ^gdMp* EEEEEF+FCFXFYFGqq] Pd-DM gdMp* %d-DM gdMp* %d-DM gdMp* %d-DM UD]gdMp* %d-DM gdMp* & F{ d-DM gdMp* & F{ d-DM gdMp* EEEEEEEEEEEFFFF*F+F-F.FBFCFEFFFݻݬn]L;!hThMp*@B*CJaJph!hThMp*@B*CJaJph!hThMp*@B*CJaJph!hThMp*@B*CJaJph!hThMp*@B*CJaJphhThMp*CJaJ!hThMp*@B*CJaJphhThMp*B*CJaJph!hThMp*@B*CJaJph!hThMp*@B*CJaJph!hThMp*@B*CJaJph!hThMp*@B*CJaJphFFWFYF\F]FFGGGGGGGGGH H H HɻɻxePE0e)hhMp*@B*CJaJmHphsHhhMp*mHsH)hhMp*@B*CJaJmHphsH%hhMp*B*CJaJmHphsH)hhMp*@B*CJaJmHphsHhMp*@B*CJaJphhMp*hMp*@B*CJaJphhMp*@B*CJaJphhMp*@B*CJaJphhMp*B*CJaJphhMp*@B*CJaJphhThMp*CJaJ!hThMp*@B*CJaJphGG HIHbHxH8IVImIIIIk d-DM ^gdMp* d-DM ^gdMp* Pd-DM gdMp* d -DM ^gdMp* d-DM ^gdMp* d-DM ^gdMp* HHHIHKHLHaHbHdHeHwHxH{H|HH7I8I:I;IUIVIXIYIlImIʷʷߔzlhZzhL>hhMp*@B*CJaJphhMp*@B*CJaJphhMp*@B*CJaJphhMp*hMp*@B*CJaJphhMp*@B*CJaJphhMp*B*CJaJphhMp*@B*CJaJph)hhMp*@B*CJaJmHphsH%hhMp*B*CJaJmHphsH)hhMp*@B*CJaJmHphsHhhMp*mHsH)hhMp*@B*CJaJmHphsHmIoIpIIIIIIIIIIIII J.J/J1J2J>J?JAJBJUJVJXJYJJJJJJJJJJJө囷ӍqcөchMp*@B*CJaJphhMp*@B*CJaJphhMp*@B*CJaJphhMp*@B*CJaJphhMp*@B*CJaJphhMp*@B*CJaJphhMp*@B*CJaJphhMp*@B*CJaJphhMp*hMp*@B*CJaJphhMp*B*CJaJphhMp*@B*CJaJph%I/J?JVJJJJKKK,K9K2Lo Pd-DM gdMp* d-DM ^gdMp* Pd-DM gdMp* d-DM ^gdMp* d-DM ^gdMp* P d-DM ] gdMp* JJJKKKKKKKKKK!K"K+K,K.K/K8K9KM?MAMBMuMvMyMzMMMMMMMMMMMMMMMMMNNNN.NөӍqcqUqhMp*@B*CJaJphhMp*@B*CJaJphhMp*@B*CJaJphhMp*@B*CJaJphhMp*@B*CJaJphhMp*@B*CJaJphhMp*@B*CJaJphhMp*@B*CJaJphhMp*@B*CJaJphhMp*B*CJaJphhMp*@B*CJaJphhMp*hMp*@B*CJaJph!MMMMN/NNNNNNNNnn -DM ^gdMp* d-DM ^gdMp* d-DM ^gdMp* Pd-DM gdMp* d-DM ^gdMp* d-DM ^gdMp* .N/N2N3NNNNNNNNNNNNNNNubK=/hMp*@B*CJaJphhMp*@B*CJaJph-hhMp*@B*CJRHVaJmHphsH%hhMp*B*CJaJmHphsH-hhMp*@B*CJRHVaJmHphsHhhMp*mHsH)hhMp*@B*CJaJmHphsH%hhMp*B*CJaJmHphsH)hhMp*@B*CJaJmHphsHhMp*@B*CJaJphhMp*B*CJaJphhMp*@B*CJaJphhMp*NNNNNNNNNOOlOOOOOOOOOOOOO P P P PKPLPNPOP`PaPdPePP:QQ巩ӛqc婷hMp*@B*CJaJphhMp*@B*CJaJphhMp*@B*CJaJphhMp*@B*CJaJphhMp*@B*CJaJphhMp*@B*CJaJphhMp*@B*CJaJphhMp*@B*CJaJphhMp*hMp*@B*CJaJphhMp*B*CJaJphhMp*@B*CJaJph&NOOO PLPaPQQ R:RRRqq d-DM ^gdMp* d-DM ^gdMp* Zd-DM gdMp* d-DM ^gdMp* d-DM ^gdMp* Zd-DM gdMp* QQQQQQQQ R R R R9R:Rs)hhMp*@B*CJaJmHphsHhhMp*mHsH)hhMp*@B*CJaJmHphsH%hhMp*B*CJaJmHphsH)hhMp*@B*CJaJmHphsHhMp*@B*CJaJphhMp*@B*CJaJphhMp*@B*CJaJphhMp*@B*CJaJphhMp*B*CJaJphhMp*@B*CJaJphhMp*hMp*@B*CJaJphVWWW WW W"W#W)hhMp*@B*CJaJmH phsH %hhMp*B*CJaJmH phsH )hhMp*@B*CJaJmH phsH hMp*@B*CJaJphhMp*@B*CJaJphhMp*@B*CJaJphhMp*@B*CJaJphhMp*hMp*@B*CJaJphhMp*B*CJaJphhMp*@B*CJaJphhhMp*mHsH)hhMp*@B*CJaJmHphsHW W=WUWWWWWXXXX {+d-DM ^+gdMp* d-DM ^gdMp* Zd-DM gdMp* d-DM ^gdMp* d-DM ^gdMp* WWWWWWWWWWWWXXXXXXXXXXXXX.Y{Y|Y~YYYYY͸~pbp~ThMp*@B*CJaJphhMp*@B*CJaJphhMp*@B*CJaJphhMp*@B*CJaJphhMp*hMp*@B*CJaJphhMp*B*CJaJphhMp*@B*CJaJph)hhMp*@B*CJaJmH phsH %hhMp*B*CJaJmH phsH )hhMp*@B*CJaJmH phsH hhMp*mH sH  X|YY ZZ[?[\[{[[[s[[[[ d-DM ^gdMp* {+d-DM ^+gdMp* -d-DM ]^`-gdMp* d-DM ^gdMp* d-DM ^gdMp* -d-DM ]^`-gdMp* YY Z Z ZZeZ~ZZZZZZ[[[>[?[A[B[[[\[^[_[z[{[}[~[[[ŷᔁlaLla)hhMp*@B*CJaJmH phsH hhMp*mH sH )hhMp*@B*CJaJmH phsH %hhMp*B*CJaJmH phsH )hhMp*@B*CJaJmH phsH hMp*@B*CJaJphhMp*@B*CJaJphhMp*@B*CJaJphhMp*@B*CJaJphhMp*hMp*@B*CJaJphhMp*B*CJaJph[[[[[[[[[[[\\ \ \\\\\&\'\)\*\1\2\4\5\9\:\<\=\E\F\I\өqccUhMp*@B*CJaJphhMp*@B*CJaJphhMp*@B*CJaJphhMp*@B*CJaJphhMp*@B*CJaJphhMp*@B*CJaJphhMp*@B*CJaJphhMp*@B*CJaJphhMp*@B*CJaJphhMp*hMp*@B*CJaJphhMp*B*CJaJphhMp*@B*CJaJph![[\\'\2\:\F\\\\\\\ ] {+d-DM ^+gdMp* d-DM ^gdMp* d-DM ^gdMp* {+d-DM ^+gdMp* d-DM ^gdMp*I\J\\\\\\\\\\\\\\\\\\\\\\\\\ ] ]]qq\)hhMp*@B*CJaJmH phsH hMp*@B*CJaJphhMp*@B*CJaJphhMp*@B*CJaJphhMp*@B*CJaJphhMp*@B*CJaJphhMp*@B*CJaJphhMp*@B*CJaJphhMp*@B*CJaJphhMp*hMp*@B*CJaJphhMp*B*CJaJph]]]]]]]]!]"](])]+],]7]8]:];]@]A]B]C]F]G]̷̍xj^PL^L>^hMp*@B*CJaJphhMp*hMp*@B*CJaJphhMp*B*CJaJphhMp*@B*CJaJph)hhMp*@B*CJaJmH phsH )hhMp*@B*CJaJmH phsH )hhMp*@B*CJaJmH phsH )hhMp*@B*CJaJmH phsH hhMp*mH sH )hhMp*@B*CJaJmH phsH %hhMp*B*CJaJmH phsH  ]]])]8]A]C]]]]m d-DM ^gdMp* d-DM ^gdMp* {+d-DM ^+gdMp*&d"-DM ^&gdMp* d-DM ^gdMp* d-DM ^gdMp* G]]]]]]]]]]]]]]]]]]]]]ŰŰ{fQŰC7hMp*B*CJaJphhMp*@B*CJaJph)hhMp*@B*CJaJmH phsH )hhMp*@B*CJaJmH phsH )hhMp*@B*CJaJmH phsH )hhMp*@B*CJaJmH phsH hhMp*mH sH )hhMp*@B*CJaJmH phsH %hhMp*B*CJaJmH phsH )hhMp*@B*CJaJmH phsH hMp*hMp*@B*CJaJph]]]]5^T^x^^^ _ _D_kd-DM ^gdMp* d-DM ^gdMp* d-DM ^gdMp* d-DM ^gdMp* {+d--DM ^+gdMp* d-DM ^gdMp* ]]]]]0^4^5^7^8^S^T^V^W^w^x^z^{^^^^^^^^^ _ _C_D_F_G_U_V_X_Y_j_k_m_n______ŷӛӛqqqhMp*@B*CJaJphhMp*@B*CJaJphhMp*@B*CJaJphhMp*@B*CJaJphhMp*@B*CJaJphhMp*@B*CJaJphhMp*@B*CJaJphhMp*B*CJaJphhMp*@B*CJaJphhMp*hMp*@B*CJaJph,D_V_k________ `u  d -DM ^ gdMp* d-DM ^gdMp* -DM gdMp*7& +Ds-D.:/&M gdMp* d-DM ^gdMp* d-DM ^gdMp* ____________` ` ` `````/`0`2`3`V`W`Y`Z`l`m`p`q`````tfhMp*@B*CJaJphhMp*@B*CJaJph"""hMp*@B*CJaJph"""hMp*B*CJaJph"""hMp*@B*CJaJph"""hMp*@B*CJaJphhMp*@B*CJaJphhMp*@B*CJaJphhMp*@B*CJaJphhMp*B*CJaJphhMp*@B*CJaJphhMp*# ``0`W`m`` a2aWaaar  -DM ^ gdMp*  -DM ^ gdMp*  d-DM ^ gdMp* -DM ^gdMp*  d-DM ^ gdMp*  d-DM ^ gdMp* ``a a a aa1a2a4a5aVaWaYaZaaaaaaaaaaaaabb b bb b"b#b2b3b5b6bUb߷ߝߏ߁߁sehMp*@B*CJaJph"""hMp*@B*CJaJph"""hMp*@B*CJaJphhMp*@B*CJaJphhMp*@B*CJaJph"""hMp*B*CJaJph"""hMp*@B*CJaJph"""hMp*B*CJaJphhMp*@B*CJaJphhMp*hMp*@B*CJaJphhMp*@B*CJaJph"""'aab b3bVbmb.ccEddeyaay d-DM ^gdMp* (d-DM ^`(gdMp* d-DM ]^gdMp*  d-DM ^ gdMp*  d-DM ^ gdMp* d-DM ^gdMp* UbVbXbYblbmbpbqbb-c.c0c1cccccDdEdGdHdddddee e ekeleoepeeeeeeeeeeeeeethMp*@B*CJaJphhMp*@B*CJaJph"""hMp*B*CJaJph"""hMp*@B*CJaJph"""hMp*@B*CJaJphhMp*@B*CJaJphhMp*@B*CJaJphhMp*@B*CJaJphhMp*B*CJaJphhMp*@B*CJaJphhMp*-eleeeee f fafofzfffmmmm d-DM ^gdMp* d-DM gdMp* d-DM ^gdMp* d-DM ^gdMp* d-DM ^gdMp* d-DM ^gdMp* eeeef f f ff f$f%f`fafcfdfnfofqfrfyfzf|f}fffffffffffffygg>h?hAhƺttfԞhMp*@B*CJaJph"""hMp*@B*CJaJphhMp*@B*CJaJphhMp*@B*CJaJphhMp*@B*CJaJphhMp*@B*CJaJph"""hMp*B*CJaJphhMp*@B*CJaJphhMp*@B*CJaJphhMp*B*CJaJph"""hMp*@B*CJaJph"""hMp*(ff?h|hhhhhijjq d-DM ^gdMp* d-DM gdMp* d -DM ^gdMp* d-DM ^gdMp* d-DM gdMp* d-DM ^gdMp* AhBh{h|h~hhhhhhhhhhhhhhhiiijiijjjjjjjjjjjj%k&k(k)kǫseǫhMp*@B*CJaJphhMp*@B*CJaJphhMp*@B*CJaJphhMp*@B*CJaJph"""hMp*@B*CJaJph"""hMp*@B*CJaJphhMp*@B*CJaJphhMp*B*CJaJphhMp*@B*CJaJphhMp*hMp*@B*CJaJphhMp*B*CJaJph"""'jj&k9kOkkkk,l]llkkk d-DM ^gdMp* d-DM ^gdMp* d-DM gdMp* d-DM ^gdMp* d-DM ^gdMp* d -DM ^gdMp* )k8k9k;kn?ngnhnjnknnnnnnnn;oooooos?sAsBsNsPsTsUsssssssssssssssssss흫큫s흫e큫흫WhMp*@B*CJaJphhMp*@B*CJaJphhMp*@B*CJaJphhMp*@B*CJaJphhMp*@B*CJaJphhMp*@B*CJaJphhMp*B*CJaJphhMp*@B*CJaJphhMp*@B*CJaJphhMp*B*CJaJph"""hMp*@B*CJaJph"""hMp*hMp*@B*CJaJphsssssss t!t#t$t.t/t1t2t8t9t;t~@~A~]~hMp*@B*phhMp*@B*phhMp*@B*phhMp*@B*phhMp*@B*phhMp*@B*phhMp*@B*phhMp*@B*phhMp*@B*phhMp*hMp*B*phhMp*@B*ph8||||||:}X}k}}}}k d -DM ^gdMp* d-DM ^gdMp* d-DM ^gdMp* d-DM gdMp* d -DM ^gdMp* d-DM ^gdMp* }~>~^~s~~~Kv;' d-DM ^gdMp* d-DM ^gdMp* d-DM ^gdMp* d-DM ^gdMp* d-DM gdMp*]~^~`~a~r~s~u~v~~~~~~~~JKMuvx:;? &')*ABDbceށ߁hMp*@B*phhMp*@B*phhMp*@B*phhMp*@B*phhMp*@B*phhMp*@B*phhMp*@B*phhMp*@B*phhMp*B*phhMp*@B*phhMp*9'Bc߁ "mm d-DM ^gdMp* d-DM ^gdMp* d-DM ^gdMp* d-DM gdMp* d-DM ^gdMp* d-DM ^gdMp* ߁ ߲áߐÆ~tp~fp\~RpH~phMp*@B*phhMp*@B*phhMp*@B*phhMp*@B*phhMp*hMp*@B*phhMp*B*phhMp*@B*ph!hhMp*@B*mHphsH!hhMp*@B*mHphsH!hhMp*@B*mHphsHhhMp*mHsH!hhMp*@B*mHphsHhhMp*B*mHphsH!hhMp*@B*mHphsH !"$%:;=>RSUVqrxy̓΃Ѓ 5689VWYZɸɸwmwccwhMp*@B*phhMp*@B*phhMp*@B*phhMp*@B*phhMp*@B*phhMp*6B*]phhMp*6@B*]phhhMp*mH sH !hhMp*@B*mH phsH hhMp*B*mH phsH !hhMp*@B*mH phsH hMp*hMp*B*phhMp*@B*ph"";Srst΃6W„Ä d-DM ^gdMp* &d-DM ^&gdMp* d-DM ^gdMp* d-DM ^gdMp* d-DM ^gdMp*ÄDŽȄ!-.01ŷߢzo^Oo:)hhMp*@B*CJaJmH phsH hhMp*B*mH phsH !hhMp*@B*mH phsH hhMp*mH sH )hhMp*@B*CJaJmH phsH %hhMp*B*CJaJmH phsH )hhMp*@B*CJaJmH phsH hMp*@B*CJaJphhMp*B*CJaJphhMp*@B*CJaJphhMp*hMp*@B*phhMp*B*phhMp*@B*ph.>Qa˅مo -DM ^gdMp* -DM ^gdMp* d -DM ^gdMp* &d -DM ^&gdMp* d-DM ^gdMp* d-DM ^gdMp* 1=>@APQST`aefʅ˅ͅ΅؅مۅ܅ŷŗŗ{mŷŗc[QhMp*@B*phhMp*B*phhMp*@B*phhMp*@B*CJaJphhMp*@B*CJaJphhMp*@B*CJaJphhMp*@B*CJaJphhMp*@B*CJaJphhMp*hMp*@B*CJaJphhMp*B*CJaJphhMp*@B*CJaJphhhMp*mH sH )hhMp*@B*CJaJmH phsH QSUVYZ[deqr~ᝏseWsIhMp*@B*CJaJphhMp*@B*CJaJphhMp*@B*CJaJphhMp*@B*CJaJphhMp*@B*CJaJphhMp*@B*CJaJphjhMp*UmHnHuhMp*B*CJ aJ phhMp*5B*CJ \aJ phhMp*@B*CJaJphhMp*5>*B*\phhMp*hMp*@B*CJaJphhMp*B*CJaJphRSVZersf -DM gdMp* & F| d-DM gdMp* & F| d-DM gdMp*Q | &`#$+D-D.:/&M gdMp*o!&`#$+D-D.:/&M gdMp*&-DM ^&gdMp*&-DM ^&gdMp* Ć܆D_Ĉ*NO d-DM ^gdMp* d-DM gdMp* d-DM ^gdMp* d-DM gdMp* -DM gdMp*ÆĆƆdžۆ܆ކ߆CDFG^_cdfh:큝seWIWhMp*@B*CJaJph"""hMp*@B*CJaJphhMp*@B*CJaJph"""hMp*@B*CJaJphhMp*@B*CJaJph"""hMp*@B*CJaJphhMp*B*CJaJph"""hMp*@B*CJaJph"""hMp*@B*CJaJphhMp*@B*CJaJphhMp*B*CJaJphhMp*@B*CJaJphhMp*hMp*@B*CJaJphÈĈƈLj)*,-MOSTө훏훏~oaөShMp*@B*CJaJphhMp*@B*CJaJphhMp*6B*CJ]aJph"""!hMp*6@B*CJ]aJph"""hMp*B*CJaJph"""hMp*@B*CJaJph"""hMp*@B*CJaJphhMp*@B*CJaJphhMp*@B*CJaJphhMp*B*CJaJphhMp*@B*CJaJphhMp*hMp*@B*CJaJph OƉ$ʊrZZ d-DM ^gdMp* d-DM ^gdMp* -DM gdMp* d-DM ^gdMp* d-DM ^gdMp* d -DM ^gdMp* d-DM gdMp* ʼnƉȉɉ#$()Ɋʊ̊͊<=?@өqөhMp*@B*CJaJphhMp*@B*CJaJphhMp*@B*CJaJphhMp*@B*CJaJphhMp*@B*CJaJphhMp*@B*CJaJphhMp*@B*CJaJphhMp*B*CJaJphhMp*@B*CJaJphhMp*hMp*@B*CJaJph,ʊ=zYjoW d-DM ^gdMp* d-DM gdMp* d-DM ^gdMp* d-DM ^gdMp* d-DM gdMp* d-DM ^gdMp* d-DM ^gdMp* @yz|}XY[\ijlmөӍqcchMp*@B*CJaJphhMp*@B*CJaJphhMp*@B*CJaJphhMp*@B*CJaJphhMp*@B*CJaJphhMp*@B*CJaJphhMp*@B*CJaJphhMp*@B*CJaJphhMp*B*CJaJphhMp*@B*CJaJphhMp*hMp*@B*CJaJph"j+BZk~Z d-DM ^gdMp* d-DM gdMp* d-DM ^gdMp* d-DM gdMp* d-DM ^gdMp**+-.ABDEYZ\]jkmnөӛөxePExehhMp*mH sH )hhMp*@B*CJaJmH phsH %hhMp*B*CJaJmH phsH )hhMp*@B*CJaJmH phsH hMp*@B*CJaJphhMp*@B*CJaJphhMp*@B*CJaJphhMp*@B*CJaJphhMp*@B*CJaJphhMp*hMp*@B*CJaJphhMp*B*CJaJphhMp*@B*CJaJphn}~YZ\]YZ\]yŷŗŷ{m_mŷŷŷ_ŷhMp*@B*CJaJphhMp*@B*CJaJphhMp*@B*CJaJphhMp*@B*CJaJphhMp*@B*CJaJphhMp*@B*CJaJphhMp*hMp*@B*CJaJphhMp*B*CJaJphhMp*@B*CJaJphhhMp*mH sH )hhMp*@B*CJaJmH phsH %ZzȏޏTct d-DM ^gdMp* -DM gdMp* d-DM ^gdMp* d-DM gdMp* d-DM ^gdMp* d-DM ^gdMp* yz|}ǏȏʏˏݏޏSTVWbcefyz|}ӐՐ|nhMp*@B*CJaJphhMp*@B*CJaJphhMp*@B*CJaJphhMp*6B*CJ]aJph!hMp*6@B*CJ]aJphhMp*@B*CJaJphhMp*@B*CJaJphhMp*@B*CJaJphhMp*B*CJaJphhMp*@B*CJaJphhMp*)czԐՐݑ o d-DM ^gdMp* d-DM ^gdMp* d-DM gdMp* d-DM ^gdMp* d-DM ^gdMp* d-DM ^gdMp* ՐِڐDܑݑߑ ÒĒƒǒɻ囷囷׷囷囷׷q廷׷chMp*@B*CJaJphhMp*@B*CJaJphhMp*@B*CJaJphhMp*@B*CJaJphhMp*@B*CJaJphhMp*@B*CJaJphhMp*hMp*@B*CJaJphhMp*@B*CJaJphhMp*@B*CJaJphhMp*B*CJaJphhMp*@B*CJaJph&Ē DTzd d-DM ]^gdMp* d-DM ^gdMp* d-DM ^gdMp* d-DM ^gdMp* d-DM gdMp*$[d-DM ][^a$gdMp*  $%CDŰ{maSEa7hMp*@B*CJaJphhMp*@B*CJaJphhMp*@B*CJaJphhMp*B*CJaJphhMp*@B*CJaJph)hhMp*@B*CJaJmH phsH )hhMp*@B*CJaJmH phsH hhMp*mH sH )hhMp*@B*CJaJmH phsH %hhMp*B*CJaJmH phsH )hhMp*@B*CJaJmH phsH hMp*hMp*@B*CJaJphDFGSTVWgjyz|} ./ķwi[MwhMp*@B*CJaJphhMp*@B*CJaJphhMp*@B*CJaJphhMp*@B*CJaJph h?FlhMp*h?FlhMp*@B*phh?FlhMp*@B*\phh?FlhMp*@B*\phh?FlhMp*B*\phh?FlhMp*@B*\phhMp*hMp*@B*CJaJphhMp*B*CJaJphhMp*@B*CJaJphz/\̔%?Vqq d-DM ^gdMp* d-DM gdMp* d-DM ^gdMp* d-DM ^gdMp* d-DM gdMp* d-DM ^gdMp* /12[\^_˔̔Дє$%'(>?ABUVXYhiөӛӆs^ShhMp*mH sH )hhMp*@B*CJaJmH phsH %hhMp*B*CJaJmH phsH )hhMp*@B*CJaJmH phsH hMp*@B*CJaJphhMp*@B*CJaJphhMp*@B*CJaJphhMp*@B*CJaJphhMp*hMp*@B*CJaJphhMp*B*CJaJphhMp*@B*CJaJphVi,Jm͗bo d-DM gdMp* d-DM ^gdMp* d-DM ^gdMp* d-DM gdMp* d-DM ^gdMp* d-DM ^gdMp* ikl ߖ+,./IJLMlmop·}ooaoaoSooahMp*@B*CJaJphhMp*@B*CJaJphhMp*@B*CJaJphhMp*@B*CJaJphhMp*hMp*@B*CJaJphhMp*B*CJaJphhMp*@B*CJaJphhhMp*mH sH )hhMp*@B*CJaJmH phsH %hhMp*B*CJaJmH phsH )hhMp*@B*CJaJmH phsH  ̗͗їҗ;abdeØĘƘǘ=?CD}~ŷӷӷӷӷqchMp*@B*CJaJphhMp*@B*CJaJphhMp*@B*CJaJphhMp*@B*CJaJphhMp*@B*CJaJphhMp*@B*CJaJphhMp*@B*CJaJphhMp*@B*CJaJphhMp*B*CJaJphhMp*@B*CJaJphhMp*hMp*@B*CJaJph&bĘ>?~™יi d-DM ^gdMp* d-DM ^gdMp* d -DM ^gdMp* d-DM ^gdMp* d-DM ^gdMp* d-DM ^gdMp* ™ęř֙יٙڙ679:_`bcrfU!hMp*6@B*CJ]aJphhMp*B*CJaJph"""hMp*@B*CJaJph"""hMp*@B*CJaJphhMp*@B*CJaJphhMp*@B*CJaJphhMp*@B*CJaJphhMp*@B*CJaJphhMp*@B*CJaJphhMp*@B*CJaJphhMp*B*CJaJphhMp*@B*CJaJphhMp*7`BTn & F} Vd-DM ^gdMp*-DM ^gdMp*-DM ^gdMp* d-DM ^gdMp* d-DM ^gdMp* d-DM ^gdMp* ABST֛כٛڛ}oaooSahMp*@B*CJaJphhMp*@B*CJaJphhMp*@B*CJaJphhMp*@B*CJaJphhMp*@B*CJaJphhMp*@B*CJaJphhMp*@B*CJaJphhMp*B*CJaJphhMp*@B*CJaJphhMp*5>*@B*\phhMp*hMp*@B*CJaJphhMp*6B*CJ]aJph?,O| d-DM gdMp* s6d-DM ]s^`6gdMp* d-DM ^gdMp* d-DM gdMp* & F} Vd-DM ^gdMp* >?ABÝ"+,./NO׻ɭ׻ןןse׻WehMp*@B*CJaJphhMp*@B*CJaJphhMp*B*CJaJph"""hMp*@B*CJaJph"""hMp*@B*CJaJphhMp*hMp*@B*CJaJphhMp*@B*CJaJphhMp*@B*CJaJphhMp*@B*CJaJphhMp*B*CJaJphhMp*@B*CJaJphhMp*@B*CJaJph!OST.ޟߟ 89;<ijnoݠŷśōsesWsŷhMp*@B*CJaJph"""hMp*@B*CJaJph"""hMp*B*CJaJph"""hMp*@B*CJaJph"""hMp*@B*CJaJphhMp*@B*CJaJphhMp*@B*CJaJphhMp*@B*CJaJphhMp*hMp*@B*CJaJphhMp*@B*CJaJphhMp*B*CJaJphhMp*@B*CJaJph!ߟ9jơҡ"Oq d-DM ^gdMp* d-DM gdMp* d-DM ^gdMp* d-DM gdMp* d-DM ^gdMp* d-DM ^gdMp* šơȡɡϡѡҡԡաg!"$%NOQRޣseӁshMp*@B*CJaJphhMp*@B*CJaJph"""hMp*@B*CJaJphhMp*@B*CJaJphhMp*@B*CJaJphhMp*B*CJaJph"""hMp*@B*CJaJph"""hMp*@B*CJaJph"""hMp*B*CJaJphhMp*@B*CJaJphhMp*hMp*@B*CJaJph'Oߣ()ǤAzڥi  d-DM ^ gdMp* d-DM ^gdMp* d-DM ^gdMp*  d-DM ^ gdMp* d-DM ^gdMp* d-DM ^gdMp* ޣߣ')-.ƤǤɤʤ@ACDyz|}٥ڥܥk)hhMp*@B*CJaJmH phsH hMp*@B*CJaJphhMp*@B*CJaJphhMp*@B*CJaJphhMp*@B*CJaJphhMp*@B*CJaJphhMp*@B*CJaJphhMp*@B*CJaJphhMp*B*CJaJphhMp*@B*CJaJphhMp*"ܥݥ +,./JKMNצئڦ̷̔zvhZvLzv>zvLhMp*@B*CJaJphhMp*@B*CJaJphhMp*@B*CJaJphhMp*@B*CJaJphhMp*hMp*@B*CJaJphhMp*B*CJaJphhMp*@B*CJaJph)hhMp*@B*CJaJmH phsH )hhMp*@B*CJaJmH phsH hhMp*mH sH )hhMp*@B*CJaJmH phsH %hhMp*B*CJaJmH phsH ڥ,KئDdħk d-DM gdMp* d-DM ^gdMp* d-DM ^gdMp* d-DM ^gdMp* d-DM ^gdMp* d-DM ^gdMp* ڦۦCDFGcdhiçħƧǧݧާqcTh?FlhMp*@B*\phhMp*@B*CJaJphhMp*@B*CJaJphhMp*@B*CJaJphhMp*@B*CJaJphhMp*@B*CJaJphhMp*@B*CJaJphhMp*@B*CJaJphhMp*@B*CJaJphhMp*@B*CJaJphhMp*hMp*@B*CJaJphhMp*B*CJaJph ħާ0D"iko d-DM gdMp* d-DM ^gdMp* d-DM ^gdMp* d-DM gdMp* d-DM ^gdMp* d-DM ^gdMp* /023CDHI&ߩСsgYKYKGsgKGhMp*hMp*@B*CJaJphhMp*@B*CJaJphhMp*B*CJaJphhMp*@B*CJaJph!h?FlhMp*@B*CJaJphh?FlhMp*B*CJaJph!h?FlhMp*@B*CJaJphh?FlhMp*@B*\phh?FlhMp*@B*\ph h?FlhMp*h?FlhMp*@B*phh?FlhMp*@B*\phh?FlhMp*B*\ph!"$%hikljkmnuvxyӷӷӛӍӍӷӍӍӷӷhMp*@B*CJaJphhMp*@B*CJaJphhMp*@B*CJaJphhMp*@B*CJaJphhMp*@B*CJaJphhMp*@B*CJaJphhMp*hMp*@B*CJaJphhMp*B*CJaJphhMp*@B*CJaJph2kv)CRq d-DM gdMp* d-DM ^gdMp* d-DM ^gdMp* d-DM gdMp* d-DM ^gdMp* d-DM ^gdMp* ()+,BCEFQRVWǬȬʬˬܬݬ߬cdfguvӷөӷӍӷӷӷӷөhMp*@B*CJaJphhMp*@B*CJaJphhMp*@B*CJaJphhMp*@B*CJaJphhMp*@B*CJaJphhMp*@B*CJaJphhMp*B*CJaJphhMp*@B*CJaJphhMp*hMp*@B*CJaJph.Ȭݬdvέ.q d-DM gdMp* d -DM ^gdMp* d-DM ^gdMp* d-DM gdMp* d-DM ^gdMp* d-DM ^gdMp* vxyͭέҭӭ-.01HIKLefhiөӍөqcөөhMp*@B*CJaJphhMp*@B*CJaJphhMp*@B*CJaJphhMp*@B*CJaJphhMp*@B*CJaJphhMp*@B*CJaJphhMp*@B*CJaJphhMp*@B*CJaJphhMp*hMp*@B*CJaJphhMp*B*CJaJphhMp*@B*CJaJph&.If&gggg d-DM ^gdMp* d-DM ^gdMp* d-DM ^gdMp* d-DM ^gdMp* d-DM ^gdMp* d-DM ^gdMp*  %&*+xy{|ǯȯʯ˯ <=?@ӷӷӷӷӷqhMp*@B*CJaJphhMp*@B*CJaJphhMp*@B*CJaJphhMp*@B*CJaJphhMp*@B*CJaJphhMp*@B*CJaJphhMp*@B*CJaJphhMp*B*CJaJphhMp*@B*CJaJphhMp*hMp*@B*CJaJph,&yȯ =:G\s d-DM ^gdMp* d-DM ^gdMp* d-DM ^gdMp* d-DM ^gdMp* d-DM ^gdMp* @9:<=FGIJ[\^_rsuvөөөӍӍhMp*@B*CJaJphhMp*@B*CJaJphhMp*@B*CJaJphhMp*@B*CJaJphhMp*@B*CJaJphhMp*@B*CJaJphhMp*B*CJaJphhMp*@B*CJaJphhMp*hMp*@B*CJaJph,*={|زs & F~ d-DM gdMp* & F~ d-DM gdMp* -DM ^ gdMp*-DM ^gdMp* d-DM ^gdMp* d-DM ^gdMp*  )*<=z|ײز#$&'YZ\]ɳʳ̳ͳ휎ŎŀrӪӪӪӪhMp*@B*CJaJphhMp*@B*CJaJphhMp*@B*CJaJphhMp*@B*CJaJphhMp*@B*CJaJphhMp*5>*B*\phhMp*@B*CJaJphhMp*B*CJaJphhMp*@B*CJaJphhMp*hMp*@B*CJaJph,$ZʳAB{{e d-DM ^ gdMp* d-DM gdMp* .d-DM ^gdMp* d-DM gdMp* d-DM gdMp* d -DM gdMp* & F~ d-DM gdMp* @B˴̴δϴ+,./:<@AµõŵƵśśqchMp*@B*CJaJphhMp*@B*CJaJphhMp*@B*CJaJphhMp*@B*CJaJphhMp*@B*CJaJphhMp*@B*CJaJphhMp*@B*CJaJphhMp*B*CJaJphhMp*@B*CJaJphhMp*@B*CJaJphhMp*hMp*@B*CJaJph&̴,;<õյ&mm d-DM ^gdMp* d-DM ^gdMp* d -DM ^gdMp* d-DM gdMp* d-DM ^gdMp* d-DM ^gdMp* ƵԵյ׵ص%&*+h#$&'fgijөөӍӍqӍӍӍөhMp*@B*CJaJphhMp*@B*CJaJphhMp*@B*CJaJphhMp*@B*CJaJphhMp*@B*CJaJphhMp*@B*CJaJphhMp*@B*CJaJphhMp*B*CJaJphhMp*@B*CJaJphhMp*hMp*@B*CJaJph)&$g1CUg d-DM ^gdMp* d-DM ^gdMp*  d-DM ^ gdMp* d-DM ^gdMp* d-DM ^gdMp* fd-DM ]fgdMp* 0134BCEFTUWXjkopǸȸʸ˸ٸڸܸݸ!";<>?XY]^ӛӛӛqhMp*@B*CJaJphhMp*@B*CJaJphhMp*@B*CJaJphhMp*@B*CJaJphhMp*@B*CJaJphhMp*@B*CJaJphhMp*@B*CJaJphhMp*B*CJaJphhMp*@B*CJaJphhMp*hMp*@B*CJaJph,Ukȸڸ<Yg 6d-DM ^`6gdMp* d-DM ^gdMp*  d-DM ^ gdMp* d-DM ^gdMp*  d-DM ^ gdMp* d-DM ^gdMp* dJKMNdeghȻɻ˻̻ŷqqqӷcqhMp*@B*CJaJphhMp*@B*CJaJphhMp*@B*CJaJphhMp*@B*CJaJphhMp*@B*CJaJphhMp*@B*CJaJphhMp*@B*CJaJphhMp*@B*CJaJphhMp*B*CJaJphhMp*@B*CJaJphhMp*hMp*@B*CJaJph&Keɻ.Fi d-DM ^gdMp*  d-DM ^ gdMp* d-DM ^gdMp*  d-DM ^ gdMp* d-DM ^gdMp* d-DM ^gdMp* -.01=EFHI_`bc}~ƺsesWeIhMp*@B*CJaJphhMp*@B*CJaJphhMp*@B*CJaJphhMp*@B*CJaJphhMp*@B*CJaJph hSPhMp*$hSPhMp*@B*CJ\aJph!hSPhMp*@B*CJaJphhMp*B*CJaJphhMp*@B*CJaJphhMp*@B*CJaJphhMp*B*CJaJphhMp*@B*CJaJphhMp*F`~ 1DXYm d -DM gdMp* d-DM ^gdMp* d-DM ^gdMp*  d-DM ^ gdMp* d-DM ^gdMp* d-DM ^gdMp* !"0134CD}h}S}EhMp*@B*CJaJph)hhMp*@B*CJaJmH phsH )hhMp*@B*CJaJmH phsH hhMp*mH sH )hhMp*@B*CJaJmH phsH %hhMp*B*CJaJmH phsH )hhMp*@B*CJaJmH phsH hMp*@B*CJaJphhMp*B*CJaJphhMp*@B*CJaJphhMp*hMp*@B*CJaJphDFGWY]^{өӍqcWIhMp*@B*CJaJphhMp*B*CJaJph"""hMp*@B*CJaJph"""hMp*@B*CJaJphhMp*@B*CJaJphhMp*@B*CJaJphhMp*@B*CJaJphhMp*@B*CJaJphhMp*@B*CJaJphhMp*@B*CJaJphhMp*hMp*@B*CJaJphhMp*B*CJaJphhMp*@B*CJaJphAc (d-DM ^`(gdMp* (d-DM ^`(gdMp* d-DM ^gdMp* d-DM gdMp* (d -DM ^`(gdMp* d -DM ^gdMp* @ACDeŷӛӛ퍁seWӛhMp*@B*CJaJphhMp*@B*CJaJphhMp*@B*CJaJph"""hMp*B*CJaJph"""hMp*@B*CJaJph"""hMp*@B*CJaJphhMp*@B*CJaJphhMp*@B*CJaJphhMp*@B*CJaJphhMp*B*CJaJphhMp*@B*CJaJphhMp*hMp*@B*CJaJphJKMNLMOP!"߷ũߛũߍs߷eߛehMp*@B*CJaJphhMp*B*CJaJph"""hMp*@B*CJaJph"""hMp*@B*CJaJphhMp*@B*CJaJphhMp*@B*CJaJphhMp*@B*CJaJphhMp*B*CJaJphhMp*@B*CJaJphhMp*hMp*@B*CJaJphhMp*@B*CJaJph"""#KM",6@s d-DM gdMp* d-DM ^gdMp* d-DM gdMp* d-DM ^gdMp* d-DM ^gdMp* d-DM gdMp* "$%+,./5689?@DEӱ}qcUGq9GhMp*@B*CJaJphhMp*@B*CJaJphhMp*@B*CJaJphhMp*@B*CJaJphhMp*B*CJaJphhMp*@B*CJaJph'hMp*@B*CJOJQJ^JaJph"""#hMp*B*CJOJQJ^JaJph"""'hMp*@B*CJOJQJ^JaJph"""hMp*@B*CJaJphhMp*hMp*@B*CJaJphhMp*B*CJaJph"""hMp*@B*CJaJph"""| ![}}}}}g d-DM gdMp* d-DM ^gdMp* d-DM gdMp* (d-DM ^`(gdMp* (d -DM ^`(gdMp* d-DM ^gdMp* W_{| ŹӫӏsseWshMp*@B*CJaJph"""hMp*@B*CJaJphhMp*@B*CJaJph"""hMp*@B*CJaJphhMp*@B*CJaJphhMp*@B*CJaJphhMp*@B*CJaJphhMp*B*CJaJph"""hMp*@B*CJaJph"""hMp*hMp*@B*CJaJphhMp*@B*CJaJphhMp*B*CJaJph  !%&Z[]^pqsӷzgRG2)hhMp*@B*CJaJmH ph"""sH hhMp*mH sH )hhMp*@B*CJaJmH phsH %hhMp*B*CJaJmH phsH )hhMp*@B*CJaJmH phsH hMp*@B*CJaJphhMp*B*CJaJphhMp*@B*CJaJphhMp*@B*CJaJphhMp*@B*CJaJph"""hMp*B*CJaJph"""hMp*@B*CJaJph"""hMp*hMp*@B*CJaJph"""[q$+29@Hl d-DM ^gdMp* d-DM ^gdMp* d-DM ^gdMp*  -DM ^ gdMp* }d-DM ^gdMp* }d-DM ^gdMp* st#$&'*+-.12458̾xj\NhMp*@B*CJaJphhMp*@B*CJaJphhMp*@B*CJaJphhMp*@B*CJaJphhMp*B*CJaJphhMp*@B*CJaJphhMp*hMp*@B*CJaJphhMp*B*CJaJph"""hMp*@B*CJaJph"""hhMp*mH sH )hhMp*@B*CJaJmH phsH %hhMp*B*CJaJmH ph"""sH 89;<?@BCGHLMfj Ķ☊zl^RDhMp*@B*CJaJph"""hMp*B*CJaJph"""hMp*@B*CJaJph"""hMp*@B*CJaJphhMp*>*@B*CJaJphhMp*@B*CJaJphhMp*@B*CJaJphhMp*@B*OJQJ^Jph"""hMp*B*OJQJ^JphhMp*@B*OJQJ^JphhMp*@B*CJaJphhMp*B*CJaJphhMp*@B*CJaJphhMp*Hqly_ d-DM ^gdMp*  d-DM ^ gdMp* #d-DM ^`#gdMp* #d-DM ^`#gdMp* d-DM ^gdMp*  d-DM ^ gdMp*lpqstQklpqŷśɁseWIhMp*@B*CJaJphhMp*@B*CJaJph"""hMp*@B*CJaJphhMp*@B*CJaJphhMp*B*CJaJphhMp*@B*CJaJphhMp*@B*CJaJph"""hMp*@B*CJaJphhMp*@B*CJaJph"""hMp*hMp*@B*CJaJphhMp*@B*CJaJph"""hMp*@B*CJaJphhMp*B*CJaJph"""Pfw d-DM ^`gdMp*  d-DM ^ gdMp* d-DM ^gdMp*  d-DM ^ gdMp* d-DM ^gdMp* OPRSeͿ͉͗}oZG2)hhMp*@B*CJaJmH phsH %hhMp*B*CJaJmH phsH )hhMp*@B*CJaJmH phsH hMp*@B*CJaJphhMp*B*CJaJphhMp*@B*CJaJphhMp*@B*CJaJph"""hMp*@B*CJaJphhMp*B*CJaJph"""hMp*@B*CJaJph"""hMp*hMp*@B*OJQJ^Jph"""hMp*B*OJQJ^Jph"""hMp*@B*OJQJ^Jph"""efhivwyz͸wfSB4hMp*@B*CJaJph!hMp*5@B*CJ\aJph$hSPhMp*@B*CJ\aJph hSPhMp*B*CJ\aJph"""(hSPhMp*@B*CJRHm\aJph"""hMp*hMp*@B*CJaJphhMp*B*CJaJph"""hMp*@B*CJaJph""")hhMp*@B*CJaJmH phsH %hhMp*B*CJaJmH ph"""sH )hhMp*@B*CJaJmH ph"""sH hhMp*mH sH (+,./oprsөӏseWIsIhMp*@B*CJaJph"""hMp*@B*CJaJph"""hMp*@B*CJaJph"""hMp*@B*CJaJphhMp*@B*CJaJph"""hMp*@B*CJaJph"""hMp*B*CJaJph"""hMp*@B*CJaJph"""hMp*@B*CJaJphhMp*@B*CJaJphhMp*hMp*@B*CJaJphhMp*B*CJaJphhMp*@B*CJaJph,pygggg d-DM ^gdMp* d-DM gdMp* d-DM ^gdMp* d-DM ^gdMp* d -DM ^gdMp* d-DM ^`gdMp* "%xy{| JKOPVYWXө훏ssөhMp*@B*CJaJphhMp*@B*CJaJph"""hMp*B*CJaJphhMp*@B*CJaJphhMp*@B*CJaJphhMp*@B*CJaJph"""hMp*@B*CJaJph"""hMp*B*CJaJph"""hMp*@B*CJaJph"""hMp*hMp*@B*CJaJph-KXDEmm d -DM ^gdMp* d -DM gdMp* d-DM ^gdMp* d-DM ^gdMp*  d-DM ^ gdMp* d-DM ^gdMp* XZ[^CEIJŷśŁśscP$hMp*5>*@B*CJ\aJphhMp*>*@B*CJaJphhMp*@B*CJaJphhMp*@B*CJaJphhMp*B*CJaJphhMp*@B*CJaJphhMp*@B*CJaJphhMp*@B*CJaJph"""hMp*hMp*@B*CJaJphhMp*@B*CJaJph"""hMp*B*CJaJph"""hMp*@B*CJaJph"""  !#$'(*+ΦΖxj^PhMp*@B*CJaJphhMp*B*CJaJph"""hMp*@B*CJaJph"""hMp*@B*OJQJ^JphhMp*B*OJQJ^Jph"""hMp*@B*OJQJ^Jph"""hMp*@B*CJaJphhMp*B*CJaJphhMp*@B*CJaJphhMp*hMp*@B*CJaJphhMp*@B*CJaJph!hMp*5@B*CJ\aJph!(/oo d-DM ^gdMp* d-DM ^gdMp* d-DM ]gdMp* d-DM ^gdMp* d-DM gdMp*d-DM ^gdMp* +./34ŷycM7+hMp*@B*CJOJQJRHM^JaJph+hMp*@B*CJOJQJRHM^JaJph+hMp*@B*CJOJQJRHM^JaJph+hMp*@B*CJOJQJRHM^JaJph#hMp*B*CJOJQJ^JaJph+hMp*@B*CJOJQJRHM^JaJphhMp*@B*CJaJphhMp*@B*CJaJphhMp*B*CJaJphhMp*@B*CJaJphhMp*hMp*@B*CJaJph]^`acdfgijlmϽ噍噍qcqSEhMp*B*OJQJ^Jph"""hMp*@B*OJQJ^Jph"""hMp*@B*CJaJphhMp*@B*CJaJphhMp*@B*CJaJphhMp*B*CJaJphhMp*@B*CJaJph+hMp*@B*CJOJQJRHM^JaJph#hMp*B*CJOJQJ^JaJph+hMp*@B*CJOJQJRHM^JaJphhMp*+hMp*@B*CJOJQJRHM^JaJph^djqx:DMV`jq d-DM ^gdMp* d-DM ^gdMp* d-DM gdMp* d-DM ^gdMp* d-DM gdMp* d-DM ^gdMp* mpqstwxz{~9:<=CDFGѧj_J)hhMp*@B*CJaJmHphsHhhMp*mHsH)hhMp*@B*CJaJmHphsH%hhMp*B*CJaJmHphsH)hhMp*@B*CJaJmHphsHhMp*@B*CJaJphhMp*@B*CJaJphhMp*@B*CJaJphhMp*B*CJaJphhMp*@B*CJaJphhMp*hMp*@B*OJQJ^JphGLMOPUVXY_`bcijnoKʷߔzvzvhZvLZ>vhMp*@B*CJaJphhMp*@B*CJaJphhMp*@B*CJaJphhMp*@B*CJaJphhMp*hMp*@B*CJaJphhMp*B*CJaJphhMp*@B*CJaJph)hhMp*@B*CJaJmHphsH%hhMp*B*CJaJmHphsH)hhMp*@B*CJaJmHphsHhhMp*mHsH)hhMp*@B*CJaJmHphsHjN9"jmm d-DM ^gdMp* d-DM ^gdMp* d-DM gdMp* d-DM ^gdMp* #d-DM ^`#gdMp* d-DM gdMp* MNPQ89=>!"$%ijlӷӛӍseWhMp*@B*CJaJph"""hMp*@B*CJaJph"""hMp*@B*CJaJphhMp*B*CJaJph"""hMp*@B*CJaJph"""hMp*@B*CJaJphhMp*@B*CJaJphhMp*@B*CJaJphhMp*@B*CJaJphhMp*hMp*@B*CJaJphhMp*B*CJaJphhMp*@B*CJaJph"lmyz|}2VWYZABᷫᝫᏫŁᝫsᝫᝫshMp*@B*CJaJph"""hMp*@B*CJaJphhMp*@B*CJaJphhMp*@B*CJaJphhMp*B*CJaJphhMp*@B*CJaJphhMp*@B*CJaJphhMp*@B*CJaJph"""hMp*hMp*@B*CJaJphhMp*B*CJaJph"""-zWeM d-DM ^gdMp* d -DM ]gdMp* 2d-DM ]^`2gdMp* 2d -DM ^`2gdMp* d -DM ^gdMp* d-DM gdMp* d-DM ^gdMp*BqB`"zk d-DM ^gdMp* d-DM gdMp* #d-DM ^`#gdMp* d-DM gdMp* d-DM ^gdMp* d-DM ^gdMp* BDEpqstABDE !_`bc!"ŹӝӏӏsehMp*@B*CJaJphhMp*@B*CJaJph"""hMp*@B*CJaJphhMp*@B*CJaJphhMp*@B*CJaJphhMp*@B*CJaJphhMp*B*CJaJphhMp*@B*CJaJphhMp*hMp*@B*CJaJphhMp*B*CJaJph"""hMp*@B*CJaJph"""""&'yz|}0134oprs%iŹӝӝӁse׫WhMp*@B*CJaJphhMp*@B*CJaJphhMp*@B*CJaJphhMp*@B*CJaJph"""hMp*@B*CJaJphhMp*@B*CJaJph"""hMp*@B*CJaJphhMp*B*CJaJph"""hMp*@B*CJaJph"""hMp*hMp*@B*CJaJphhMp*B*CJaJphhMp*@B*CJaJph"1p3Llo d-DM gdMp* d-DM ^gdMp* d-DM ^gdMp* d-DM gdMp* d-DM ^gdMp* d-DM ^gdMp* 2356KLNOklno!"$%noqrŹӝӁsesӝӝŹWhMp*@B*CJaJph"""hMp*@B*CJaJph"""hMp*@B*CJaJphhMp*@B*CJaJphhMp*@B*CJaJphhMp*@B*CJaJphhMp*@B*CJaJphhMp*B*CJaJph"""hMp*@B*CJaJph"""hMp*hMp*@B*CJaJphhMp*B*CJaJphhMp*@B*CJaJph""o>Jaq d-DM ^gdMp* d-DM ^gdMp* d-DM gdMp* d-DM ^gdMp* d-DM ^gdMp* =>@AIJLM`aӷ픁laL7a)hhMp*@B*CJaJmH phsH )hhMp*@B*CJaJmH phsH hhMp*mH sH )hhMp*@B*CJaJmH phsH %hhMp*B*CJaJmH phsH )hhMp*@B*CJaJmH phsH hMp*@B*CJaJph"""hMp*@B*CJaJphhMp*@B*CJaJphhMp*B*CJaJphhMp*@B*CJaJphhMp*hMp*@B*CJaJphacdpqstpSTVWөӛseWӛsӛhMp*@B*CJaJphhMp*@B*CJaJphhMp*@B*CJaJphhMp*@B*CJaJphhMp*B*CJaJphhMp*@B*CJaJphhMp*@B*CJaJph"""hMp*@B*CJaJphhMp*@B*CJaJph"""hMp*hMp*@B*CJaJphhMp*B*CJaJph"""hMp*@B*CJaJph"""!T?}cI d-DM ^gdMp* .d-DM ^gdMp* .d-DM ^`gdMp* .d -DM ^gdMp* .d-DM ^gdMp* d-DM ^`gdMp* d-DM ^gdMp* x>?EFHIYZ\]jkmn{|~өӍzl^hMp*@B*CJaJphhMp*@B*CJaJph$hMp*5>*@B*CJ\aJphhMp*@B*CJaJphhMp*@B*CJaJphhMp*@B*CJaJphhMp*@B*CJaJphhMp*@B*CJaJphhMp*hMp*@B*CJaJphhMp*B*CJaJphhMp*@B*CJaJph#?FZk|)Yooo & F -d-DM ^gdMp* & F -d-DM ^gdMp*+ -DM ^+gdMp*&-DM ^&gdMp* *d-DM ^gdMp*"d -DM ^"gdMp* ()<=XYmnrsDEɻ孟僟僑qqcUqhMp*@B*CJaJphhMp*@B*CJaJphhMp*@B*CJaJphhMp*hMp*@B*CJaJphhMp*@B*CJaJphhMp*@B*CJaJphhMp*@B*CJaJphhMp*@B*CJaJphhMp*@B*CJaJphhMp*@B*CJaJphhMp*B*CJaJphhMp*@B*CJaJph!YnElV d-DM gdMp*  -DM ^gdMp* d-DM ^gdMp* "d-DM ]"^gdMp* d-DM ^gdMp* d-DM ]gdMp*~d-DM ^~gdMp*EIJ[\`a()+,XY[\mnӷөөӛӷөhMp*@B*CJaJphhMp*@B*CJaJphhMp*@B*CJaJphhMp*@B*CJaJphhMp*@B*CJaJphhMp*@B*CJaJphhMp*hMp*@B*CJaJphhMp*B*CJaJphhMp*@B*CJaJph,\)Ynu -DM gdMp* d-DM ^gdMp* d-DM ^gdMp* -DM gdMp* d-DM ^gdMp* d-DM ^gdMp* nrs-/34өөӷӛqcqhMp*@B*CJaJphhMp*@B*CJaJphhMp*@B*CJaJphhMp*@B*CJaJphhMp*@B*CJaJphhMp*@B*CJaJphhMp*@B*CJaJphhMp*@B*CJaJphhMp*hMp*@B*CJaJphhMp*B*CJaJphhMp*@B*CJaJph$./mm d-DM ^gdMp* d-DM ^gdMp* d -DM ^gdMp* d-DM gdMp* d-DM ^gdMp* d-DM ^gdMp* bhikl-.01;өӛ卷өqchMp*@B*CJaJphhMp*@B*CJaJphhMp*@B*CJaJphhMp*@B*CJaJphhMp*@B*CJaJphhMp*@B*CJaJphhMp*@B*CJaJphhMp*@B*CJaJphhMp*hMp*@B*CJaJphhMp*B*CJaJphhMp*@B*CJaJph%i.Hmmm d-DM ^gdMp* d-DM ^gdMp* d-DM gdMp* d-DM ^gdMp* -d-DM ^`-gdMp* d-DM gdMp* ;GHLMөӍ}o_QӍhMp*@B*CJaJphhMp*@B*OJQJ^JphhMp*B*OJQJ^JphhMp*@B*OJQJ^JphhMp*@B*CJaJphhMp*@B*CJaJphhMp*@B*CJaJphhMp*@B*CJaJphhMp*@B*CJaJphhMp*B*CJaJphhMp*@B*CJaJphhMp*hMp*@B*CJaJph@KLNOSTVW\]abŷߩŷߛߍߍśqqśahMp*@B*OJQJ^JphhMp*@B*CJaJphhMp*@B*CJaJphhMp*@B*CJaJphhMp*@B*CJaJphhMp*@B*CJaJphhMp*@B*CJaJphhMp*B*CJaJphhMp*@B*CJaJphhMp*hMp*@B*CJaJphhMp*@B*CJaJph LT]o d-DM gdMp* d-DM ^gdMp* d-DM ^gdMp* d-DM gdMp* d-DM ^gdMp* d-DM ]gdMp*  !=>@ATUWXõݧÙݧË}ÙݙËoÙ݋õaݧaݧhMp*@B*CJaJphhMp*@B*CJaJphhMp*@B*CJaJphhMp*@B*CJaJphhMp*@B*CJaJphhMp*@B*CJaJphhMp*@B*CJaJphhMp*B*CJaJphhMp*@B*CJaJphhMp*hMp*@B*OJQJ^JphhMp*B*OJQJ^Jph&>Um d-DM ^gdMp* d-DM ^gdMp* d-DM ^gdMp* d-DM gdMp* d-DM ^gdMp* d-DM ^gdMp* X&')*3467s^S>s)hhMp*@B*CJaJmH phsH hhMp*mH sH )hhMp*@B*CJaJmH phsH %hhMp*B*CJaJmH phsH )hhMp*@B*CJaJmH phsH hMp*@B*CJaJphhMp*@B*CJaJphhMp*@B*CJaJphhMp*@B*CJaJphhMp*B*CJaJphhMp*@B*CJaJphhMp*hMp*@B*CJaJph'4@Xt%Igq d-DM ^gdMp* d-DM ^gdMp* d -DM ^gdMp* d-DM gdMp* d-DM ^gdMp* d-DM gdMp* 7?@BCWXZ[stvwʷߔzvhZvZLv>hMp*@B*CJaJphhMp*@B*CJaJphhMp*@B*CJaJphhMp*@B*CJaJphhMp*hMp*@B*CJaJphhMp*B*CJaJphhMp*@B*CJaJph)hhMp*@B*CJaJmH phsH %hhMp*B*CJaJmH phsH )hhMp*@B*CJaJmH phsH hhMp*mH sH )hhMp*@B*CJaJmH phsH  $%'(HIKLfgij 89;<YZ\]өqөchMp*@B*CJaJphhMp*@B*CJaJphhMp*@B*CJaJphhMp*@B*CJaJphhMp*@B*CJaJphhMp*@B*CJaJphhMp*@B*CJaJphhMp*@B*CJaJphhMp*B*CJaJphhMp*@B*CJaJphhMp*hMp*@B*CJaJph& 9Z^po d-DM ^gdMp* d-DM ^gdMp* d-DM gdMp* d-DM ^gdMp* d-DM ^gdMp* d-DM gdMp* ]^`aoprsKөӛӍqchMp*@B*CJaJphhMp*@B*CJaJphhMp*@B*CJaJphhMp*@B*CJaJphhMp*@B*CJaJphhMp*@B*CJaJphhMp*@B*CJaJphhMp*@B*CJaJphhMp*hMp*@B*CJaJphhMp*B*CJaJphhMp*@B*CJaJph&&_ L1X d-DM gdMp*$d-DM ]^a$gdMp* d-DM ^gdMp* d-DM ^gdMp* d-DM gdMp*%&()^_cd KL^ohMp*>*@B*CJaJphhMp*@B*CJaJphhMp*@B*CJaJphhMp*@B*CJaJphhMp*@B*CJaJphhMp*@B*CJaJphhMp*@B*CJaJphhMp*@B*CJaJphhMp*hMp*@B*CJaJphhMp*B*CJaJph(^!0156WXZ[ ÷ѩћqcUhMp*@B*CJaJphhMp*@B*CJaJphhMp*@B*CJaJphhMp*@B*CJaJphhMp*@B*CJaJphhMp*@B*CJaJphhMp*@B*CJaJphhMp*B*CJaJphhMp*@B*CJaJphhMp*hMp*@B*CJaJphhMp*@B*CJaJphhMp*@B*CJaJph Xa&deq d-DM gdMp* d-DM ^gdMp* d-DM ^gdMp* d-DM gdMp* d-DM ^gdMp* d-DM ^gdMp* `acd%&()ceij?ӷөӛ׍qcUhMp*@B*CJaJphhMp*@B*CJaJphhMp*@B*CJaJphhMp*@B*CJaJphhMp*@B*CJaJphhMp*@B*CJaJphhMp*@B*CJaJphhMp*@B*CJaJphhMp*@B*CJaJphhMp*hMp*@B*CJaJphhMp*B*CJaJphhMp*@B*CJaJph!N[fxod-DM ^gdMp* d-DM ^gdMp* d-DM ^gdMp* d-DM gdMp* d-DM ^gdMp* d -DM ^gdMp*   MNPQZ[]^efhiseWIhMp*@B*CJaJph"""hMp*@B*CJaJphhMp*@B*CJaJph"""hMp*@B*CJaJphhMp*@B*CJaJphhMp*@B*CJaJphhMp*@B*CJaJphhMp*B*CJaJphhMp*@B*CJaJphhMp*@B*CJaJphhMp*B*CJaJph"""hMp*@B*CJaJph"""hMp*hMp*@B*CJaJphiwxz{t ABDEŝq\I%hhMp*B*CJaJmHphsH)hhMp*@B*CJaJmHphsHhMp*@B*CJaJphhMp*@B*CJaJphhMp*>*@B*CJaJphhMp*@B*CJaJphhMp*B*CJaJphhMp*@B*CJaJphhMp*@B*CJaJphhMp*B*CJaJph"""hMp*@B*CJaJph"""hMp*hMp*@B*CJaJphBJS\eo )6 sd-DM ^gdMp* d-DM ^gdMp* d-DM gdMp* d-DM ^gdMp* d-DM ^gdMp* d-DM gdMp* EIJLMRSUV[\^_deghnoʷߍzeߢWK=9hMp*hMp*@B*CJaJphhMp*B*CJaJphhMp*@B*CJaJph)hhMp*@B*CJaJmHphsH%hhMp*B*CJaJmHph"""sH)hhMp*@B*CJaJmHph"""sH)hhMp*@B*CJaJmHphsH%hhMp*B*CJaJmHphsH)hhMp*@B*CJaJmHphsHhhMp*mHsH)hhMp*@B*CJaJmHphsHost  ()+,56H      ŹӫӏӁŹscUŹhMp*@B*CJaJphhMp*>*@B*CJaJphhMp*@B*CJaJphhMp*@B*CJaJphhMp*@B*CJaJphhMp*@B*CJaJphhMp*@B*CJaJphhMp*B*CJaJphhMp*@B*CJaJphhMp*hMp*@B*CJaJphhMp*B*CJaJph"""hMp*@B*CJaJph"""!   P   K z   kk d-DM ^gdMp* d-DM gdMp* -d-DM ^`-gdMp* d-DM ^gdMp* d-DM ^gdMp* d-DM gdMp*       O P R S          J K O P y z | }     ӷ坏ӷseWhMp*@B*CJaJphhMp*@B*CJaJphhMp*@B*CJaJph"""hMp*@B*CJaJphhMp*@B*CJaJphhMp*@B*CJaJphhMp*B*CJaJph"""hMp*@B*CJaJph"""hMp*@B*CJaJphhMp*hMp*@B*CJaJphhMp*B*CJaJphhMp*@B*CJaJph                }hS}<,hhMp*>*@B*CJaJmH phsH )hhMp*@B*CJaJmH phsH )hhMp*@B*CJaJmH ph"""sH hhMp*mH sH )hhMp*@B*CJaJmH phsH %hhMp*B*CJaJmH ph"""sH )hhMp*@B*CJaJmH ph"""sH hMp*@B*CJaJphhMp*B*CJaJph"""hMp*@B*CJaJph"""hMp*hMp*@B*CJaJph       / X q y^ & F d-DM gdMp* & F d -DM gdMp* & F d-DM gdMp* -DM ^ gdMp* -DM gdMp* d-DM ^gdMp* d-DM ^gdMp*       . / A B W d e p q               # $ < = Z [     ͳ͗ͥ۳۳{f)hhMp*@B*CJaJmH phsH hMp*@B*CJaJphhMp*@B*CJaJphhMp*@B*CJaJphhMp*@B*CJaJphhMp*@B*CJaJphhMp*B*CJaJphhMp*@B*CJaJphhMp*@B*CJaJphhMp*$hMp*5>*@B*CJ\aJph'q    $ [     $6i d-DM gdMp* d-DM ^gdMp* d-DM ^gdMp* & F d-DM gdMp* & F d-DM gdMp* & F d-DM gdMp*              #$&'56:;̷̢̔zvhZvLzvhhMp*@B*CJaJphhMp*@B*CJaJphhMp*@B*CJaJphhMp*hMp*@B*CJaJphhMp*B*CJaJphhMp*@B*CJaJph)hhMp*@B*CJaJmH phsH )hhMp*@B*CJaJmH phsH hhMp*mH sH )hhMp*@B*CJaJmH phsH %hhMp*B*CJaJmH phsH R}hS}}E7hMp*@B*CJaJphhMp*@B*CJaJph)hhMp*@B*CJaJmH phsH )hhMp*@B*CJaJmH phsH hhMp*mH sH )hhMp*@B*CJaJmH phsH %hhMp*B*CJaJmH phsH )hhMp*@B*CJaJmH phsH hMp*@B*CJaJphhMp*B*CJaJphhMp*@B*CJaJphhMp*hMp*@B*CJaJphSj(o d-DM gdMp* d-DM ^gdMp* d-DM gdMp* d-DM ^gdMp* d-DM ^gdMp* d -DM ^gdMp* RSUVijlm'(*+<=?@IJLMrdhMp*@B*CJaJphhMp*@B*CJaJphhMp*@B*CJaJphhMp*@B*CJaJphhMp*@B*CJaJphhMp*@B*CJaJphhMp*@B*CJaJphhMp*@B*CJaJphhMp*@B*CJaJphhMp*B*CJaJphhMp*@B*CJaJphhMp*#(=Jar.o d-DM gdMp* d-DM ^gdMp* d-DM ^gdMp* d-DM gdMp* d-DM ^gdMp* d-DM ^gdMp* M`acdqrtu-.23ӛӛqӛӛqhMp*@B*CJaJphhMp*@B*CJaJphhMp*@B*CJaJphhMp*@B*CJaJphhMp*@B*CJaJphhMp*@B*CJaJphhMp*@B*CJaJphhMp*B*CJaJphhMp*@B*CJaJphhMp*hMp*@B*CJaJph*CP^uoU d -DM ^gdMp* d-DM ^gdMp* d-DM ^gdMp* d-DM gdMp*d-DM ^gdMp* d-DM ]^gdMp* d-DM ^gdMp* BCEFOPRS]^`atuwx巩ӛӍӍqcqhMp*@B*CJaJphhMp*@B*CJaJphhMp*@B*CJaJphhMp*@B*CJaJphhMp*@B*CJaJphhMp*@B*CJaJphhMp*@B*CJaJphhMp*@B*CJaJphhMp*hMp*@B*CJaJphhMp*B*CJaJphhMp*@B*CJaJph&'4CqY d-DM ^gdMp* d-DM gdMp* d-DM ^gdMp* d-DM ^gdMp* d -DM ^gdMp* d-DM gdMp* d-DM ^gdMp*   &')*3467BCGHԿԊԿԿ|pbTpTFphMp*@B*CJaJphhMp*@B*CJaJphhMp*@B*CJaJphhMp*B*CJaJphhMp*@B*CJaJph)hhMp*@B*CJaJmH phsH )hhMp*@B*CJaJmH phsH hhMp*mH sH )hhMp*@B*CJaJmH phsH %hhMp*B*CJaJmH phsH )hhMp*@B*CJaJmH phsH hMp*"#'(rsuv3467ӷөӷӷөhMp*@B*CJaJphhMp*@B*CJaJphhMp*@B*CJaJphhMp*@B*CJaJphhMp*B*CJaJphhMp*@B*CJaJphhMp*hMp*@B*CJaJph=#so d-DM gdMp* d-DM ^gdMp* d-DM ^gdMp* d-DM gdMp* d-DM ^gdMp* d-DM ^gdMp* 48@Io d-DM ^gdMp* d-DM gdMp* d-DM ^gdMp* d-DM gdMp* d-DM ^`gdMp* d-DM ^gdMp* 78:;?@BCHIMNGHLMӷөӍqchMp*@B*CJaJphhMp*@B*CJaJphhMp*@B*CJaJphhMp*@B*CJaJphhMp*@B*CJaJphhMp*@B*CJaJphhMp*@B*CJaJphhMp*@B*CJaJphhMp*B*CJaJphhMp*@B*CJaJphhMp*hMp*@B*CJaJph&H4Ko d-DM gdMp* d-DM ^gdMp* d-DM ^gdMp* d-DM gdMp* d-DM ^gdMp* d-DM ^gdMp*  3467JKOP#rhMp*@B*CJaJphhMp*@B*CJaJphhMp*@B*CJaJphhMp*@B*CJaJphhMp*@B*CJaJphhMp*@B*CJaJphhMp*@B*CJaJphhMp*@B*CJaJphhMp*B*CJaJphhMp*@B*CJaJphhMp*,$#x -DM ^gdMp* d-DM ^gdMp* d-DM gdMp* d-DM ^gdMp* d-DM gdMp* d-DM ^gdMp* #$()"#'(xy{|rhMp*@B*CJaJphhMp*@B*CJaJphhMp*@B*CJaJphhMp*@B*CJaJphhMp*@B*CJaJphhMp*@B*CJaJphhMp*@B*CJaJphhMp*@B*CJaJphhMp*B*CJaJphhMp*@B*CJaJphhMp*,#yT\xbH d-DM ^gdMp* d-DM gdMp* d-DM ^gdMp* d -DM ^gdMp* d-DM gdMp* -DM ^gdMp* -DM ^gdMp* -DM gdMp*KSTVW[\^_deijӷӛӷqchMp*@B*CJaJphhMp*@B*CJaJphhMp*@B*CJaJphhMp*@B*CJaJphhMp*@B*CJaJphhMp*@B*CJaJphhMp*@B*CJaJphhMp*@B*CJaJphhMp*B*CJaJphhMp*@B*CJaJphhMp*hMp*@B*CJaJph#\e[loW d-DM ^gdMp* d-DM ^gdMp* d-DM gdMp* d-DM ^gdMp* d-DM ^gdMp* d-DM gdMp* d-DM ^gdMp* SZ[]^klno~4578KLNO\]_`mnө婷ӍӛhMp*@B*CJaJphhMp*@B*CJaJphhMp*@B*CJaJphhMp*@B*CJaJphhMp*@B*CJaJphhMp*@B*CJaJphhMp*hMp*@B*CJaJphhMp*B*CJaJphhMp*@B*CJaJph15L]n Wqqq d-DM ^gdMp* d-DM gdMp* d-DM ^gdMp* d-DM ^gdMp* d-DM gdMp* d-DM ^gdMp* npq  VWYZ0156   ӷөөӛ׍ӷөӷӛhMp*@B*CJaJphhMp*@B*CJaJphhMp*@B*CJaJphhMp*@B*CJaJphhMp*@B*CJaJphhMp*@B*CJaJphhMp*hMp*@B*CJaJphhMp*B*CJaJphhMp*@B*CJaJph21 A    qq d-DM ^gdMp* d-DM gdMp* d-DM ^gdMp* d-DM ^gdMp* d-DM gdMp* d-DM ^gdMp*   @ A E F                         a!!!!!!!qcqhMp*@B*CJaJphhMp*@B*CJaJphhMp*@B*CJaJphhMp*@B*CJaJphhMp*@B*CJaJphhMp*@B*CJaJphhMp*@B*CJaJphhMp*@B*CJaJphhMp*@B*CJaJphhMp*hMp*@B*CJaJphhMp*B*CJaJph$     !!!!!m d-DM ^gdMp* d-DM ^gdMp* d-DM gdMp* d-DM ^gdMp* d-DM ^gdMp* d -DM ^gdMp* !!!!!!!!!!!!!!!!!!!Z"g"h"j"k"o"p"r"s"x"y"}"~"""өө囍qcUhMp*@B*CJaJphhMp*@B*CJaJphhMp*@B*CJaJphhMp*@B*CJaJphhMp*@B*CJaJphhMp*@B*CJaJphhMp*@B*CJaJphhMp*@B*CJaJphhMp*@B*CJaJphhMp*hMp*@B*CJaJphhMp*B*CJaJphhMp*@B*CJaJph!!!h"p"y"""#"#5#q#oooY d-DM gdMp* d-DM ^gdMp* d -DM ^gdMp* d-DM gdMp* d -DM ^gdMp* d -DM gdMp* d -DM ^gdMp* """""""## # #!#"#$#%#4#5#9#:#p#q#s#t#########ӷӛӍqcUӷhMp*@B*CJaJphhMp*@B*CJaJphhMp*@B*CJaJphhMp*@B*CJaJphhMp*@B*CJaJphhMp*@B*CJaJphhMp*@B*CJaJphhMp*@B*CJaJphhMp*@B*CJaJphhMp*hMp*@B*CJaJphhMp*B*CJaJphhMp*@B*CJaJphq####$>$$$Y%%%h&i d-DM ^gdMp* -d-DM ^`-gdMp* d-DM ^gdMp* d-DM gdMp* d-DM ^gdMp* d-DM ^gdMp* ###$$$$$$=$>$B$C$$$$$$$$$C%X%Y%[%\%%%%%%%%&g&h&өӍӍqөcӍcӍcӍhMp*@B*CJaJphhMp*@B*CJaJphhMp*@B*CJaJphhMp*@B*CJaJphhMp*@B*CJaJphhMp*@B*CJaJphhMp*@B*CJaJphhMp*@B*CJaJphhMp*B*CJaJphhMp*@B*CJaJphhMp*hMp*@B*CJaJph#h&l&m&&&&&&&&&&'' ' '3'4'6'7'V'W'Y'Z'|'}'''''ŷśqcUhMp*@B*CJaJphhMp*@B*CJaJphhMp*@B*CJaJphhMp*@B*CJaJphhMp*@B*CJaJphhMp*@B*CJaJphhMp*@B*CJaJphhMp*@B*CJaJphhMp*hMp*@B*CJaJphhMp*@B*CJaJphhMp*B*CJaJphhMp*@B*CJaJphh&&&'4'W'}''''''' (!(X( d-DM ^gdMp* d-DM gdMp* d-DM ^gdMp* d-DM ^gdMp* d-DM gdMp*''''''''''''''''''''''' ( ((( (!(%(&(W(X(Z([(l(m(өӍqcөhMp*@B*CJaJphhMp*@B*CJaJphhMp*@B*CJaJphhMp*@B*CJaJphhMp*@B*CJaJphhMp*@B*CJaJphhMp*@B*CJaJphhMp*@B*CJaJphhMp*hMp*@B*CJaJphhMp*B*CJaJphhMp*@B*CJaJph$X(m((((((()):))q d-DM gdMp* d-DM ^gdMp* d -DM ^gdMp* d-DM gdMp* d-DM ^gdMp* d-DM ^gdMp* m(o(p((((((((((((((((((((())))))) )9):)>)өӛqcUhMp*@B*CJaJphhMp*@B*CJaJphhMp*@B*CJaJphhMp*@B*CJaJphhMp*@B*CJaJphhMp*@B*CJaJphhMp*@B*CJaJphhMp*@B*CJaJphhMp*@B*CJaJphhMp*hMp*@B*CJaJphhMp*B*CJaJphhMp*@B*CJaJph!>)?)))))))))))))))))))))))**6*7*9*:*I*J*L*M*d*e*g*h***********hMp*@B*CJaJphhMp*@B*CJaJphhMp*@B*CJaJphhMp*@B*CJaJphhMp*@B*CJaJphhMp*@B*CJaJphhMp*@B*CJaJphhMp*hMp*@B*CJaJphhMp*B*CJaJph/))))))7*J*e*****kk d-DM ^gdMp* d-DM ^gdMp* d-DM gdMp* d-DM ^gdMp* d-DM ^gdMp* d -DM ^gdMp* ******* ++++,+-+/+0+@+A+E+F+++++++++++++ , ,,, ,!,#,$,4,5,9,:,~,,өөӛөӍөөqhMp*@B*CJaJphhMp*@B*CJaJphhMp*@B*CJaJphhMp*@B*CJaJphhMp*@B*CJaJphhMp*@B*CJaJphhMp*@B*CJaJphhMp*hMp*@B*CJaJphhMp*B*CJaJphhMp*@B*CJaJph,*+-+A++++ ,!,5,,,q d-DM ^gdMp* d-DM gdMp* d-DM ^gdMp* d-DM ^gdMp* d-DM gdMp* d-DM ^gdMp* ,,,,,,,,,,,,,,,,,,,-.u.v.x.y......... / / / /T/U/W/X/k/l/p/q/өӛ׍өөөөqhMp*@B*CJaJphhMp*@B*CJaJphhMp*@B*CJaJphhMp*@B*CJaJphhMp*@B*CJaJphhMp*@B*CJaJphhMp*@B*CJaJphhMp*hMp*@B*CJaJphhMp*B*CJaJphhMp*@B*CJaJph,,,,,v... /U/l///0mm d-DM ^gdMp* d-DM ^gdMp* d-DM ^gdMp* d-DM gdMp* d-DM ^gdMp* d -DM ^gdMp* q/////////000010204050Ű}h]OC5ChMp*@B*CJaJphhMp*B*CJaJphhMp*@B*CJaJphhhMp*mHsH)hhMp*@B*CJaJmHphsH%hhMp*B*CJaJmHphsH)hhMp*@B*CJaJmHphsHhhMp*mH sH )hhMp*@B*CJaJmH phsH %hhMp*B*CJaJmH phsH )hhMp*@B*CJaJmH phsH hMp*hMp*@B*CJaJph020Q0l0m0061t11 2L22o d-DM gdMp* d-DM ^gdMp* d-DM ^gdMp* -d-DM ^`-gdMp* d-DM gdMp* d-DM ^gdMp* 50P0Q0S0T0k0m0q0r000002151618191s1t1v1w11111 2 2 2 2K2L2P2Q222222233I3J3өōөөөөqөөөhMp*@B*CJaJphhMp*@B*CJaJphhMp*@B*CJaJphhMp*@B*CJaJphhMp*@B*CJaJphhMp*@B*CJaJphhMp*@B*CJaJphhMp*B*CJaJphhMp*@B*CJaJphhMp*hMp*@B*CJaJph+22J33,4~4445(5R5c5e d-DM ^gdMp* d-DM ^gdMp* d-DM gdMp* d-DM ^gdMp* ;d-DM ]^`;gdMp* d-DM ^gdMp* J3L3M333333+4,4.4/4}4~444444444445555'5(5*5+5Q5R5T5U5b5c5g5h555555ŷŷũŷŷŷqũŷhMp*@B*CJaJphhMp*@B*CJaJphhMp*@B*CJaJphhMp*@B*CJaJphhMp*@B*CJaJphhMp*@B*CJaJphhMp*hMp*@B*CJaJphhMp*@B*CJaJphhMp*B*CJaJphhMp*@B*CJaJph,c5555555666t66o d-DM ^gdMp* d-DM ^gdMp* d-DM gdMp* d-DM ^gdMp* d-DM ^gdMp* d-DM gdMp* 55555555555555555566666656668696s6t6v6w66666666666666rhMp*@B*CJaJphhMp*@B*CJaJphhMp*@B*CJaJphhMp*@B*CJaJphhMp*@B*CJaJphhMp*@B*CJaJphhMp*@B*CJaJphhMp*@B*CJaJphhMp*B*CJaJphhMp*@B*CJaJphhMp*,66666K7S7\7777oo d-DM ^gdMp* d-DM ^gdMp* d-DM gdMp*  d-DM ] ^gdMp* d-DM ^gdMp* d-DM gdMp* 666666677J7K7M7N7R7S7U7V7[7\7`7a7777777777777888 8 888888rhMp*@B*CJaJphhMp*@B*CJaJphhMp*@B*CJaJphhMp*@B*CJaJphhMp*@B*CJaJphhMp*@B*CJaJphhMp*@B*CJaJphhMp*@B*CJaJphhMp*B*CJaJphhMp*@B*CJaJphhMp**7888v8~8888oo[ d-DM gdMp* d-DM ^gdMp* d-DM ^gdMp* d -DM gdMp* d-DM ^gdMp* d-DM ^gdMp* d-DM gdMp*888u8v8x8y8}8~8888888888899999 999999999999өӛqӷӷcӷhMp*@B*CJaJphhMp*@B*CJaJphhMp*@B*CJaJphhMp*@B*CJaJphhMp*@B*CJaJphhMp*@B*CJaJphhMp*@B*CJaJphhMp*@B*CJaJphhMp*hMp*@B*CJaJphhMp*B*CJaJphhMp*@B*CJaJph#8999999990:s] d-DM gdMp* d-DM ^gdMp* d-DM ^gdMp* d -DM ^gdMp* d -DM gdMp* -DM ^gdMp* -DM ^gdMp* 99999999999999999/:0:2:3:J:K:M:N:`:a:c:d:m:n:p:q:::::::::::::ӛӍӛhMp*@B*CJaJphhMp*@B*CJaJphhMp*@B*CJaJphhMp*@B*CJaJphhMp*@B*CJaJphhMp*@B*CJaJphhMp*B*CJaJphhMp*@B*CJaJphhMp*hMp*@B*CJaJph,0:K:a:n::::;:;];;;<<%<6<J< d-DM ^gdMp* d-DM gdMp* d-DM ^gdMp* d-DM gdMp* d-DM ^gdMp*:;;;;9;:;<;=;\;];_;`;;;;;;;;;<<<<<<<<$<%<'<(<5<ӛӛumcUhMp*@B*CJaJphhMp*@B*phhMp*B*phhMp*@B*phhMp*@B*CJaJphhMp*@B*CJaJphhMp*@B*CJaJphhMp*@B*CJaJphhMp*@B*CJaJphhMp*@B*CJaJphhMp*B*CJaJphhMp*@B*CJaJphhMp*hMp*@B*CJaJph!5<6<8<9<I<J<L<M<Z<[<_<`<<<<<<<<<<<<<<<<=====(=)=-=.=S=T=V=W=_=`=b=c=rhMp*@B*CJaJphhMp*@B*CJaJphhMp*@B*CJaJphhMp*@B*CJaJphhMp*@B*CJaJphhMp*@B*CJaJphhMp*@B*CJaJphhMp*@B*CJaJphhMp*B*CJaJphhMp*@B*CJaJphhMp*+J<[<<<<<=)=T=`=m=qW d-DM ^gdMp* d-DM ^gdMp* d-DM gdMp* d-DM ^gdMp* d-DM ^gdMp* d-DM gdMp* d-DM ^gdMp* c=l=m=o=p=y=z=|=}=========x>>?@@@B@C@E@F@H@I@K@L@N@O@R@ӛoaQhMp*@B*OJQJ^JphhMp*B*OJQJ^JphhMp*@B*OJQJ^JphhMp*@B*CJaJphhMp*@B*CJaJphhMp*@B*CJaJphhMp*@B*CJaJphhMp*@B*CJaJphhMp*@B*CJaJphhMp*B*CJaJphhMp*@B*CJaJphhMp*hMp*@B*CJaJphm=z===@@F@L@S@Z@a@b@ C"Co d-DM ^gdMp* d-DM ^gdMp* d -DM ^gdMp* d-DM ^gdMp* d-DM gdMp* d-DM ^gdMp* R@S@U@V@Y@Z@\@]@`@b@f@g@@;AABBC C C C!C"C$C%CVCWCYCZCqCrCtCuCCCЦИ|ИnbnbhMp*B*CJaJph"""hMp*@B*CJaJph"""hMp*@B*CJaJphhMp*@B*CJaJphhMp*@B*CJaJphhMp*@B*CJaJphhMp*@B*CJaJphhMp*@B*CJaJphhMp*B*CJaJphhMp*@B*CJaJphhMp*B*phhMp*@B*phhMp*""CWCrCCCDDE)EHEdEG#Gi d-DM ^gdMp* d-DM ^gdMp* d-DM ^gdMp* d-DM ^gdMp* d -DM ^gdMp* d-DM ^gdMp* CCCCCCCLDDDDDDDDDDEE E E(E)E+E,EGEHEJEKEcEdEhEiE?FF׷ӛqeӛW卷hMp*@B*CJaJphhMp*B*CJaJph"""hMp*@B*CJaJph"""hMp*@B*CJaJphhMp*@B*CJaJphhMp*@B*CJaJphhMp*@B*CJaJphhMp*@B*CJaJphhMp*@B*CJaJphhMp*hMp*@B*CJaJphhMp*B*CJaJphhMp*@B*CJaJph"FGGGG"G#G%G&GnGoGqGrGGGGGGGGG1H2H6H7H IzIITJJKK K KKŹ흹ӫӫӫseWhMp*@B*CJaJphhMp*@B*CJaJphhMp*@B*CJaJphhMp*@B*CJaJphhMp*@B*CJaJphhMp*@B*CJaJph"""hMp*@B*CJaJphhMp*B*CJaJph"""hMp*@B*CJaJph"""hMp*B*CJaJphhMp*@B*CJaJphhMp*hMp*@B*CJaJph"#GoGGG2HKK2KGKKLLLmm d-DM ^gdMp* d-DM gdMp* d-DM ^gdMp* d-DM ^gdMp* d-DM ^gdMp* d-DM ^gdMp* KKK K1K2K4K5KFKGKIKJKKKKK LyLLLLLLLƺԞ{hSHhhMp*mHsH)hhMp*@B*CJaJmHphsH%hhMp*B*CJaJmHphsH)hhMp*@B*CJaJmHphsHhMp*@B*CJaJphhMp*@B*CJaJphhMp*@B*CJaJphhMp*B*CJaJph"""hMp*@B*CJaJph"""hMp*@B*CJaJphhMp*B*CJaJphhMp*@B*CJaJphhMp*LLLLLLLLLLLLLLLMMM··ugcUuGc9hMp*@B*CJaJph"""hMp*@B*CJaJphhMp*@B*CJaJphhMp*hMp*@B*CJaJphhMp*B*CJaJphhMp*@B*CJaJph%hhMp*B*CJaJmHph"""sH)hhMp*@B*CJaJmHph"""sHhhMp*mHsH)hhMp*@B*CJaJmHphsH%hhMp*B*CJaJmHphsH)hhMp*@B*CJaJmHphsHLLLMMNN?N{N|N~NNNNNN O OOOiOjOlOŹӏӁsseP)hhMp*@B*CJaJmHphsHhMp*@B*CJaJphhMp*@B*CJaJph"""hMp*@B*CJaJph"""hMp*@B*CJaJph"""hMp*@B*CJaJphhMp*@B*CJaJph"""hMp*B*CJaJphhMp*@B*CJaJphhMp*hMp*@B*CJaJphhMp*@B*CJaJphhMp*B*CJaJph"""lOmOpOqOsOtOOOOOOOOOOOOOOOOOOO̷seaSsEasEahMp*@B*CJaJphhMp*@B*CJaJphhMp*hMp*@B*CJaJphhMp*B*CJaJphhMp*@B*CJaJph)hhMp*@B*CJaJmHphsH)hhMp*@B*CJaJmHphsH)hhMp*@B*CJaJmHphsHhhMp*mHsH)hhMp*@B*CJaJmHphsH%hhMp*B*CJaJmHphsHqOOOOOOPP#P7PPo d-DM gdMp* d-DM ^gdMp* d-DM ^gdMp* d-DM gdMp* d-DM ^gdMp* d-DM ^gdMp* OOOPP P PPPPP"P#P%P&P6P7P;P*@B*CJaJphhMp*@B*CJaJphhMp*@B*CJaJphhMp*>*@B*CJaJphhMp*@B*CJaJphhMp*@B*CJaJphhMp*@B*CJaJphhMp*B*CJaJphhMp*@B*CJaJphhMp*hMp*@B*CJaJph*RRRR3SLSySSS9TPTdTyTT d-DM ^gdMp* d-DM gdMp* d-DM gdMp* d-DM ^gdMp* d-DM ^gdMp* RRRRRRRRRRRRR2S3S5S6SKSLSNSOSxSyS{S|SSSSSSSSST8T9T;T*@B*CJaJphhMp*@B*CJaJphhMp*@B*CJaJphhMp*hMp*@B*CJaJphhMp*B*CJaJphhMp*@B*CJaJph/yT{T|TTTTTTTTTUU U U@UAUCUDUUUUUUUUUUUUUUUUUVVVVөӛӍqcӛhMp*@B*CJaJphhMp*@B*CJaJphhMp*@B*CJaJphhMp*@B*CJaJphhMp*@B*CJaJphhMp*@B*CJaJphhMp*@B*CJaJphhMp*@B*CJaJphhMp*hMp*@B*CJaJphhMp*B*CJaJphhMp*@B*CJaJph&TTUAUUUUUV:V[V\VVVq d-DM ^gdMp* d-DM ^gdMp* d-DM ^gdMp* d-DM gdMp* d-DM ^gdMp* d-DM gdMp* V9V:V\?\A\B\F\G\I\J\O\P\T\U\\\\\\\\\\\\\qchMp*@B*CJaJphhMp*@B*CJaJphhMp*@B*CJaJphhMp*@B*CJaJphhMp*@B*CJaJphhMp*@B*CJaJphhMp*@B*CJaJphhMp*@B*CJaJphhMp*@B*CJaJphhMp*hMp*@B*CJaJphhMp*B*CJaJph!G\P\\\\\]]$]4]F]rZZZ d-DM ^gdMp* d-DM ^gdMp* -DM gdMp* d-DM ^gdMp* d-DM ^gdMp* d-DM gdMp* d-DM ^gdMp* \]] ] ]]]]]#]$]&]']3]4]6]7]E]F]H]I]v]w]{]|]]]]]]]]]]]]]^^өөөqchMp*@B*CJaJphhMp*@B*CJaJphhMp*@B*CJaJphhMp*@B*CJaJphhMp*@B*CJaJphhMp*@B*CJaJphhMp*@B*CJaJphhMp*@B*CJaJphhMp*B*CJaJphhMp*@B*CJaJphhMp*hMp*@B*CJaJph&F]w]]]]^^0^^^^^_6_r d-DM ^gdMp* d-DM gdMp* d-DM ^gdMp* d-DM ^gdMp* -DM gdMp* d-DM ^gdMp* ^^^^^ ^!^/^0^4^5^^^^^^^^^^^^^^^^^____5_6_:_;_g_h_j_k_өөӍqchMp*@B*CJaJphhMp*@B*CJaJphhMp*@B*CJaJphhMp*@B*CJaJphhMp*@B*CJaJphhMp*@B*CJaJphhMp*@B*CJaJphhMp*@B*CJaJphhMp*hMp*@B*CJaJphhMp*B*CJaJphhMp*@B*CJaJph&6_h_t_______`>`S``o d -DM ^gdMp* d-DM ^gdMp* d-DM gdMp* d-DM ^gdMp* d-DM ^gdMp* d-DM gdMp* k_s_t_v_w________________________`````=`>`@`A`R`S`ӷӛӍqӛӛhMp*@B*CJaJphhMp*@B*CJaJphhMp*@B*CJaJphhMp*@B*CJaJphhMp*@B*CJaJphhMp*@B*CJaJphhMp*@B*CJaJphhMp*B*CJaJphhMp*@B*CJaJphhMp*hMp*@B*CJaJph&S`U`V``````iajalamaza{a}a~aaaaaaaaaaaaaaaaa b b׷өӍqqchMp*@B*CJaJphhMp*@B*CJaJphhMp*@B*CJaJphhMp*@B*CJaJphhMp*@B*CJaJphhMp*@B*CJaJphhMp*@B*CJaJphhMp*@B*CJaJphhMp*hMp*@B*CJaJphhMp*B*CJaJphhMp*@B*CJaJph!`ja{aaaaa b$b:bLb^bsb d-DM ^gdMp* d-DM ^gdMp* d-DM ^gdMp* d-DM ^gdMp* d-DM gdMp* bbb#b$b&b'b9b:b$h% << 00P/ =!"#>$h% P0jI* 00P/ =!h"#>$h% << 00P/ =!h"Y#>$h% P0  '0P/ =!A"K#$h% <'0P/ =!;"'#$h% <'0P/ =!?"I#$h% <* 00P/ =!?"p#$h% <'0P/ =!;"#$h% <'0P/ =!?""#E$h% <'0P/ =!W"#$h% <'0P/ =!%" #9$h% <'0P/ =!",#7$h% << 00P/ =!"#7$h% P0d * 00P/ =!",#7$h% <'0P/ =!"M#%$h% <'0P/ =!M"(#$h% <'0P/ =!f"#$h% <'0P/ =!:"#$h% <'0P/ =!^"L#$h% << 00P/ =!l"X#$h% P0 * 00P/ =!^"L#$h% <'0P/ =!?"#!$h% <'0P/ =!"f#$h% <'0P/ =!+",#$h% <'0P/ =!1",##$h% <'0P/ =!",#$h% <'0P/ =!;"6#$h% <'0P/ =!c"B#@$h% <'0P/ =!D",#$h% <'0P/ =!e"j#0$h% <'0P/ =!"#$h% <'0P/ =! " #.$h% <'0P/ =!"f#$h% <'0P/ =!<"#'$h% <'0P/ =!"#$h% <'0P/ =!"%#$h% <'0P/ =!%"#0$h% <'0P/ =!3"#$h% <'0P/ =!"#3$h% <'0P/ =!"#$h% <'0P/ =!v"#$h% <-0P:p8/ =!o"?#$% <n/%*h2QF78PNG  IHDR]sRGBIDATx^wlE=q8fŬAr9dAPIb@@DAI"9Q1+*f}.~=wo}G=N]VZ{W'ϟԣZ`:9_oo=iO{'=S??W?_믿OyS {T T ̱@-:`oN;3<{/>_{'x?yPu]3]ggxs re=ZZ]aj @[PxUVЇ>tp{",7.___"ԏh&^jj-PmU ) 0 |QgBa{,^l xW|/ 7#} 0zT T toj bK 1Zwu>c9gW Lo}!NxwH q10e!vMy3oo<'Pc߯~+{-e`oVK? T6or``{ka#n#{$A=>V^Lkckc>x6 %/y-ܯMsYbtu-Ia4mҪGnze|,P1F&v6H~tΰ+Dzٛ{h1|s?\Kcz~ nIݽ^Y-P&oZ=5ne*suȌ׿nyl=@9vEY Nλ Tjñ3A}j[b ñn>FOyS~nu>Z/@@:6f 3s?> >wZoe j@:Z"2Z VYņɕ':9pUNxS((ʗ" ('*7}{>g=Y/Xb Q;T|O,P1zzg@!) C@U`3XM7[m!E%}ǡ??fL[|;**Er-W^y~#e)^z[dEjjvQ@Qm>WX3ho}K^_rp U79g*׎Kںp΄~ZtEk/g*n;9_k]K9 T9;M<l! wFwmW﷿g__?!g>^G.FėH~/]eUӞ4>zÀs>?G?VǜUnU> Tn1m hlSO?q_؛qzeD7ܟ @Y!otj;VG~G?{˦n r}ia6 nzA-P:>]֭@/}KW_} .h+_ʷ-pguڙ ,ʌ;/$\h ~A$|N1z#1ɀ:dMulVkT-л*Fn񼲤[|k_6ݒj%KF/.woh7b4kg$X u0]\uYgk0p:%N[`1n9SO MLA1S;|}c]v{-2|J_Z|~Nj $'xTr"4h0,Lց{e .(Apx +@&R|;xbߚmUo^EprҟHL㻨WmN1Mj38KsPtɴǸvҟȵ{j@Bd>VHV'eeYcTKc8d0Z*G{Ԭb9u Xt?6O3tւK!eSЦɸ[v,^%D!!"{U9幩oRQ/?˻ʚFbt#fB8ӓ5e}?XP`G.;3Ml;]nvM}v6Wħ;LkYNw~~_;7HkeqA= b`sy+Lnf?/| Ta 򗿬Ʃ5eq,#9ɭJ?O/G#gBJ:'yW]u=c8&ڠ:7l4_wuRD\IپQ]/ pGen3|_]Æ}t.bF/_=;A6p"㥖Zr>o-K۴iƂF@L{DƝH"=E=e/ {'7T<:ĞbtϦα~uWp y}}ы^4]zR8o<K3{ώ7^,w׿uXwi'6 Y+crqgsh^&`"l>_$IWz;`_@v/9R<[y:F$sd+VSY~V(p%L۴>aYV5gҁ_ +( zիV=(ؼ'T*SO=X;$h pQ{덚Fj-yaoxyۍ7xw+ęh5Sw GObɤ tŇwxdɡvSYЏ|#8Ee>Oy){Gɸ&l ]xᅽfڎ&)r6:8hgUVY)O!J@ÝiM:,9iW38K/j45\/|!eu^MlEp9UՀ/I(h/~*(7D1YC$z'> 4WF$,J3T3^zi=DӠrwzd!SۀuB+N* ý}BR|x{:Ə>M)}ְ^ S,¬v ,e[y:{,QED7{jl)zʣ[Q>!Zi ʐyaiLeWPL>CP<:[NoѰ@t|{#s9R %A2 'Cy* &zDII8]T£$]U/<:TҭHԧ>ejȓqk 9?fJݝvqjÌ^8-bt@0k^T(P؛44/E!ȋiL&SrNd*h5Xve쪬ҏeZYKظ.B*bwRI7ؗgP\dڮ"'XE+k/J!sZA?AHv[%c{2*cI_]:vd@1KA(h KV~l n H?*F1{*g|/ܹQ˴UQq}s7=hS'!Yهru]&Qəfdz@''SΟL)@m8P.2$ӁKΆJӗPi~MQ'bSJGu:_RHsOiԒ$)%0cKI!Т8;G,"&TvuL)̶4W8q y~XFooFe3Ǵ#F ܂`'t]w֑XA(L]FiEwcAY'}$ LWZi%1 dzz# pS4w؝?Ď1VjE`hS}'Ln|`}oq)J37~Z|'H |AE3%'U[HBx [&\'PI p CYwu LA( v?&\M6Y^ `(lZt:RD , @gqF`oN@@ $ρ`/a18RF􁺪[=.Hi<ϞPT h˴f>S8!f19e =Wc ;}Rlhz=?g"yZ@dD8$[=5x9#U9d~M`Մk"ӝr)0#U0~GhɿmEX>hͫV<;ypcDQ[!;-ҙG7t]ޅYOϬ1"D#:@]EƬ熆 A"%77B陡>bɦ4w`}ѻ[oYbtt@06 o_2.>;p76Hd&BI瀓Yr'8!BPۙz׻ /r'J?fɤb yN"0YLgiAS8餓Œf:M7WNxL V_ Y/⥶2U"8fԡF w翼w75JH3f]b"&4|d5)qRL 5 Fw{#/>͍Kw )Gj$3!DZJ؇ 8Y|0dLx(7El;pt:$x@D9 Nc=0hur^FϦH/P(I͕gDYgjZl80243OG Xֲf,̦ ᐤ6Kf!1qb&*6ӃJ'5s5W1)gw9>GQzғTl N%2‰%^QPT[42`=F3ur,%HY!nJƁ.F0jhGo~'Ƙ\ԧASmnbnDvw/3b?Koy8oЉwA'K̑n#B>ި$yU^ٌI4SEYaSyd֟= b2ے$ ga38Y~S# +.|S+y: w>Ҋģċ9 ު'ܧJZoX zv [$F jK]jn;i@\Ւ@ 0xѰrk>IpGD\܎ V\eO$u1*F7s T&wh==.bj%Z&hTyK4yBR~o;UDI3+- FԡE\Nÿ`CVo cT W@:6c* JA6+ buUeAfe-M9޹53֘4 9S5."!wF]MAmlI7zE#WʉfBpāၲQJ:A&,[ٔ*[DIm(ybBx1=Ez{mTK OT0IGoiu.np7n*d]U/-g@G)@ Şp۫#_,#;7=U:^e9k &]!+;"ѝ^ĻP`|>^-N }|A7 N2 2NIy_6t!N޿8ŏ',!=HBܤeiiVTf賍qI!,}]ZfjY q#R!{-h36':([2~Ї>U>cF"C3nwT+MN xj&@f]&jLĜݵ%SCo0UF >ݜT'?5!aT3EnVT0v'uAwOIÓfҺ`.GcDmzg/!0 .v'Jn ݣ P̍ ԻGRE\1#՛gj+l4[8ף[eKG tŢo=&$HçJoSX iUnƅ0 &LZ#F"IJ)撑%AV+AebLRlHt?+D3ԂM%ٵсSy"菿4c^@'mk{CCpfO9hOYOߖr[﷊\mIb R'U 4'+$ 'ŭm>6P3RZyt3.-T\-%%X/-e"q⦂>qf37_Te&3zrcA)"Kz2LF8ݣИp@Io25̊+RTܳ"հL3VmZ/6DK|E%6TCRC$8wQdc qDYy^D?SAI!wZݗlnhiXh9I7md2eflmb{$;U%&q'f8'5UZRX9X P")0W,TEiTPz#/mf3WX$ݶ5&mkLyh,ǀsVYT #Kfym˗H Սwz84!5V4C4lUjDz @F˵-(%Q&D +u*[>$SvղU&2%l^<mRD8hoAu۪Qss+fKUAT}i&  xX-1m AF,ik$ BĆ{VY(ٝ uHճk g{EG=%t0v6xˏ}c6}{e Umy(ί J&,u*i9(@Dy92 He\̀H؎:T!ܧ'=^p%ϡIIEc&]zЩ΀G%1 |i脣XdlsK2QCdʣ{٧́2PN_$i6X_/. $B:X&߫ML5KR=V`zH M3l!K ? wŧ>|;rd7a#R~nF9!dȜvp)1rdrxu8AW:G>zM7=ÂC$S9sw_{*}sjؕn`N?SAVo5).q'?P]w`ekPwD}}sf$[aKmУhXm!)#3z?y0 e#gV)]%|ȵ~6oJ#7C4GJi/êZX骸!g oNYnLwW?Xq뮻nP^_^{e&y)/*Ɵk@@;A~C9dR7M{g  lM;^u<3H^n٤amSWOeu$-tN3{(yo)Q&'Xf0:DCdZҳě+xi#Rc؃𿞱#QɛoLk$Yb>}{0TEI(?$wLtãuIg!=YjjdJ at2Gvr`7<` kH(FЀ*& /rL c,>|*US2,]Fːv{@#d4Qh~E )078@x< s{f~WxU< ,>aH0fu$6~iZMŲuvI uU>-SDuJz~螇ATTUGwt7m[h"Z.;Iym]Y,[G 3LҲ]Ō%=cfBS 2ZfEgO h#"ܘ:L?H+=+|v@m¼*FkSF)Q=yb[ӘeF[Ќ%̿$玓7MdfsT[o=$R$o+0 =?g.@;cz^0`DP N$aJWAuy4=Ar6րv_'Ax`ApG; z_PO X',$<7H$&+ \+Npr`c%`Wo]'giK2}]>ꨣC3 NanObf/L_˷G"123ՌI3G 9eߠz(co[oZ&=knS=a|7B {|X;L6q#GJL<@o)?H09GS @{cjWt\R0ڊmDr-@"@lUPy$~۵TŭЖf|RLh~aXFK/RTbNș鍾*n}sNH:翤CL ꪌLy1J?.E|'v:ǤLb枙+2B+܇gzWitg"oiB ̘*V)œgs-F0pdp ^MD,i; Q1@nѹU-Heڎy;a6`O+}嗃+BSʝrSΊzuYxl߄etqFZ1T+#NOEt΀\gwy;W[l5@@L5†Xqcö49XȜVQ33)T1Ejd Ք{(4k=A+Qn$xG?d1¤S:$Ոj)'N:$;C=')"ŀMӸ!_h5m>?ltf pΓl9E"v\K=y3qp@d^5KRqfl&%(17'f|d4Ufdi=p;P@I%0X9B$A Wn\p]sw6'NTrBB{Uq`A, x A:)ҤƌD})zr\]]2ɮ1 D.&2#]v=6uc9'p9 t#@Ζf"a)7! e $ ΀L'hxCΔB7mĵ>I2  ]\Ԉee^bF5"nӎh#9\h (Uqecf AdA3ɘ*-T(ŝqʱ7:iFt9.I>\%$sCmɱ8Tg"#|YDg[UFg zN/ꇓ+"wdXMr)-?#MC 83 >K5vf ov;WLHhۙpZ/Da76M!њZDa'L(4fXd"a(s;P1wu阙uO0/oFǟDz*PM\\u2͆!|`F[2j5&`8n`,sl儡2Ųg[$AF\X ."k 7Lt f^lG͔;yt2\2dUP!LCXns.ItsɁtںо uA&ŀN"L wP +1qa</ a+8u&|ѝ3c L0zW|Z[{Ej.iaM$fewU,t6KU8 A4D5ֳ[M ƪ|Ln%$/+lQB#lWx3 $:WW1Kdƥ髳.L^cy:@1f`H&\ӷS!"@ȕWOgJmI DB.$T;"ҡQ6 퍯͂N'9x]8Pl:As I&o5 ٢"|9BgdI|:n o$` Цr 7DekC32e HXE ‹n}3堳NN9 IEAxy[qHQ SO3 ˄<ɦ }\bbFH΋4utt,(Y\rRN@@*_]Kp<[d MND1DM$hga8WgjeꉰUj M+MhDQfaOd|nueM_`q4Q0sutA9[ {Mbꙵ-Q[ TUxS' ]Y?i4(ȴXĸSd>]sAa 8CN IDP[˨"脪*JΆW8Dn%Zd^EQRA@! 4%^Trɸ,ciOIJX HE]ǀhf1FbIʣzr$S!L*K63 Ay]3>'R9Z[čF046\j2jXl?I=,417x]K䲎AM":@}:tjc5`ȩ =c6.;; )A%qi>à~IFlburvn#W݆kPW4\۔q\  KL"%4:02S?/FL@oGp,Xa/"bhG$5 98O8_JE qɲN=SI -@s=-+f̮>pjāe~'+27UN=,ѹ.KWoL680s;V %}+ Y .hFYqE]7=$o3rݺ4u=mADF0 jțJ$5٭ns"QyK l N"[55@M%ɬȊ!gd sJAa_yơ~E O=6Bnڪ>#~r# \^hВ,ߞsp3A¢*Qt]%|ZZ7plPeM^ epPH Y8uV><:I(L^dn74iF<ka}7OL\-G|v\zNc y!>3-|n:#$Ϩyo&IXL^&WJjOhO/ɾUGj-RҍD[5dZI#QTVsiU>εmTL cX?# _g,9 aAcX@;fy{Ɵ_?|E QԄmڦ'd1OV1h2ϸ Y0:$leתyaEm!"'h "8q K$IqnQՏLi&ǺNm 4ct=yFѤZЈ ۴͞<$4 'e!9iȭ7u>1tLj枲`1<Iި=n~9|+6 մʤln_`t$,li- ȀMb4;RmyLŤK >qe4meTCN-4䎣QlILkႣΐ˫AkčrSma̠1Aa ZjkJZJ7ǝm<=<`qe3k=|b'9 ~'c&GY>KڦUv(>=׹1-,n\=-dVl](E̫tZ ,`0 TsȻ| )z.db1ö hۯ|zڙ-yXJF &a _zIx3[ziYGI<lBǯF,o[\:ȡ dcͦؾ_H=-Ɂc5 )/Ò<.ɲzʎja?A+͊[g,6jayixZ 4I3|\ Lk|.މ*7 C]d@s{ f|B50lh;0ʑ c譨1IV1 Y GoO4Wq3M(Q-lLt ׆p~̆f!dT{ uxt8Bn!FQ?/[i Fg`:)C)Cלί4\.}k!#LfqP Kq)# l`)/eWgoP&1Zcj/Z/sj)͞Cp"3ꐥAkJcrԃ7dO.|E}hVL?eG 0^7ʋyua8A[ ~ >/lnh %I)mhա1 -[L;1 ?St6ʠ=8n`Z4ިg hѮ5wC seU^|gtrL;<ۻMȋM\6is `EIR]wN3O{.l U?5Ħg'V7_Lk1zcSWeO75i$r4v8jA/yl^0atH>xrߎ1eӮ NUy@'{;iߠ$iHAW8S?1BGtDkb {yao[4V/<#A_ (~9CgyjxХ^jM_T:_!rʕ@q0/j07(Œܱ~y2%wx㍽kۑ,Ʃw;Тz 'Uo t(>YIdyjPE=UxC&x[xvPoӅ2s=^ɧZj)^D% RYD|ziGSrRND>9bBgsIZy!e_C յ͸k8 c( { Ɯs9%^g\Oqts9%ytF:B2UWtM]t<\ލ6h-APϸ 98S̳+vЛgz38y k+~qgNRASfy{챇sYgI7ל)g`޺䡥߬`c{T"zuy]5jC#ہF"z!h0:k/{2,~+A/nֲ6>). Z|#Snf(Ae`!ΜJP(> S+RDU4/{1rpHO+$Z7Y_ovgX_t;0Kk~GG0$'[rG#ĒK.(i$FADB0Pώ^Rm\چqKT(9@s@Dz3`&y9'ec % DwiCR Q1S& !UbtzI*4PhUSԃmFײKoOq *CYK UhT[Dʔ9G$Dz+W tkةVaIAb^vi=]HEHTW</xY?:#{Ҫ 'o5~/H@ Cʎ$`F^5mD@^),3ϓ dg_:#-J iY;M?zF3ތ?=fb]?qgr j'̋hC6\D_Roox0R=+KTB7kU}|]g.4=0voό$ =Mj 11&.XUFݛ}gQmef?ǣ-H V#"cLcb4qp 9XD0u]]_4YqA \Нxּ8t&N & *ꉥisT5!rXɢ$f*7nD^>Zt d "șlq3pObV f|_T'go|WV3gJWd iiqعxV6r SiRC^D QDRh0hj0S˕P]&.+LόϿ/Dp$q&O3/aLUwQ2 qgㄓCI& 2 Vgq֐c$sȼx NVmqLATN0(K@P , #w+} ܒˣײ"A9+9愡Ef,-DF\&b!|f0%=g]S !/a"4\ll1[=j>jN $k;I < JAGr)H$i߲[2x]un["R?@6 L ]ڃY'M+V ş h~b:U?Af+xd(K f?E8L%泪Kr9#A<&F- BRlG!{͚j-m,Lv !+CY*,ȴ&! "H+|h EX茮tc5فH^mItt34;/iϏB=4Sf8bJI ϖ{:<̀Йm掅vcz,CZ١i"ob$W7։8gCXE25᠄6PT: } ,!:KgKVU2(LAڨP$KZOiO3!/dKf]rW,pYMJf̎}Ъ׳L=.崚jyGO#sq6r}-$rڮaLV" 駀|KksYA$N|e˾$JR#j%ߙ%x=joTW\!juvuHJ'e߄"go۸^X-P-0{-$KO ֈ Z Z!c~,b>Mȃ .#,iS %quǙN)z4QZZ` -伤tX** J (Yڐ:[_Le Y=ݗdwҮ,egRT``{9&O|@z 6ʄE hS?5ڄ"ڋmp%W T T sщfTY~d9 g0KM뽪`?%4[m!28$&Ca7 E6֡ZZZ`@xcG< -캼2\bb@gU뽪c'IayŃ4迶EٷjMfa-E8 + \_-P-0-:Z,ZG2+ \_-P-0-0 gW T T ꒐h#k#۴FG#T T *Fl TFP-P-0=M[ZZ`,P1z>B@Zb6m}jj@h#k#۴FG#T T *Fl TFP-P-0=M[ZZ`,P1z>B@Zb6m}jj@h#k#۴FG#T T *Fl TF0v{qzYGgv饗G?@?3*Uc]ݢZ`6Y~7tӿb WN=ڲVNjvXkoW_կ~SO +nG+5YMZU-08 o'=O~/~y _75y #R*PK FQG͈'՛>9zիo~'_ſ![`D0,ρM88XoW-0 pr O}*=o?쳟/éI,0"pfhׄ9P0-)#tx׾5_?Y>^CF#;Cy{j!ޢZ`H zp<)!ìC,0"kS:묇~{+_Jf]T, NJ*Poi͑0pw͑Iiiq'|#`n_-Ls7#K/49G#:>:T{)O7<7}vy5*F"p OFDN([nַUB4l^C@r<"Uchf/70:sNKO~v4 Y%t̙3@kpD1C!Ϥңmx 1":64Pà-裏l 5C: y^:`ӷvؑN~}|+_A[zl" ^kzr?]g@`Lmg=#_%g^MW-0Lַj+;IHO|b%Xk<xH VY< %{6="} J7 Ԑd_Eevhۅ^_t֮K~Q3>2DZ֨k`3.FVj lTO.W$N}ǴNO9o|< Z`$-X^WsuI tδ/7`nrᑸl2"\.|b"NvyrRA\r ?;l'yIx>[{/FS+B-$R/}T(!(#+8- J5aB\2P="ozӛ2_|^+֚SgT-^.xW^yy|ɟg@. ~h+Az]wFM.$0fhП%/yKFpk#ǡ3#$pݶgf~GY 8? O{h}{/[jjȘvd}fuU\sk|"g#N~[ wǾKʙ S O߳D%_)X~quqDtiթf øK_RgUyRc6?u[@J)'O +;&!?%~fܯCU[ dVi %54anQD{ht1賢)k%{.*Mw r{_%fZw֡uϹi5FOnٝ@lB : 0#o(w 'bz4n+F7k/-\|{;_ -P;]0ⶫ7@\?xK]v}٧U̓mXN'tNT* hZc5l4%*Sj1)]Opp9lAUk^-0] pxEeRqCe*FOEs,L#韗_~%~|_B+ %Vi;uܽ*Fwoz@V{qg0.|#ҥ->+ X~#+F/5Vbּe{*ظ qmjY Yo,tUnƬZ-AP~(=*mwR+:ߘY>n-k^W Si+FZ[Mtnv77no jŚoYl& 7;Xbks{pA>[@/G#(4VcѳWHo~g;uri=>˸9D*F 5XC^h,Y-Zi DGnTty{;E#:Ճ>*Flq0mks%}ĸ>-1y,iY#t5٧= +FGfLt9e덃6;mL~o-W?-? oz,"{ +>ݯ[TӀr =>.rݚߟ}PV `|3 ! 7PȄKt7&gNbu'q{iVKxOw.%<P6J9kfM%ozVY$S?Ժl/rpasUMm]#X.[ Bxqřz%tdhkđ;!#jU% dq)8-7ڿruqr3HKmmKzVt@ԨY&nzylp~H6wJ.K?ey``?3S_$nb"x#쿪+F'̟nVoWRyEʬ|1a~~dhZ/y7|UWa p8w"s0BGYi3v9GoOBQ)j3H9ZbZZ%3yCYguf[5 Sy8E@#կ.~>Y|RO*駟Jxs_-%gP'n|SV0-P~= Dj;X]5O#<=|}NЯZ0[n_z;u818s@U5F. =^6 -?P#@s 7$Aжk3]Tel|ɼM:H湽K!X.R(Μ`HCtƊ. mPTmݖa$GLSd@yM7=?4zS(0nj9:=s%Ӻc=%0rxz:~:?q(*P Od-%N|> ׵ln-е|/z\sqD8R1 hW+ <\ϳ4&8#ќ 0 h$M7r Yvњ#8Ÿ|ۧ > ޥ]wݕmodlH/^]y/~>e ɟ9 *c9DSj*UݧryyXY _B`zfӖ7ܝz c` Xc =cCaRĵ=PG3dVJ qK,A~8z/.> x d'B5C?(B7b>tӰBrӦ;Dr`h6Ll4{G472cy$HnE 3FD&\%-{J<\"JO#B?u*o0thS=. :.Bq4gҺ@L3Oc+WΏ?3: 7#^;L&)U%; q|u tQNSs|١?& xOT?N4fg1>TDD0Hɫ]qMih)b#)iV'8?q$Qtg4 !PsV$wCT _Q]SD/oz,I:9.""lHg >KÍA~·..gb;6KIjbŸѿ=V\/ _V jst:MȬm: Eծ)9) Hm'Y֡0tI6IE !tН`<Lx [ KN7%30DgʅXfJd03܉g` wfO!FvgyN'^2LL%sY&l`ȡKQP(AE {l<5B*0㏗, 1DbPeč(ANw:E.hxDˤy-.b~ . kr3~5\1~((hEn;!Hrf,w[ ='wB6e *8EM}rJ,`P%~I&B@.Y9m.,ݘl2x*U7䯖wA$V,Tź_dBySPJ!I(N Da0(q#87udZa*o033VIt0#gF`0BDsOY3]>=|Zϕ:H JQāZG7Jv>Q/wB*FO';9Mke Uqg eɋ3F]"z@jB:,& 5~JBy(qv n}oLH8{yZf-:5Rc^*#8Ǟ[aNˤ?/hD#9DQ? #͗Y^z)Vg˺0R2yD%H5M09ƞ]wڰQț7pgz; )\6QHiY=(tNz-O7h~wa#<⣾rkT$qzmd~`XBX[vAj_Z5SyzsƕpX ` XwW);u}wYSeG<SZ$Q@@ ny-KnYfnȪEy␟eoDOeA*O1'Q`]*A<=(žs7f[k5T 4#0h8XL? w_ G=0|0 58Es!WoEcUd_wrIiU=kev`FOY*2(jvƠcN3qrDHRzr<*F l ڂN{z1d2|ځ,#N xV>-=}j hӨ$7?hf1A9k[hYT% # }Y?bHѳ̔\Kh;:ʏ;P?,;ਤAE1nUHb†4 FsO49sI[XꆚeU0$WubY~"HDtGOͯ &›>[4 >uoB_0G}ÈͳщzYze+;ӊ}Q.SΎ 8X8Zzz8$F&|ݗb"6@;:ZB5&rJj {.s8΃% aC%ѿs ǒbT/4Pi;bH*6 ƹVXTT+AZH"6tE(YҘ .(&6RUAG-Bv>!3pЦ좓-*FK )e%rpqvdq$mV^ n ɶ:<=.{ 2{Yo]#oLUs?ݧߒ:̌btö#z= y7gyOK CPU}֒ZYdnu-꟭2V)d}"=]0yEj3nT!*<9,5lObQsu]kow*g7؃㮨Ol[$j#-}Č.:^q lδ|*w#8bt_NczI{7߼{\!/g?kI'& e.v31A9MZh4,.fAnm}1v^ 4?} 'E y+7y˄l칲:7ȝwٍ:/tNB c'vB^$>ZYo} R T?~Jp{u+2j*M5q S¥nH%wqGK4lˤ/~|f|-:M36tЎN SIžcXg+}U pQ^{ߞO~rv?C]A`t)鬀W׵e.52CbL9-oy .Uyt_ޒfPM6d2fJobN coYgN68N/{_;ɇQ|1_|KT@[: Zsm2-pNg _Uc/| 5$w-2"hS 4),W]uս˒[oe,Tħ:8^R'"$79jF:{~|.Df B?Э2DP-<!yDs Wli4ܧfK.Y! ''=z8KwM8" .Oz׻ NK#@1C@jd|\G)/yK|򕯤?[oSx衇^W;se}Gu gJ:?֗@vaWK̾ F'\K/47p-"o|QV[mN7C&rA뿹K!᧦ewԔ(*9 i5Xa^WiV3<E>k77xE]Z^x>9J+2Z\8Ae饗@{qt#ꪫxw\x@p3-% XA)9!>#LmhO+* H0GQ }{`&iE/#w}|.5q#$1HF/P >'*Hq+z;d_7 Vz #ilڟ.D*'&Srv4Sh4 ~+Bd[bSJ hZ3IT7ַ9眯|+!xi;xS VIy-=S|:O4\^ÃTh6x,QB]PeZyٸ3baeӨ޺7qKKGE.HŦl^3SIxEI.k!,=`椱3f%~;\I49%Q,jON ΁`4M7ԏY擮e&};UΌxڕ> f8#^:Q~);M l(b]j4nqF5wJHp'gȫ)l̮evrֈB0*Dp;v 8R(u ΋-46Dk m)F4\ͧp@bhJae<=`_9/y2WIޖ[nw31q N/-uGj̞viH^C[3~#KP@{Ba# y4?Z.Ȯ :;U4c"}#?Bۃed. Z=EM} fW_}OnoN@gֲh,r`5uFG~XdvNw-A[b#/"7̴,*AZ6gzHq** 7h#jRh癄) ;plCSHNd5rD{Z˚0F [&RI7F^\MKNҜ[lEn +̰9TdSP a3gGNu0.JyE|:L]6O%߲|.p^z<8ó%ϟuY@né0JK@to4][n[v XCO,:ke/aHc$mQ3D \2=C%cH' X$fPH8$焛 <ӟ(?oiPIc]wݕNJgg.h'6+0` D`WkPUԟʛH 缦j83l_/Q1Y뗢 zHkF/%Ze/oWL1sDd2\G@=¿L ;.-=DjrFC;(x['E!y)D &Fb򧹠Q`/T=Hq]ӞXWr(ܴ@6d4^i6N5LAۨ(Q8);0'&b\!#?3eM# gh2IW[c I ~rkYyNhm楙7P? |4͠ 21Lys<VC0xda 6- |6uczδ,[oGN@"# 0\e]DT/JEauɜqDTv~<j-tD2rw%Qє2(hPX!*eP Qy$l`M8tjM74:F%_-5g%ELl1Ar5 vn\nd#_q89]J PP"agN[u7ujr3 Y-NOt^5X˿ƽ g':?(O00:.+?<H`1If[1M|ȸa]YdKN-8b6v@Ts=/P J(YnSH'ͲQOdH؍T4Xdc׊FJ,qw'c./1Ngtե4V,tn>b&xh2,chm -KXtSWH;  Kև^eݥA&?-tg}0%1#3e6! W$kJ ٴ,:ЙelLApMbAkd혈B^2a Ib Cϔt"T̚VyK^/jLK>*YTt݇w:7`Y9=$oE *d"P.Tø/guNtm>{5ԣ=H iQuw9ן8y68NDSRMG~Y{kqv؁𢳤SLlU%z]DŌat>wG,&}+țzficnn0L>e^ _\xd2 ";ġ!)Ҍ[';'y('.8 Y 4o(ʻnҽe 륦$WhќWwkV *{_bf9# &ಠ9Sq ]=r*h +aA&'jSy8@ܼw1Zg{ W?}~Xeu1o0$mZr0$Y$a|MRL"zŴJhUAj#b e8u6T fb^aY%ss @|mlxP>2xبUYQTUz.}cuưuqaC90ҰOvg^K(,)vKՓѶ4Zrpn:`$!ZIvN6 ,v螮 O?KL1 ޮ/LKnPO4^"qUL\H 5(@C]qo6hRxuONnoT"ϭ⤺܆edO}Sy}j8j$]LAh|#y[VMXW_!&a=h$HυB]zikSg"A/QTpXm˅ZPB?¥D>Wc婃 jcƃ Hl & T٤ #R㕙P 7CP8< <)7U7d @m!Al&ng!EN5RoMxN}[`UI ͎I=]|xl48p62}8.̢zuR hEPi_s!Tg2$-O1&G͚NCdXm[S:t\0n2(! R=Hyi76\,ۂ餓[6#hCWN6ѡٻ5%Ez .k3:Z1 iD# exa/j CEx{&1 [;cgBmN1AÛR-ig0vvZ~ȎmhR/'j j'*Ӭ)-1\Qճ> ي9m7FΒ*lOg` ɳ\% HؤiU+T)A3ѭ[&鈪5H3k7R\lr3 ^[ASg.bA䬜CaNgkyTK&Sʺk_jμa@gi޻3bk^gf_A;RVJO mB/14sNJ4,mV(pPM lh@V=_Em*LgʢyB30o]Տ"qȾPQ$7d{/}U2&Iq )j䎆1:'&"r/ =u?WEH0D $8|.#gVISg|)/1Q+^ V;atVxm8,Oq[d8GiփCw$ld VѢaY֞1ﴪbͺ5m `ff3^m.$NEmJ%ٞ J ctx(48# /l ׈xsMM7 2}bd>[en?dC]n&af52hضܻ"yzL7@rJNjGۺl%XAݑvD.̆VUm-du˲[)۰)ģS52m`5c GU_h#tXUhA=#^\@d*"賅 'DJfDq7b..gG4{@jsGz#g= Vz4y<4[jeI!hCP;M @t (8gf̓Ԥl _(̆M,a%¨5&Va顈C=RVmx?rP^7^~ώ0fSF .Xi)ZZDQQxNąf5E-sq*<:t ]"({&D06j ʴ'͖si׃*v?xtKJΉ6uZ ]99Aے{Ԋ|GdhNn7&1Ԟ.iv3]~X?B4$b$CjhTZX{\afMt@P'=mvm@ (8G(+ [ţU%'m5qgKM=Ъ:'W1J{ )aö(3t&u[fխoi='@BD#8uF@ySm8ivT-\y@ס,a' )m_3}gZkg6:Yג cp]dEkr d+ޙ7ڴV= / EM7苙{XKqH1;xF(bUY3hУ[c4꜒kbFfu0M1xtr 6Ej#AytEvyVGeTBuwMJeըDKt(U:Vi2?]BkH=m~FI a~rV4̂azTK,ěo9/ZuU[n95 L*K|Q%oFqm5l!oyfcih<Xg{UW}ӟvl{ %EM*@t8 'L Zb*w׾5MS[nŸFB&͎egyWu]W_xFCg@Bl3:/)LM՚!d6#8']n2uj{K]q3&!zw߽ /nTTGqgٰЏyͱL',(vnHL&4]85:g?YIt^ztUeCKKW#Ⓝ ~ _ Bm%)\ !}/FWfx 7prS,g,߮ _X-r+2,obQu~P1z8@[*?JPBA]S! w-Р[*_4Dqεa8 bq!}k;[o5)^{mop>g焸qnRonF?P ~Zk-,gBz캪I Š5<_d}"0thN%R1bz\ƝG3s{-p*3Ć:{YgWEWiCf%K祗^/~kc( N;_e4lւpNhH԰}B{S?},˔jveAh?BjkF _ʎ1F=33N6錈98V)[n*[CѲ-,IfqF<,/>FB"8|A-2.!]܋&l_z ̞;A^;r/˧r _ʏF;[ Vzޑ)@L>B0}X^zi H0o|dhkK#&pqW`EyK29GBQ.Cw}u]qιSM DbF>?'+8pU^{>yua@ȸMo{:0O\/> ^{qP>ԚLJ bdoƜ/*%2S;*3#{pП~ ~nגbsKV=B(X<R=Aa]T%uV[3kvd -$;KnQࠀ9Z-hPQDD#1NLcE1q] @knw}MA^2% kGl''4FmVVsr^R2_B 8$*J p9orFZ3t@,A5Gq Je]V nR@v*)'S@vAy@x?o&zEHFJBg3u;_T}h h3t)4~?EiJDJv&ENRC1) ISRh& = r8Nܡ_r1Cf>υ9qanJ<rty'^*48DAU#{HQ徝L+OSQ͌#s/X} @caD ٕ)݂J7 E%e%cT% S 7 M4M} YJ$5)f5tk3Sd8 t'.zB<)RFZՙ+'Čx.pqmtG^UN ɉ&β`Je.Ҥ0}Q{`-cu{3 p}{鿹`iQxB:y:j}wm7>dD!즡\8Y3MwFuÝQ FNN=NҹT5 ='|e ak&; 5Mq^~fȸu@茍!wOn=lpjΔ Մ pECd5k3GVûb}H`- @CgOL+A.P2գ F›Ī4'㘁b (֣QHpd1H?41MWWg5lFGyܔz@iܹ/ϔ%Q#.q-}.nQy5c$|18@6->;>{Ѻ!@AzSD툅!weѢ5Sagtd,:U'l*\]mH"1]A{kQjj@9Nr  pҡsg䂚<Φݮ: qL0 2@񀳒@i+"@(F /•&WsRpD=.U4ԃdt{2DI0t'#+U¤EOd6sB~lƟ%sڼjD`0'{+: +DAQy{U| bby(layڪCNOq Hsuay;^'zj̯7f,(3όΤpg3Gqʒ 峍m~qtcI5݈ԆidB2Yn,찛b?La$6TWX&dKN8!1=HvV0qꩧ$m>@lD "kR)DUOnGCıup'39Y%Ad見N]^'a~T[Puæ-Ȟ Ҳ%۪g%e5iX@ۡxR-#"mj e.o:uv䝄i{qePLӹKLˠ3S [WҘ"+_p/60t5f7M0 5!|F Ч :~?EAyN(EJ4 h^Ҍ pU`ty  e~]wմ#W8#Qts@hY"0zECg_meSV{C笸  @DW &X$rJ"T!pMjثP(C.Xl $8k27t U!'lSeZZu0԰t[)0Z8L@dn ܙ.9"lԁb͸<0.G $ l*gldAk$01!4& 6 :ǧGeݩtLuS2l WPT,g:iI*.g!F=yYI;T#$e,5<кͳ`R{eT*=Q澣f2Z8ygœ-DwTBpQ􄙭C0-0C.ZV9ނ&yů%EB;Kn-⯃}#O)=P'6.͗cYb=Qf¯4<- +Tg^QD'z<䎯}kmGv֧ /Zj/x ^{b^?hwBK*Ֆ5: =?}^<^s=5t%oȭHZ`41::OٍCN;.+|ALWo}Mc$-0Z 8h62ԧrlPcbn,N>Kxի^ť\pg>~ǟٟ51G r!G%٣:hܛ3OPYF o7 /HB -T=:}@l茽XF4hB?袋!5%YBlhHr>%{h?E} ǹW T F * y+A[b 5tn)jb*!fF ҝ?Gž-~'Qzn:|Q3 0\'7~O~dͽB[F F% 4Oh w1d:7 Qj@>-0R=cV?Ccq']v 4령 jWcV7}@-P1z£cb???Y*c`/LQ|Lð@t{-\p~<z衻O:郦qgנ|%LꫯwG/0]az V1 -P,lTh^=mm9裏=<%/y lg 0_7h#w_77ΩِiZZbtKfVT+37W_}UVy_?'tC=;v 'kwy]vُc,m(֚k%gg#rS#ƶ6 2lc36_D7t;{Մ@uAsӊLtM\sMvdhv #U on*ah"=mE'6+_JBa10j <=:J=@ͽ^ѻ/})k̿կ R .ݬy??jM/ W]uU" wAr-Pz &'h>&C%htpEA/~ 'Pxw]d1Nfgєkv.<FG]TY"l7 #ԣZZ7 TnyXKlUgg-wTU T Lez-rM. uݼzL@ݧs,%ɌI^P;+Vz@MW/?ŒeIK#t +FWG^XѣNIT cX^,Uvjjj-P1g 1?/D-f%U/4`[v؛x3,9Y.R-Џ*FczM}+r<,cw]K@~7+Eu >bk5kQchc4Cn\^"[ԋl7F?_җQcca@F9K `?#o榶԰ .1gkjV 4cqvPrc^fUX,cI^#T *FϾF/gG?яy qU6eR9FTk[/VX4Lr[q[x^%c ͻeAoH>GjZlmZA7.a+{7iBi{.-I.^.iYOZ/x<`T+We0G%J`?0:/=fMkA/PFuU25Qß'^x)"3~>([e)o2PZ 3l'>Iݩ>L}7tSovȫ]P)y1?qw{k=_)6ZqQ 㦛n,§?Ӎ.}s_꫿%/q#G|: X5=-q @_|W?`nW^m՞gV]ujrRa@'?~]b%`hK3<nP)',zgsۻ կ_}<nkUjѭk]r-?EuB -?ߎE_WPW~;-o.'֫%G_wuYgy4)o5\WR;4 ηdS,?]vWU5:j*FMU(_|饗BO4\`zW\/~1s!)k,;8_a,ݼ F1sYg~;<"M5Tpy]dY;NʇtM6&OrȔZM[pn?׶=QO{,P1z"DE6NaIJ+d^A@ >O;u(`P>䘛ਣ:s?Je[N".3Bhy6B]\V_+ 3N3x:j5Zm ftWJ {UYb4fҜK` .%D;0 ԑ%X ' $>htrjӌpJQrJhvmG&&|,ʄY:<gP] lsckr@Ptgw}T7M@e/{Y@fIkzff#Z ־,Z%L #׾vǢ2t giY 9H@_|q_Y9nrP&$70ox׈ڡF_@Xm3l-ѿyN@7 q4& h֪A {@yGРiVUVfV[h> F1Ys".%\R^OPvVԃK&נAݑ6GHx|;) cuG>K.d 68C)6L0uBHᥒ#,F>9^^%*e0W@w駟np"ӗ^zi_rTdM0´,&*vtӆ% 8JQrU(rde?O3o.lkw@Pۘ28Xc 3OpYIOFy>Y-P,P1zA z_|فHm=)ClcXIn]WX | ZAd8Sh.Q'zrhކn @(O9`"զɈnޠf;/ <~ u13/簭JHC/8/~7"~'2d450UVY% p|UM6YikD4j*FO.-P. i zl9@ A V[m#Ts4J,1< h{C8 _,6=wN;N{;>hUep#wbT7S&f"pY(`\t緿UqaFCC Q2+n`=F,jJ/+7t!رʙ,vDgAg $kL>0̄n63-BzfeA)B5E~5%VnUhP$`y@,Gh_q\#\黈%5D^٦YRN%|X Fle=EU1hf2kh:njm#09g@Ŏch<2Gc(>IRILrwK mC'tdFGML8X2Cfhֲ ϽoRBPF#GlJԂ*-pj6}#F}"X@"~9@vٚ M3HLQ_ZBj-)״wp>~j,1+ 7]"X5p>'T㠱т#XRut Ɖ=%G7idXxtLv%YNFͶНu%X_ \hl-Z˪i TneȵT IFɬM鲂ѐWj04]°wn AӶ卅*F7fQ&1.ǭŋkjpcGY.(ƒ7*8 !j3eMZ-&>[01Qj)&h[("1 cm}ҐUMNtY˪h*F7 OB -HoYtt([ݧ[f2+ ~J[;9W tcXs`(My+{^T29ykg h;f+ |:',ݪjM퓚,J7Wp%K>1y#On' ?O-Z`h)LAZqE6>&ѭӼ*}fZJV٘2ͽ7wfv㜭al@&[XIL̽ɢ- 9v(ͦ"Y}-,+V !jXަ8a7W ꓭ44* [^:xjCKm*b6r7D[U llăy(^n,>&II6ʟ7]EijYm@u&[OxKCC4|V[4GeD]죛W۴Yc 7fj/f*ҭy=fyѰuY\d*"//Uf1jYlM6^WrJhF V'˲֦c9h{*y whjV[btASs۰e tc'4wNH1I ģ X6쵵[btP$uyI[~^g;%|Ina~1<0WXZ=:lZV@k,P1ɦ9GmIHyPQ<|^nPe&wYy۷,&%f'HZK~Z@D OD,&yJ [_׵\¼k0:5nZ-P1{[M}f^D?}mkh3v͗)>EHo[tB.}SG=Z`[btmAxk-`x}ٯ#Fk_=.3G~Qs$% 0wGuLNkjY-@&I=Z~qJo!dm_E]PQmCW\3(%Bւyw092Q:dO yiH?~žɘn8ӤKղ5IO訣ꪫ6lKN&zQ+>'K} ͊`/NI)X 7!,#yU4 'X;GVOi0J(]D|@UrO~r饗PSw(mqkY-@ ~^zJ+{XvBgqI'Ybs%Ixa?[ 'Xޥu)ʧ99ڰuْ0W(y6!YLJƒ4'{{Z+]vZ\[-P1 -O}jM6e]ɠ<چߩ[T(Q?/OUC ptDOluvId ( 1`}ˏJ~4PK,_\ (_qrkn'I;W_]A7㲵YbbtHaU̸ufX`8AI`eCkwRN/;CNtvmwi'i xcB}Id\=j#</? +صr#2Orl^ (v_ Q-^d*}׼5b}$QX{8Oww K_Au{ kp:cM}K+Ag-2ax]yL[ahz.Q:/M3 T2%P{Ex_|q1ema#%Zbs8^RrIunM&Y"8^;wcMS˯BQ#n LA-ss&^fRKfiq-W!at8]!pxn5րT =\j dt4e&Ok{/BW\qŷmD)RZ>3{bdc'=zE(9L/zئƘj@[0:o&37O!/ٔI48?J¯}R<4mA6H =̼(S+|TLԃu j0H ZndNiR vuWZ>zBiKpFDdUVQV[PA:[zj{xZE4 b2~*;`nΜcfcDP~5\wu iK@IsS¦$9!]߱c@V9GDgz/\6*N~;߹KF㭓h=y2f%Xoܒ@sJEdzקuhFfndIZxq a2lfņni0@SBTm }O  La# MA}]J3~PxwD{F9EnF /YE>ԍe2P>#Woley$ͺP+\-r S)tveHtc BCF?Hщ>?a/ 𝊍DS!`u*0` vm@Ё>uyW( Q_gL̿#F;Xg380Ag` 'A0 ľMм+;9$z´=0FH ^!*`4`[r%3ߒ%Xt!RO*>GPZ*FgHN>dm&4#PK1C/I*\;RguԃMOƀ@/~{߳+cvqI(L\cG6A $ lŇOu}b_'sGI(YYLo1u40$2/bBdaҨ.%jSi#U T Ch /|s)P#Ef1Q 7*Ydʯm0>TF$?AiWłN+r 0qcC( I(mنOh#F/29isR> Nlb!=eQqX작5)5*X|X` Mbz{ \H A<'0YL`%!&f У+sQ4wO]tv }P0|B@G~$A Z}un*gO~t֚7ͳR\V=%Ş7JL}*(7"j1:ث@P}>t8ӄP]VKK~HQL RWCHw2+ LpyLg 8PX$A S^ (KQmq#v% :33uQ3`ߞ{; LX`ÕQ}L%5רkB" \#JV#yfdŭZ5};4 WɌLFtVۏ}cf hA Q"VevjQy4&8 cEqhH' 1Y|B;Az1 JTML8}BdcOMuJ$SzRݎ>b{キ4jJsMsҞjYmb4|g37i5bH"H aZN}6tf12BāDUd&K^Dn,WJVF*)oi!0]1{3f ~'" 3|G!PB?q@ }IȗAT:fE/F Uݙ?|4Ct.,Αaؑb:,6b,iiϑߡ< dzX֤Z`l-0(+ya1aA@`^$]p'r,#%YnriO$8.L0LI[` +8ƄTP2=/L0}5 ;%xf1$Cl]ԞfԞ՚T T h8=,ֆz܃~)W];i%r}CꧬmNH"qȶ]>B=ZZ7 Ë!1THВpɶn` ^oUx!#V]t1 *_ˬhJrv9_i(!$6[Bl$ZG[9شk F1F)ߨX-P-6 Uϼ1%0hW3uP!sԳT##a>f`4,S]@a{q*{qg慊ym{,Ret qֽN=mjGfTp/ʜ=lԊ"5]eS\͊:^| tQ=kYȢ!@`Guެ>la1A j *Y^;xc:ZAN@$S{4pmup>(E7|ꁺ0:}) \$THUZ=D+gF N3 74=:pŌkGtN7w|HhuW^)(v=P Fr 3.-|M6@=dp!ĪչR#fB_vܔdĶFߦLx<-[dQͷoFZL2x`c@^fhAc͈<zɸ],;U ƎݻwgyR_feXU DGC#50zܹ4d?if8sQx{@;1RHMѣo޼Q4ZRoAu'}0mgϞ=xǏ)o GN… H4_~U1Q.14&ou֪E8ϟ߷o߉'4CJl3fܸq#Gf1Mky|zb?MucϟߺuԩSIUԩSׯ_>~ҥK4pD ӱZȬ]E:._|ه̩2d߿'RGIfbO:eI 7n\3qĕ+W6L`^:- @0ؙF6| hӧt;Cɓ'+C}L{A(aِ6u{;~AFw4Q;3>0hZ9͛7a$r*}G1Ar[_l٤Ibe Z}99V+ٳMf^@%s99Ik sW%L:t9s,Nhճna9xx!h+ Bih ^A+@{A#X6$pO?y[TwBFj \֣#3gBjTQ[ׯ_'9%POhn2\kךE'vFvA'{n5,Xnt}\XŌ⁶xe")+O>ݽ{wy͘1#J(~BXZUxbI-1Ç53ci0l.tG Y@/܀\.TŽy3/`{ `4S3d j&|;;wR06w$)AAKXOo˕Q)(hZh+W1@o߾+V@~ԫoEU8FLQfTتԌ^B:@)mjlMUç4'1lESy)\cO Щ6l=va į$=-ETx/&u̮%nބ+P9sF""kͰ 7DƝՖȼ) 1Nx=$#Vo18'©1__"~VodQUT =u/b7H0INњ˘ߜZhYKTTHu"4 > ӧO+0`͛V}Wp-2:@~`Tn:tT$j~/>T|׮]ܣSƍ\$RV3eD$-wA6! KFl3DΥo*t-`4rիR*IV^Ѫ`]Q`0&K?(>_G CX B,q Йp 9q_#fu7՝: vFG@Zb-/wn``:= uS.*(]˗/#gD=O# Lþm۶%MuV 0V5sFH9f*Hڻ=uMFާ?L$`tzYނ\ޡS%|'L$ R%%hܹN2{Y4-`r]ern ڰaˆVE$ҕlȾV5 a:M-Z$  K`L… )l0 \FpXc-El Cu"-ɩٝAOxYd;7 (f4-` UqؼyWDMzkThs銖Q!d( H(& U!f<ڮ :o o%Z:mn߾4zQMA/>Wғ֖s|ChT3ݔ8ըCR!/ h,),bgU^᧩GuIDATͨB;[ݱ/Ѱ ՃN^M@5`ߚ5k Y^3fѩm2}c)IL=_Z2x@@=U s@IENDB`L@L Mp*Normal$CJOJQJ^J_HaJmH sH tH DA@D Default Paragraph FontViV  Table Normal :V 44 la (k(No List 4 @4 Mp*Footer  !.)@. Mp* Page Number5  Q#+458=Y>FP WWaPkt|S~~OՈ?)|B<YǿE/e  m(0b8 G\NT[`l/lXlllll%mcmmmmnHntnnnnoAooooo&pOpxppp q4q]qqqqr*rSr|rrrr sIsrsssst 4h s^D|QT# 1 _  , = n 4 5  * 4 = I U 7 B N Z f n .Pt"9EKr=M^EawCP{3U+P  n|Ux3G\r4aR f { !!/!8!B!N!!!!!!!+"Q"""#P#Q##### $$g$~$$$$$3%P%_%o%%%%&Q&&&&;'P'k'}''''(.(?(Z(m(():)s)))*3*R***++d++++ ,h,,,-$-N-y-------..6.B.M.T.r./#0R0w000Z111$2Z2233 333@3P3Q3R333333 4 44b4444&5]5n555556(6F6W6g6|666667#737F7u777777"888888949t9999~:::: ;0;<<<=)=C=D=E=V=========>+>C>X>Y>?? @I@b@x@8AVAmAAAA/B?BVBBBBCCC,C9C2DKDeDvDDDD E&E2E?EvEEEEEF/FFFFFFFFGGG HLHaHII J:JJJ KK(K9KHKiKKLLLLMcNiNoNvNNNNNO O=OUOOOOOPPPP|QQ RRS?S\S{SSSSTT'T2T:TFTTTTTTT UUU)U8UAUCUUUUUUU5VTVxVVV W WDWVWkWWWWWWWW XX0XWXmXX Y2YWYYYYZ Z3ZVZmZ.[[E\\]l]]]]] ^ ^a^o^z^^^^?`|`````abbb&c9cOcccc,d]ddefv^vsvvvKwvwwww;xxy'yBycyyyyyyzzz {"{;{S{r{s{t{{|6|W||||}}.}>}Q}a}}}}}}~~R~S~V~Z~e~r~~~~~~~~~~D_Ā*NOƁ$ʂ=zYj+BZk~ZZzȇއTczԈՈ݉ Ċ DTz/\̌%?Vi,Jm͏bĐ>?~‘ב7`BT?,Oߗ9jƙҙ"Oߛ()ǜAzڝ,K؞Ddğޟ0D"ikv)CRȤݤdvΥ.If&yȧ =:G\s*={|ت$ZʫAB̬,;<íխ&$g1CUkȰڰ<YKeɳ.F`~ 1DXYAKM",6@| ![qƿǿ$+29@HqlPfw,pyKXDE!(/^djqx:DMV`jN9"jzWBqB`"z1p3Ll"o>JaqT?FZk|)YnE\)Yn./i.HLT]>U'4@Xt%Ig 9Z^p&_ L1Xa&deN[fxBJS\eo )6PKz/Xq$[ $6Sj(=Jar  .      C P ^ u       ' 4 C      # s       4    8@IH4K$#yT\e[l5L]n W1Ahpy"5q>Yh4W} ! X m       !!:!!!!!!!7"J"e"""""#-#A#### $!$5$$$$$$v&&& 'U'l'''(2(Q(l(m((6)t)) *L***J++,,~,,,-(-R-c-------.6.t......K/S/\////000v0~00001111111102K2a2n22223:3]33344%464J4[444445)5T5`5m5z555@8F8L8S8Z8a8b8 ;";W;r;;;<<=)=H=d=?#?o???2@CC2CGCCDDDDDEEFXSXXjY{YYYYY Z$Z:ZLZ^ZsZ"[h[[[[[[j\\\\\\\f]g]]]^T^^^^^^^__$_%____`@`c`d`e`n`o`p`|`}`~``00000000000000000000000000000000000000000000000000000000000000000000000000000000000000000000000000000000000000000000000000000000000000000000000000000000000000000000000000000000000000000000000000000000000000000000000000000000000000000000000000000000000000000000000000000000000000000000000000000000000000z!0z!0z!0z!0z!0z!0z!0z!0z!0z!0 z!0 z!0 z!0 z!0 00000000000000000000000000000000000{!0{!{!0{!0{!000000000000000000000000000000000000000000000000000000000000000000000000000000000000000000000000000000000000000000000000000000000000000000000000000000000000000000000000000000000000000000000000000000000000000000000000000000000000000000000000000000000000000000000000000000000000000000000000000000000000000000000000000000000000000000000000000000000000000000|!0|!0|!0|!0|!000000000000000000000000000000000000000000000000000000000000000000000000000000000000000000000000000000000000000000000000000}!0}!0}!0}!0}!00000000000000000000000000000000000000000000000000000000000000000000000000000000000000000000000000000000000000000~!0~!0~!0~!0~!0~!000000000000000000000000000000000000000000000000000000000000000000000000000000000000000000000000000000000000000000000000000000000000000000000000000000000000000000000000000000000000000000000000000000000000000000000000000000000000000000000000000!0!0!0!000000000000000000000000000000000000000000000000000000000000000000000000000000000000000000000000000000000000000000000000000000000000000000000000000000000000!0!0!0!0!0!0!0!0!00000000000000000000000000000 0 0 0 0 0 0 0 0 0 0 0 0@0&00000000000000000000000000000000000000000000000000000 000 000 @0&0x000000000000000000000000000000000000000 00 00 00 00 00 00 0@0&0x00000000000000000000000000000000000000 00 0 00(0 00 0080 00 0@0&00 0 0 0 0 0 000 0 0 00 0 000 0 0 00 0 00 0 0 0 0 0000 000 0000000000000000000000(@0&0x0(0(0(0(0(0(0(00(0(00(0(000000000000(0(0(0(0(0000000(0(0(000(000(000(00000 000(@0&0x0000000000000000000000000000000000000000000000000000 000 0 00(0 0 0 0 000 0 0 0 @0&008080808080800080808008080008080800@0@00@0@0@0@0@0000@000@00000@0&0x0@0@0@0@0@0@0@00H0H000H0H0H000000000000H0H0H0000000000000 0 0 00 0 00 0 0000 0 0 0 @0&0x0H0H0H0H0H0H00H0H00P0P00P0P00000P000P0P0P00000000000 00 00 00 00 00 0P0 0 080 0 0 @0&00P0P0P0P0P0P0000P0P00P0P000P0P0P00P0P00P0P0P0P0P0000X000X0000000000000000000000X000X000X@0&00X0X0X0X0X0X000X0X0X00X0X0X0X0X00X0X0X0X00X0X0X0`0000`@0@0@0@0@0@00`  4h s^D|QT# 1 _  , = n 4 5  * 4 = I U 7 B N Z f n .Pt"9EKr=M^EawCP{3U+P  n|Ux3G\r4aR f { !!/!8!B!N!!!!!!!+"Q"""#P#Q##### $$g$~$$$$$3%P%_%o%%%%&Q&&&&;'P'k'}''''(.(?(Z(m(():)s)))*3*R***++d++++ ,h,,,-$-N-y-------..6.B.M.T.r./#0R0w000Z111$2Z2233 333@3P3333334b4444&5]5n555556(6F6W6g6|666667#737F778888949t9999~:::: ;0;<<<=)=C=V=========>+>C>X>Y>?? @I@b@x@8AVAmAAAA/B?BVBBBBCCC,C9C2DKDeDvDDDD E&E2E?EvEEEEEF/FFFFFFFFGGG HLHaHII J:JJJ KK(K9KHKiKKLLLLMcNiNoNvNNNNNO O=OUOOOOOPPPP|QQ RRS?S\S{SSSSTT'T2T:TFTTTTTTT UUU)U8UAUCUUUUUUU5VTVxVVV W WDWVWkWWWWWWWW XX0XWXmXX Y2YWYYYYZ Z3ZVZmZ.[[E\\]l]]]]] ^ ^a^o^z^^^^?`|`````abbb&c9cOcccc,d]ddefv^vsvvvKwvwwww;xxy'yBycyyyyyyzzz {"{;{S{r{{|6|W||||}}.}>}Q}a}}}}}}~~R~S~V~Z~e~r~~~~~~~~D_Ā*NOƁ$ʂ=zYj+BZk~ZZzȇއTczԈՈ݉ Ċ DTz/\̌%?Vi,Jm͏bĐ>?~‘ב7`BT?,Oߗ9jƙҙ"Oߛ()ǜAzڝ,K؞Ddğޟ0D"ikv)CRȤݤdvΥ.If&yȧ =:G\s*={|ت$ZʫAB̬,;<íխ&$g1CUkȰڰ<YKeɳ.F`~ 1DXYAKM",6@| ![qƿǿ$+29@HqlPfw,pyKXDE!(/^djqx:DMV`jN9"jzWBqB`"z1p3Ll"o>JaqT?FZk|)YnE\)Yn./i.HLT]>U'4@Xt%Ig 9Z^p&_ L1Xa&deN[fxBJS\eo )6PKz/Xq$[ $6Sj(=Jar  .      C P ^ u       ' 4 C      # s       4    8@IH4K$#yT\e[l5L]n W1Ahpy"5q>Yh4W} ! X m       !!:!!!!!!!7"J"e"""""#-#A#### $!$5$$$$$$v&&& 'U'l'''(2(Q(l(m((6)t)) *L***J++,,~,,,-(-R-c-------.6.t......K/S/\////000v0~00001111111102K2a2n22223:3]33344%464J4[444445)5T5`5m5z555@8F8L8S8Z8a8b8 ;";W;r;;;<<=)=H=d=?#?o???2@CC2CGCCDDDDDEEFXSXXjY{YYYYY Z$Z:ZLZ^ZsZ"[h[[[[[[j\\\\\\\f]g]]c`d``@0@0@0@0@0@0@0@0@0@0@0@0@0@0@0@0@0@0@0@0@0@0@0@0@0@0@0@0@0@0@0@0@0@0@0@0@0@0@0@0@0@0@0@0@0@0@0@0@0@0@0@0@0@0@0@0@0@0@0@0@0@0@0@0@0@0@0@0@0@0@0@0@0@0@0@0@0@0@0@0@0@0@0@0@0@0@0@0@0@0@0@0@0@0@0@0@0@0@0@0@0@0@0@0@0@0@0@0@0@0@0@0@0@0@0@0@0@0@0@0@0@0@0@0@0@0@0@0@0@0@0@0@0@0@0@0@0@0@0@0@0@0@0@0@0@0@0@0@0@0@0@0@0@0@0@0@0@0@0@0@0@0@0@0@0@0@0@0@0@0@0@0@0@0@0@0@0@0@0@0@0@0@0@0@0@0@0@0@0@0@0@0@0@0@0@0@0@0@0@0@0@0@0@0@0@0@0@0@0@0@0@0@0@0@0@0@0@0@0@0@0@0@0@0@0@0@0@0@0@0@0@0@0@0@0@0@0@0@0@0@0@0@0@0@0@0@0@0@0@0@0@0@0@0@0@0@0@0@0@0@0@0@0@0@0@0@0@0@0@0@0@0@0@0@0@0@0@0@0@0@0@0@0@0@0@0@0@0@0@0@0@0@0@0@0@0@0@0@z!0@z!0@z!0@z!0@z!0@z!0@z!0@z!0@z!0@z!0 @z!0 @z!0 @z!0 @z!0 @0@0@0@0@0@0@0@0@0@0@0@0@0@0@0@0@0@0@0@0@0@0@0@0@0@0@0@{!0@{!0@{!0@{!0@{!0@0@0@0@0@0@0@0@0@0@0@0@0@0@0@0@0@0@0@0@0@0@0@0@0@0@0@0@0@0@0@0@0@0@0@0@0@0@0@0@0@0@0@0@0@0@0@0@0@0@0@0@0@0@0@0@0@0@0@0@0@0@0@0@0@0@0@0@0@0@0@0@0@0@0@0@0@0@0@0@0@0@0@0@0@0@0@0@0@0@0@0@0@0@0@0@0@0@0@0@0@0@0@0@0@0@0@0@0@0@0@0@0@0@0@0@0@0@0@0@0@0@0@0@0@0@0@0@0@0@0@0@0@0@0@0@0@0@0@0@0@0@0@0@0@0@0@0@0@0@0@0@0@0@0@0@0@0@0@0@0@0@0@0@0@0@0@0@0@0@0@0@0@0@0@0@0@0@0@0@0@0@0@0@0@0@0@0@0@0@0@0@0@0@0@0@0@0@0@0@0@0@0@0@0@0@0@0@0@0@0@0@0@0@0@0@0@0@0@0@0@0@0@0@0@0@0@0@0@0@0@0@0@0@0@0@0@0@0@0@0@0@0@0@0@0@0@0@0@0@0@0@0@0@0@0@0@0@0@0@0@0@0@0@0@0@0@0@0@0@0@0@0@0@0@0@0@0@0@0@0@0@0@0@0@0@0@0@0@0@0@0@0@0@0@0@0@0@0@0@0@0@0@0@0@0@0@0@0@0@0@0@0@0@0@0@0@0@0@0@0@0@0@0@0@0@0@0@0@0@0@0@0@0@0@0@0@0@0@0@0@0@0@0@0@0@0@0@0@0@0@0@0@|!0@|!0@|!0@|!0@|!0@0@0@0@0@0@0@0@0@0@0@0@0@0@0@0@0@0@0@0@0@0@0@0@0@0@0@0@0@0@0@0@0@0@0@0@0@0@0@0@0@0@0@0@0@0@0@0@0@0@0@0@0@0@0@0@0@0@0@0@0@0@0@0@0@0@0@0@0@0@0@0@0@0@0@0@0@0@0@0@0@0@0@0@0@0@0@0@0@0@0@0@0@0@0@0@0@0@0@0@0@0@0@0@0@0@0@0@0@0@0@0@0@0@0@0@0@0@0@0@0@}!0@}!0@}!0@}!0@}!0@0@0@0@0@0@0@0@0@0@0@0@0@0@0@0@0@0@0@0@0@0@0@0@0@0@0@0@0@0@0@0@0@0@0@0@0@0@0@0@0@0@0@0@0@0@0@0@0@0@0@0@0@0@0@0@0@0@0@0@0@0@0@0@0@0@0@0@0@0@0@0@0@0@0@0@0@0@0@0@0@0@0@0@0@0@0@0@0@0@0@0@0@0@0@0@0@0@0@0@0@0@0@0@0@0@0@0@0@0@0@0@0@~!0@~!0@~!0@~!0@~!0@~!0@0@0@0@0@0@0@0@0@0@0@0@0@0@0@0@0@0@0@0@0@0@0@0@0@0@0@0@0@0@0@0@0@0@0@0@0@0@0@0@0@0@0@0@0@0@0@0@0@0@0@0@0@0@0@0@0@0@0@0@0@0@0@0@0@0@0@0@0@0@0@0@0@0@0@0@0@0@0@0@0@0@0@0@0@0@0@0@0@0@0@0@0@0@0@0@0@0@0@0@0@0@0@0@0@0@0@0@0@0@0@0@0@0@0@0@0@0@0@0@0@0@0@0@0@0@0@0@0@0@0@0@0@0@0@0@0@0@0@0@0@0@0@0@0@0@0@0@0@0@0@0@0@0@0@0@0@0@0@0@0@0@0@0@0@0@0@0@0@0@0@0@0@0@0@0@0@0@0@0@0@0@0@0@0@0@0@0@0@0@0@0@0@0@0@0@0@0@0@0@0@0@0@0@0@0@0@0@0@0@0@0@0@0@0@0@0@0@0@0@0@0@0@0@0@0@0@0@0@0@0@0@0@0@0@0@0@0@0@0@0@0@0@!0@!0@!0@!0@0@0@0@0@0@0@0@0@0@0@0@0@0@0@0@0@0@0@0@0@0@0@0@0@0@0@0@0@0@0@0@0@0@0@0@0@0@0@0@0@0@0@0@0@0@0@0@0@0@0@0@0@0@0@0@0@0@0@0@0@0@0@0@0@0@0@0@0@0@0@0@0@0@0@0@0@0@0@0@0@0@0@0@0@0@0@0@0@0@0@0@0@0@0@0@0@0@0@0@0@0@0@0@0@0@0@0@0@0@0@0@0@0@0@0@0@0@0@0@0@0@0@0@0@0@0@0@0@0@0@0@0@0@0@0@0@0@0@0@0@0@0@0@0@0@0@0@0@0@0@0@0@0@0@0@!0@!0@!0@!0@!0@!0@!0@!0@!0@0@0@0@0@0@0@0@0@0@0@0@0@0@0@0@0@0@0@0@0@0@0@0@0@0@0@0@0@0@0@0@0@0@0@0@0@0@0@0@0@0@0@0@0@0@0@0@0@0@0@0@0@0@0@0@0@0@0@0@0@0@0@0@0@0@0@0@0@0@0@0@0@0@0@0@0@0@0@0@0@0@0@0@0@0@0@0@0@0@0@0@0@0@0@0@0@0@0@0@0@0@0@0@0@0@0@0@0@0@0@0@0@0@0@0@0@0@0@0@0@0@0@0@0@0@0@0@0@0@0@0@0@0@0@0@0@0@0@0@0@0@0@0@0@0@0@0@0@0@0@0@0@0@0@0@0@0@0@0@0@0@0@0@0@0@0@0@0@0@0@0@0@0@0@0@0@0@0@0@0@0@0@0@0@0@0@0@0@0@0@0@0@0@0@0@0@0@0@0@0@0@0@0@0@0@0@0@0@0@0@0@0@0@0@0@0@0@0@0@0@0@0@0@0@0@0@0@0@0@0@0@0@0@0@0@0@0@0@0@0@0@0@0@0@0@0@0@0@0@0@0@0@0@0@0@0@0@0@0@0@0@0@0@0@0@0@0@0@0@0@0@0@0@0@0@0@0@0@0@0@0@0@0@0@0@0@0@0@0@0@0@0@0@0@0@0@0@0@0@0@0@0@0@0@0@0@0@0@0@0@0@0@0@0@0@0@0@0@0@0@0@0@0@0@0@0@0@0@0@0@0@0@0@0@0@0@0@0@0@0@0@0@0@0@0@0@0@0@0@0@0@0@0@0@0@0@0@0@0@0@0@0@0@0@0@0@0@0@0@0@0@0@0@0@0@0@0@0@0@0@0@0@0@0@0@0@0@0@0@0@0@0@0@0@0@0@0@0@0@0@0@0@0@0@0@0@0@0@0@0@0@0@0@0@0@0@0@0@0@0@0@0@0@0@0@0@0@0@0@0@0@0@0@0@0@0@0@0@0@0@0@0@0@0@0@0@0@0@0@0@0@0@0@0@0@0@0@0@0@0@0@0@0@0@0@0@0@0@0@0@0@0@0@0@0@0@0@0@0@0@0@0@0@0@0@0@0@0@0@0@0@0@0@0@0@0@0@0@0@0@0@0@0@0@0@0@0@0@0@0@0@0\>0v7\>0v70Rh   n57fR@ "#$&')R)+*+b-}/u1355:; >? ABEFF HmIJ4L)M.NNQiSrVVWY[I\]G]]_`UbeAh)kn\pqsst]vw{|]~߁ 1@nyՐD/iOޣܥڦv@ƵD" s8eX+mGlB"aEn;X7^iEo    RM#n !"#h&'m(>)*,q/50J35689:5<c=R@CFKLMlOOPRyTVWXYZ[\^k_S` b^bdh   !#%&(*+-/13479:<>?BCEGIJLNPQSUVXZ\]_abdfgikmoqsuwy{}~   "$%')+-.013D trP "|$G&'B)Q+,.Z0R2h457;<=F?tADEGI2LMNRTWX[ ]]D_ `aefjlprsuwz|}'"OʊjczVbOڥħk.&&UF[Hj?YX  q (#\ !q#h&X()*,02c56780:J<m="C#GLqOPRTVrXY[G\F]6_`sbdgh   "$'),.02568;=@ADFHKMORTWY[^`cehjlnprtvxz|   !#&(*,/245h !!/Xb$/%*h2QF78Etb$-dfpE  )@.(    c A?Î!aZ8Z88t8t`T`T`T`T8`T8`TZ8`TZ8`TMM#" B S  ?Z~` :T@<?v  -AOesfq@LP]X`mvUXZf #NW(2* q{6=JQ_f? I U \ i p ~ !!.!E!M!!!!!!!!!!!!!!!!!!!# #F#O#U#]###$$$$$$%%6%?%|%%S']'u'|'''))))z****/+:+g+v+++L,Y,[,_,,,>-M---W.`.:1?1A1G1M2Y222)32363?3J4W4555566(666W6`6p6x6|66666666666666666667$7,7?7E7777777C8K8b8i8r8w8x88888 9999 9!939496979s9999999@:F:H:K::;;;;;;;< < <<<<<<<<3=;========>>>>*>.>7>???@ @ @ @H@I@K@L@a@b@d@e@w@YAcAAAHBUBCC?DJDNDXDYDdDDDDDDDDDE EEEBEKELEPEzEEEEEEEEE F FFF&F'F.FFFFFFFFFFFFFAGOGHHoIyIrJ{JJJJK+K8KNNNNNNNNOOO*O_G______```````````UbWbobqbbbbccc{I{J{R{V{c{|&|B|L|||}}"}-}1}=}T}`}}}}}}}}~~~~~:>ր0;w.8]dejns}IOblˇՇWbfy}DH֊'2H(5BKOU`hsyzƓ7;KZҕmqCMɗӗ"ISԘۘݘLOBO5B\iћޛjyݝ/59CaeYfT[ɣӣ,45BFQU\^b1;=HLVXewɪتìϬڬ۬/67:FLhjoq|~׮#09]ft} &04BFTXcϰҰ&2CNñͱ#do hxcpڴ 4:߷ɹҹ_l^gt}~lo5>?E :<=CDFGLMOPU\_fixzTXZ] !%8?CS*5fhpx@K[kAIMWY_t@N |wAL*3CLMW[jksw 6;LU <Hjox KMkx) >I:=?DQZ{]kBDEIJLMRSUV[\^_d,5",1;Zc   /9;@Xdqz|  1;#'5w&@I}   # r ~             % - S ] x           & * 3 7 B           c q v          ,6FRo~8@O\|05akko %4KW-7KNp   !&!1!!!!!0#7#$$'''''''''''((#(T(]((())$)))+++,, ,,#,$,+,s,|,,,,--'-+-A-H-P-l.p.000011F2J22222330454`4j444444444445566 66i6m6V7]77788999999{:~:::::@;L;;;;;<<<=:=G=X=c==>>>DDDDDDDDDDDDFFjGlGmGpGqGsGtGGGGGGGGGGH"HI#IJJKKKKKKKKLLLLMMUM]MQRRRRRRRRRRRRRRRSS>SHS`ShSlStSSSTTUUwWWWWXXXXFYLYmYzY~YYYYZZ'Z-Z/Z8Z=ZEZFZKZOZWZXZ]ZZZZZ [[[[;[E[Z[`[|[[[[[[]]]]]^J^R^^^^^^^^__"____`3`?`L`T`e`~``  , - = > = @   * + 4 5 = > I J 7 8 B C N O Z [ f g rsX`V\ 34+,qsno|}UVxy45ab!!!!!!!!!!+","Q"R"""""##3%4%P%Q%_%`%o%p%%%%%&&Q&R&&&&&;'<'P'Q'k'l'}'~'''(((()):);)s)t)))**3*4*R*S*****d+e+++++++ ,!,,,,,--$-%-N-O---T.U.//#0$0R0S0w0x000Z1[11111$2%2Z2[23343@3A33333333333b4c4444444&5'537;7F7L7u7|777777788B893949s9==C>D>?@ @H@I@a@b@w@ BB/B0B?B@BVBWBBBBBCCCCC C,C-C EE&E'E2E3EEEEEFFFFFFFFFFFFGGGGGG H HLHMHIIII J J:J;JJJ K KKK(K)K9K:KHKIKoNpNNNNOPPTTTT'T(T2T3T:T;TTTTTTTTTTT8U9UUU5V6VTVUVxVyVVVVVDWEWVWWWkWlWWWWWWW X X0X1XWXXXZZ3Z4ZVZWZ'[-[]]]]]] ^ ^a^b^o^p^z^{^^^^^_```Rbubbcccdd=h>hJhKh\h]hmhnhvhwhhhhhi i5i6iJiKijj1j2jFjGj^j_jvjwjjj k kkk#k$k?k@kkkkkkkkkkk!l"l/l0l9l:lJlKl_l`lllllmm=m>mRmSmmmnnBnCnZn[nnnnnnnoooo/o0oNqfqsssssssssstttttttttt:u;uXuYukuluuuuuvv>v?v^v_vsvtvvvKwLwvwwwwwwwwwxxyy'y(yByCycydyyyyyyyzzzz {{S{T{{{| |6|7|W|X|||>}?}Q}R}}}}}}}}}}}~~~~DEĀŀ*+Ɓǁʂ˂=>z{YZjk+,BC~Z[Z[z{ȇɇއ߇TUcdz{]ĊŊ%)DETUz{/0\]ݎގ,-ҙәBP.7:;խ֭hj`aAB "*,46> $)+0279>@FpqPQ:CDLMUVW`a01@Axy5=%5?KBIJRS[\def6Hz{IR'  c r  ioKN45WX$$$$d%g%&&&&)',''''(h-k-6.7.......001111111111 66_7a777@8D8F8J8L8Q8S8X8Z8_8::3A>A2C3CGCCDDDDDDDDEE&F+FOFTFFFjGpGqGGGGoHwHJJkPpPrPwPyP~PPPPPPPPPPP4U5U5V7V;W>WWW ^^%____e`~``33333333333333333333333333333333333333333333333333333333333333333333333333333333333333333333333333333333333333333333333333333333333333333333333333333333333333333333333333333333333333333333333333333333333333333333333333333333333333333333333333333333333333333333333333333333333333333333333333333333333333333333333333333333333333333333333333333333333333333333333333333333333333333333333333333333333333333333333333333333333333333333333333333333333333333333333333333333333333333333333333333333333333333333333333333333333333333333333333333333333555F7==S~~|)[T^^@`d`e`~``d``^Rw+gd҆52&kzntꦤx[,o~2lSZL( vH}hڥf1 y/8q)|lpGPR<}rkP+aHT#0q %6 f6.\>&>Bnvwj ?ʾePR-^ɂhNj M ^9O D"v QF "~e$ 0 ot7 ؎+*! \1"n. : ' p2:  kDJ pO:e  Gh@aIk +BPA6twk6&T Ÿ<vfQz%F-30|Gzn>SCBB+,tkZv5f.Yu^J!<S7V}"Xa@ٞWYDfQ[)X4Eg- ΎoE6Xh\ *W{2{hI*OrNF.X{wpmJ1mDՌ_Pr:dhz~R\XE2 &>"ҹ S.*"<3>ʟj" e0| IGěHULy4W:xp!^ %,'[` YF DjH B@  :<M $K\o!L`YTb"JY#^l#v+#N$($$9xa)$>S6$Rn$<F$.$EN\$n0y>o$}%dgq7%~``&%%Bvul%^EQ%B/&&%*[V&l0m&V:1J&꣒)G'Boq'#rj(F'T(,V*@ * Hz<*ұRBd<+~+?+q-oD,2|)r-஼..-'oza-fdm.p./*o\.rtKp2.$UI.L3Vf.̱MF4/d\e0J&A%0Kh0j~0 c0ּ@O^0(HP1$&A:[1v 1HpX1Mk&1rɌm5180S^ 2@Cl2ÖJ1-2>jN3Z3PA_`4Z4p[q5,y|5?96V 7?6: J7 aIW7q<0 t7:% Jv9 h9(<09<9ZS9Dt:9D4%:7?: [>:u[;e`; I4@>bLxV?;&}O?\rm?`?v_?>o;AbqAAzZP_MB:; eB-3B]\eDB*W1B0nT_C(|TDz>Dd\eD*WD-b QDT,6$E}HfEMh >F8Ԙ6lsF0.R`F bfFpy+GpORGR|#Ggb6_tH}x H#SGaIv]IEI:L1JuML-`LޘƈLrMj(~M|P$}M*J,pN HzRNp"rI]OykalL0|a`EB^ ajea@$XbzQZbRJb8bbLcsL sUc$LchQ c%rd be6$)eW[DeoRL g$,@g<j"gbTg:Cg:*QsZ hL`YLUhz if& i຿IiYbGi&Z+^vjtܛ@k| Okd [clRI_lehXn~Ran Tn`I(oX:i1o* Aeob$N,Ho?ojXV@Hp\mS(qmJ|'qtVnlr006enrg \|sxcsKNd*s*$/sD4%NOtO&tN&LuJYnu஼v hv.u<0wJYIbwDem wtgxj >y,P:ZQy6?]Qy 8~B!>yvd,QUz,ҳzW zL:&F{0"H{zpC|.:%_|0f|,86n9|(0<|T?/}\\4}f,:}Z69Vs}?}Ib*~dEF7T~z~,86g~:}"|~:6y^V>oؾ]>6zJQg*h^`OJQJo(hHvh^`OJQJ^Jo(hHohpp^p`OJQJo(hHh@ @ ^@ `OJQJo(hHh^`OJQJ^Jo(hHoh^`OJQJo(hHh^`OJQJo(hHh^`OJQJ^Jo(hHohPP^P`OJQJo(hHh^`OJQJo(hHvh^`OJQJ^Jo(hHohpp^p`OJQJo(hHh@ @ ^@ `OJQJo(hHh^`OJQJ^Jo(hHoh^`OJQJo(hHh^`OJQJo(hHh^`OJQJ^Jo(hHohPP^P`OJQJo(hHhpp^p`OJQJo(hHoh@ @ ^@ `OJQJ^Jo(hHoh^`OJQJo(hHh^`OJQJo(hHh^`OJQJ^Jo(hHoh^`OJQJo(hHhPP^P`OJQJo(hHh  ^ `OJQJ^Jo(hHoh^`OJQJo(hH@OJQJ^Jo(.88^8`o() ^`hH.  L ^ `LhH.   ^ `hH. xx^x`hH. HLH^H`LhH. ^`hH. ^`hH. L^`LhH.@OJQJ^Jo(.h^`OJQJo(hHvh^`OJQJ^Jo(hHohpp^p`OJQJo(hHh@ @ ^@ `OJQJo(hHh^`OJQJ^Jo(hHoh^`OJQJo(hHh^`OJQJo(hHh^`OJQJ^Jo(hHohPP^P`OJQJo(hH ^`OJQJo("  ^`OJQJo("  pp^p`OJQJo("  @ @ ^@ `OJQJo("  ^`OJQJo("  ^`OJQJo("  ^`OJQJo("  ^`OJQJo("  PP^P`OJQJo(" hhh^h`OJQJo(hHoh88^8`OJQJ^Jo(hHoh^`OJQJo(hHh  ^ `OJQJo(hHh  ^ `OJQJ^Jo(hHohxx^x`OJQJo(hHhHH^H`OJQJo(hHh^`OJQJ^Jo(hHoh^`OJQJo(hH88^8`o(.^`o()  ^ `o()  ^ `.xx^x`.HLH^H`L.^`.^`.L^`L.@qOJQJ^Jo(.h^`OJQJo(hHh^`OJQJ^Jo(hHohpp^p`OJQJo(hHh@ @ ^@ `OJQJo(hHh^`OJQJ^Jo(hHoh^`OJQJo(hHh^`OJQJo(hHh^`OJQJ^Jo(hHohPP^P`OJQJo(hH @OJQJ^Jo(.h^`OJQJo(hHh^`OJQJ^Jo(hHohpp^p`OJQJo(hHh@ @ ^@ `OJQJo(hHh^`OJQJ^Jo(hHoh^`OJQJo(hHh^`OJQJo(hHh^`OJQJ^Jo(hHohPP^P`OJQJo(hHh^`OJQJo(hHh^`OJQJ^Jo(hHohpp^p`OJQJo(hHh@ @ ^@ `OJQJo(hHh^`OJQJ^Jo(hHoh^`OJQJo(hHh^`OJQJo(hHh^`OJQJ^Jo(hHohPP^P`OJQJo(hHh^`OJQJo(hHohpp^p`OJQJ^Jo(hHoh@ @ ^@ `OJQJo(hHh^`OJQJo(hHh^`OJQJ^Jo(hHoh^`OJQJo(hHh^`OJQJo(hHhPP^P`OJQJ^Jo(hHoh  ^ `OJQJo(hH88^8`o(.^`o(.  L ^ `LhH.   ^ `hH. xx^x`hH. HLH^H`LhH. ^`hH. ^`hH. L^`LhH.@qOJQJ^Jo(.h^`OJQJo(hHh^`OJQJ^Jo(hHohpp^p`OJQJo(hHh@ @ ^@ `OJQJo(hHh^`OJQJ^Jo(hHoh^`OJQJo(hHh^`OJQJo(hHh^`OJQJ^Jo(hHohPP^P`OJQJo(hHh^`OJQJo(hHh^`OJQJ^Jo(hHohpp^p`OJQJo(hHh@ @ ^@ `OJQJo(hHh^`OJQJ^Jo(hHoh^`OJQJo(hHh^`OJQJo(hHh^`OJQJ^Jo(hHohPP^P`OJQJo(hH.8@ @ ^@ `OJPJQJ^Jo(-8@ @ ^@ `OJQJ^Jo(hHo8^`OJQJo(hH8^`OJQJo(hH8^`OJQJ^Jo(hHo8^`OJQJo(hH8PP^P`OJQJo(hH8  ^ `OJQJ^Jo(hHo8^`OJQJo(hHh^`OJQJo(hHvh^`OJQJ^Jo(hHohpp^p`OJQJo(hHh@ @ ^@ `OJQJo(hHh^`OJQJ^Jo(hHoh^`OJQJo(hHh^`OJQJo(hHh^`OJQJ^Jo(hHohPP^P`OJQJo(hH^`OJQJo(hH^`OJQJ^Jo(hHopp^p`OJQJo(hH@ @ ^@ `OJQJo(hH^`OJQJ^Jo(hHo^`OJQJo(hH^`OJQJo(hH^`OJQJ^Jo(hHoPP^P`OJQJo(hH@OJQJ^Jo(.@OJQJ^Jo(.h^`B*OJQJo(phhHh^`OJQJ^Jo(hHohpp^p`OJQJo(hHh@ @ ^@ `OJQJo(hHh^`OJQJ^Jo(hHoh^`OJQJo(hHh^`OJQJo(hHh^`OJQJ^Jo(hHohPP^P`OJQJo(hHh^`OJQJo(hHh^`OJQJ^Jo(hHohpp^p`OJQJo(hHh@ @ ^@ `OJQJo(hHh^`OJQJ^Jo(hHoh^`OJQJo(hHh^`OJQJo(hHh^`OJQJ^Jo(hHohPP^P`OJQJo(hH`@OJQJ^Jo(. 88^8`56o(. ^`hH.  L ^ `LhH.   ^ `hH. xx^x`hH. HLH^H`LhH. ^`hH. ^`hH. L^`LhH.h^`OJQJo(hHh^`OJQJ^Jo(hHohpp^p`OJQJo(hHh@ @ ^@ `OJQJo(hHh^`OJQJ^Jo(hHoh^`OJQJo(hHh^`OJQJo(hHh^`OJQJ^Jo(hHohPP^P`OJQJo(hHh^`OJQJo(hHvh^`OJQJ^Jo(hHohpp^p`OJQJo(hHh@ @ ^@ `OJQJo(hHh^`OJQJ^Jo(hHoh^`OJQJo(hHh^`OJQJo(hHh^`OJQJ^Jo(hHohPP^P`OJQJo(hH88^8`o()h^`OJQJo(hHh^`OJQJ^Jo(hHohpp^p`OJQJo(hHh@ @ ^@ `OJQJo(hHh^`OJQJ^Jo(hHoh^`OJQJo(hHh^`OJQJo(hHh^`OJQJ^Jo(hHohPP^P`OJQJo(hHh^`OJQJo(hHvh^`OJQJ^Jo(hHohpp^p`OJQJo(hHh@ @ ^@ `OJQJo(hHh^`OJQJ^Jo(hHoh^`OJQJo(hHh^`OJQJo(hHh^`OJQJ^Jo(hHohPP^P`OJQJo(hH88^8`o(. ^`hH.  L ^ `LhH.   ^ `hH. xx^x`hH. HLH^H`LhH. ^`hH. ^`hH. L^`LhH.h^`OJQJo(hHvh^`OJQJo(hHoh| | ^| `OJQJo(hHhLL^L`OJQJo(hHh^`OJQJ^Jo(hHoh^`OJQJo(hHh^`OJQJo(hHh^`OJQJ^Jo(hHoh\\^\`OJQJo(hHh^`OJQJo(hHh^`OJQJ^Jo(hHohpp^p`OJQJo(hHh@ @ ^@ `OJQJo(hHh^`OJQJ^Jo(hHoh^`OJQJo(hHh^`OJQJo(hHh^`OJQJ^Jo(hHohPP^P`OJQJo(hH88^8`o(.   ^ `hH.0^`0o(. xx^x`hH. HH^H`hH. L^`LhH. ^`hH. ^`hH. L^`LhH.@OJQJ^Jo(.h^`OJQJo(hHh^`OJQJ^Jo(hHohpp^p`OJQJo(hHh@ @ ^@ `OJQJo(hHh^`OJQJ^Jo(hHoh^`OJQJo(hHh^`OJQJo(hHh^`OJQJ^Jo(hHohPP^P`OJQJo(hH^`o(.   ^ `hH.  L ^ `LhH. xx^x`hH. HH^H`hH. L^`LhH. ^`hH. ^`hH. L^`LhH.2@OJQJ^Jo(. hh^h`o(.88^8`OJPJQJ^J.h^`OJQJo(hHh^`OJQJ^Jo(hHohpp^p`OJQJo(hHh@ @ ^@ `OJQJo(hHh^`OJQJ^Jo(hHoh^`OJQJo(hHh^`OJQJo(hHh^`OJQJ^Jo(hHohPP^P`OJQJo(hHh^`OJQJo(hHvh^`OJQJ^Jo(hHohpp^p`OJQJo(hHh@ @ ^@ `OJQJo(hHh^`OJQJ^Jo(hHoh^`OJQJo(hHh^`OJQJo(hHh^`OJQJ^Jo(hHohPP^P`OJQJo(hHh^`OJQJo(hHohpp^p`OJQJ^Jo(hHoh@ @ ^@ `OJQJo(hHh^`OJQJo(hHh^`OJQJ^Jo(hHoh^`OJQJo(hHh^`OJQJo(hHhPP^P`OJQJ^Jo(hHoh  ^ `OJQJo(hH88^8`OJPJQJ^J.hhh^h`.h 88^8`hH.h L^`LhH.h   ^ `hH.h   ^ `hH.h xLx^x`LhH.h HH^H`hH.h ^`hH.h L^`LhH.88^8`o()h^`OJQJo(hHvh^`OJQJ^Jo(hHohpp^p`OJQJo(hHh@ @ ^@ `OJQJo(hHh^`OJQJ^Jo(hHoh^`OJQJo(hHh^`OJQJo(hHh^`OJQJ^Jo(hHohPP^P`OJQJo(hH(@^OJQJ^Jo(.@OJQJ^Jo(.hhh^h`OJQJo(hHh88^8`OJQJ^Jo(hHoh^`OJQJo(hHh  ^ `OJQJo(hHh  ^ `OJQJ^Jo(hHohxx^x`OJQJo(hHhHH^H`OJQJo(hHh^`OJQJ^Jo(hHoh^`OJQJo(hH88^8`o(. ^`hH.  L ^ `LhH.   ^ `hH. xx^x`hH. HLH^H`LhH. ^`hH. ^`hH. L^`LhH.@OJQJ^Jo(.h^`OJQJo(hHh^`OJQJ^Jo(hHohpp^p`OJQJo(hHh@ @ ^@ `OJQJo(hHh^`OJQJ^Jo(hHoh^`OJQJo(hHh^`OJQJo(hHh^`OJQJ^Jo(hHohPP^P`OJQJo(hHh^`OJQJo(hHvh^`OJQJ^Jo(hHohpp^p`OJQJo(hHh@ @ ^@ `OJQJo(hHh^`OJQJ^Jo(hHoh^`OJQJo(hHh^`OJQJo(hHh^`OJQJ^Jo(hHohPP^P`OJQJo(hHh^`OJQJo(hHh^`OJQJ^Jo(hHohpp^p`OJQJo(hHh@ @ ^@ `OJQJo(hHh^`OJQJ^Jo(hHoh^`OJQJo(hHh^`OJQJo(hHh^`OJQJ^Jo(hHohPP^P`OJQJo(hHh^`OJQJo(hHh^`OJQJ^Jo(hHohpp^p`OJQJo(hHh@ @ ^@ `OJQJo(hHh^`OJQJ^Jo(hHoh^`OJQJo(hHh^`OJQJo(hHh^`OJQJ^Jo(hHohPP^P`OJQJo(hHh^`OJQJo(hHvh^`OJQJ^Jo(hHohpp^p`OJQJo(hHh@ @ ^@ `OJQJo(hHh^`OJQJ^Jo(hHoh^`OJQJo(hHh^`OJQJo(hHh^`OJQJ^Jo(hHohPP^P`OJQJo(hH88^8`o()h^`OJQJo(hHvh^`OJQJ^Jo(hHohpp^p`OJQJo(hHh@ @ ^@ `OJQJo(hHh^`OJQJ^Jo(hHoh^`OJQJo(hHh^`OJQJo(hHh^`OJQJ^Jo(hHohPP^P`OJQJo(hHh^`OJQJo(hHh^`OJQJ^Jo(hHohpp^p`OJQJo(hHh@ @ ^@ `OJQJo(hHh^`OJQJ^Jo(hHoh^`OJQJo(hHh^`OJQJo(hHh^`OJQJ^Jo(hHohPP^P`OJQJo(hHh^`OJQJo(hHh^`OJQJ^Jo(hHohpp^p`OJQJo(hHh@ @ ^@ `OJQJo(hHh^`OJQJ^Jo(hHoh^`OJQJo(hHh^`OJQJo(hHh^`OJQJ^Jo(hHohPP^P`OJQJo(hHh^`OJQJo(hHh^`OJQJ^Jo(hHohpp^p`OJQJo(hHh@ @ ^@ `OJQJo(hHh^`OJQJ^Jo(hHoh^`OJQJo(hHh^`OJQJo(hHh^`OJQJ^Jo(hHohPP^P`OJQJo(hH@OJQJ^Jo(.88^8`o()h^`OJQJo(hHvh^`OJQJ^Jo(hHohpp^p`OJQJo(hHh@ @ ^@ `OJQJo(hHh^`OJQJ^Jo(hHoh^`OJQJo(hHh^`OJQJo(hHh^`OJQJ^Jo(hHohPP^P`OJQJo(hH @OJQJ^Jo(.h^`OJQJo(hHvh^`OJQJ^Jo(hHohpp^p`OJQJo(hHh@ @ ^@ `OJQJo(hHh^`OJQJ^Jo(hHoh^`OJQJo(hHh^`OJQJo(hHh^`OJQJ^Jo(hHohPP^P`OJQJo(hH@OJQJ^Jo(.hhh^h`OJQJo(hHoh88^8`OJQJ^Jo(hHoh^`OJQJo(hHh  ^ `OJQJo(hHh  ^ `OJQJ^Jo(hHohxx^x`OJQJo(hHhHH^H`OJQJo(hHh^`OJQJ^Jo(hHoh^`OJQJo(hH88^8`OJPJQJ^J.h^`OJQJo(hHh^`OJQJ^Jo(hHohpp^p`OJQJo(hHh@ @ ^@ `OJQJo(hHh^`OJQJ^Jo(hHoh^`OJQJo(hHh^`OJQJo(hHh^`OJQJ^Jo(hHohPP^P`OJQJo(hHh^`OJQJo(hHh^`OJQJ^Jo(hHohpp^p`OJQJo(hHh@ @ ^@ `OJQJo(hHh^`OJQJ^Jo(hHoh^`OJQJo(hHh^`OJQJo(hHh^`OJQJ^Jo(hHohPP^P`OJQJo(hH@OJQJ^Jo(.88^8`OJPJQJ^J.h^`OJQJo(hH.8^`OJPJQJ^Jo(hH-hpp^p`OJQJo(hH@ @ ^@ `OJQJo(hH^`OJQJ^Jo(hHo^`OJQJo(hH^`OJQJo(hH^`OJQJ^Jo(hHoPP^P`OJQJo(hHh^`OJQJo(hHh^`OJQJ^Jo(hHohpp^p`OJQJo(hHh@ @ ^@ `OJQJo(hHh^`OJQJ^Jo(hHoh^`OJQJo(hHh^`OJQJo(hHh^`OJQJ^Jo(hHohPP^P`OJQJo(hHh^`.h ^`hH.h pLp^p`LhH.h @ @ ^@ `hH.h ^`hH.h L^`LhH.h ^`hH.h ^`hH.h PLP^P`LhH.(@^OJQJ^Jo(.88^8`o()@OJQJ^Jo(.h^`OJQJo(hHvh^`OJQJ^Jo(hHohpp^p`OJQJo(hHh@ @ ^@ `OJQJo(hHh^`OJQJ^Jo(hHoh^`OJQJo(hHh^`OJQJo(hHh^`OJQJ^Jo(hHohPP^P`OJQJo(hH hh^h`o(.@OJQJ^Jo(.hh^h`o(.h^`OJQJo(hHvh^`OJQJ^Jo(hHoh^`OJQJo(hHhs s ^s `OJQJo(hHhCC^C`OJQJ^Jo(hHoh^`OJQJo(hHh^`OJQJo(hHh^`OJQJ^Jo(hHoh^`OJQJo(hH@OJQJ^Jo(.h^`OJQJo(hHvh^`OJQJ^Jo(hHohpp^p`OJQJo(hHh@ @ ^@ `OJQJo(hHh^`OJQJ^Jo(hHoh^`OJQJo(hHh^`OJQJo(hHh^`OJQJ^Jo(hHohPP^P`OJQJo(hHh^`OJQJo(hHh^`OJQJ^Jo(hHohpp^p`OJQJo(hHh@ @ ^@ `OJQJo(hHh^`OJQJ^Jo(hHoh^`OJQJo(hHh^`OJQJo(hHh^`OJQJ^Jo(hHohPP^P`OJQJo(hHh^`OJQJo(hHvh^`OJQJ^Jo(hHohpp^p`OJQJo(hHh@ @ ^@ `OJQJo(hHh^`OJQJ^Jo(hHoh^`OJQJo(hHh^`OJQJo(hHh^`OJQJ^Jo(hHohPP^P`OJQJo(hH@@OJQJ^Jo(.@OJQJ^Jo(.88^8`o(. ^`hH.  L ^ `LhH.   ^ `hH. xx^x`hH. HLH^H`LhH. ^`hH. ^`hH. L^`LhH. 9 ^ `9o(M 9M ^M `9o(-9^`9o(-.Y9Y^Y`9o(-.. 9^`9o( -... e9e^e`9o( -.... 9^`9o( -..... q9q^q`9o(-...... 9^`9o(-.......h^`OJQJo(hHvh^`OJQJ^Jo(hHohpp^p`OJQJo(hHh@ @ ^@ `OJQJo(hHh^`OJQJ^Jo(hHoh^`OJQJo(hHh^`OJQJo(hHh^`OJQJ^Jo(hHohPP^P`OJQJo(hHh^`OJQJo(hHh^`OJQJ^Jo(hHohpp^p`OJQJo(hHh@ @ ^@ `OJQJo(hHh^`OJQJ^Jo(hHoh^`OJQJo(hHh^`OJQJo(hHh^`OJQJ^Jo(hHohPP^P`OJQJo(hHh^`OJQJo(hHoh^`OJQJ^Jo(hHohpp^p`OJQJo(hHh@ @ ^@ `OJQJo(hHh^`OJQJ^Jo(hHoh^`OJQJo(hHh^`OJQJo(hHh^`OJQJ^Jo(hHohPP^P`OJQJo(hHC@OJQJ^Jo(.h^`OJQJo(hHh^`OJQJ^Jo(hHohpp^p`OJQJo(hHh@ @ ^@ `OJQJo(hHh^`OJQJ^Jo(hHoh^`OJQJo(hHh^`OJQJo(hHh^`OJQJ^Jo(hHohPP^P`OJQJo(hHh88^8`OJQJo(hHvh^`OJQJ^Jo(hHoh  ^ `OJQJo(hHh  ^ `OJQJo(hHhxx^x`OJQJ^Jo(hHohHH^H`OJQJo(hHh^`OJQJo(hHh^`OJQJ^Jo(hHoh^`OJQJo(hHh^`OJQJo(hHh^`OJQJ^Jo(hHohpp^p`OJQJo(hHh@ @ ^@ `OJQJo(hHh^`OJQJ^Jo(hHoh^`OJQJo(hHh^`OJQJo(hHh^`OJQJ^Jo(hHohPP^P`OJQJo(hHh^`OJQJ^Jo(hHoh^`OJQJ^Jo(hHohpp^p`OJQJo(hHh@ @ ^@ `OJQJo(hHh^`OJQJ^Jo(hHoh^`OJQJo(hHh^`OJQJo(hHh^`OJQJ^Jo(hHohPP^P`OJQJo(hHh^`OJQJo(hHh^`OJQJ^Jo(hHohpp^p`OJQJo(hHh@ @ ^@ `OJQJo(hHh^`OJQJ^Jo(hHoh^`OJQJo(hHh^`OJQJo(hHh^`OJQJ^Jo(hHohPP^P`OJQJo(hH88^8`o(.^`o(.  L ^ `LhH.   ^ `hH. xx^x`hH. HLH^H`LhH. ^`hH. ^`hH. L^`LhH.88^8`o(.h88^8`OJQJo(hHoh^`OJQJ^Jo(hHoh  ^ `OJQJo(hHh  ^ `OJQJo(hHhxx^x`OJQJ^Jo(hHohHH^H`OJQJo(hHh^`OJQJo(hHh^`OJQJ^Jo(hHoh^`OJQJo(hHhhh^h`OJQJo(hHh88^8`OJQJ^Jo(hHoh^`OJQJo(hHh  ^ `OJQJo(hHh  ^ `OJQJ^Jo(hHohxx^x`OJQJo(hHhHH^H`OJQJo(hHh^`OJQJ^Jo(hHoh^`OJQJo(hH^`o(.^`.hLh^h`L.88^8`.^`. L ^ `L.  ^ `.xx^x`.HLH^H`L.^`5o(.   ^ `hH.  L ^ `LhH. xx^x`hH. HH^H`hH. L^`LhH. ^`hH. ^`hH. L^`LhH.^`OJPJQJ^J. pp^p`hH. @ L@ ^@ `LhH. ^`hH. ^`hH. L^`LhH. ^`hH. PP^P`hH.  L ^ `LhH.hhh^h`OJQJo(hHoh88^8`OJQJ^Jo(hHoh^`OJQJo(hHh  ^ `OJQJo(hHh  ^ `OJQJ^Jo(hHohxx^x`OJQJo(hHhHH^H`OJQJo(hHh^`OJQJ^Jo(hHoh^`OJQJo(hHh^`OJQJo(hHoh^`OJQJ^Jo(hHohpp^p`OJQJo(hHh@ @ ^@ `OJQJo(hHh^`OJQJ^Jo(hHoh^`OJQJo(hHh^`OJQJo(hHh^`OJQJ^Jo(hHohPP^P`OJQJo(hHh^`OJQJo(hHvh^`OJQJ^Jo(hHohpp^p`OJQJo(hHh@ @ ^@ `OJQJo(hHh^`OJQJ^Jo(hHoh^`OJQJo(hHh^`OJQJo(hHh^`OJQJ^Jo(hHohPP^P`OJQJo(hHh^`OJQJo(hHh^`OJQJ^Jo(hHohpp^p`OJQJo(hHh@ @ ^@ `OJQJo(hHh^`OJQJ^Jo(hHoh^`OJQJo(hHh^`OJQJo(hHh^`OJQJ^Jo(hHohPP^P`OJQJo(hHh@ @ ^@ `OJQJo(hHoh^`OJQJ^Jo(hHoh^`OJQJo(hHh^`OJQJo(hHh^`OJQJ^Jo(hHohPP^P`OJQJo(hHh  ^ `OJQJo(hHh^`OJQJ^Jo(hHoh!!^!`OJQJo(hHhh^h`o(.h88^8`OJQJo(hHh^`OJQJ^Jo(hHoh  ^ `OJQJo(hHh  ^ `OJQJo(hHhxx^x`OJQJ^Jo(hHohHH^H`OJQJo(hHh^`OJQJo(hHh^`OJQJ^Jo(hHoh^`OJQJo(hH@OJQJ^Jo(. ^`OJQJo("  ^`OJQJo("  pp^p`OJQJo("  @ @ ^@ `OJQJo("  ^`OJQJo("  ^`OJQJo("  ^`OJQJo("  ^`OJQJo("  PP^P`OJQJo(" h^`OJQJo(hHvh^`OJQJ^Jo(hHohpp^p`OJQJo(hHh@ @ ^@ `OJQJo(hHh^`OJQJ^Jo(hHoh^`OJQJo(hHh^`OJQJo(hHh^`OJQJ^Jo(hHohPP^P`OJQJo(hHh^`OJQJo(hHvh^`OJQJ^Jo(hHohpp^p`OJQJo(hHh@ @ ^@ `OJQJo(hHh^`OJQJ^Jo(hHoh^`OJQJo(hHh^`OJQJo(hHh^`OJQJ^Jo(hHohPP^P`OJQJo(hH @hOJQJ^Jo(.h^`OJQJo(hHvh ^`o(hH.hpp^p`OJQJo(hHh@ @ ^@ `OJQJo(hHh^`OJQJ^Jo(hHoh^`OJQJo(hHh^`OJQJo(hHh^`OJQJ^Jo(hHohPP^P`OJQJo(hH^`o(.   ^ `hH.  L ^ `LhH. xx^x`hH. HH^H`hH. L^`LhH. ^`hH. ^`hH. L^`LhH.h^`OJQJo(hHh^`OJQJ^Jo(hHohpp^p`OJQJo(hHh@ @ ^@ `OJQJo(hHh^`OJQJ^Jo(hHoh^`OJQJo(hHh^`OJQJo(hHh^`OJQJ^Jo(hHohPP^P`OJQJo(hHh^`OJQJo(hHvh^`OJQJ^Jo(hHohpp^p`OJQJo(hHh@ @ ^@ `OJQJo(hHh^`OJQJ^Jo(hHoh^`OJQJo(hHh^`OJQJo(hHh^`OJQJ^Jo(hHohPP^P`OJQJo(hHh^`OJQJ^Jo(hHoh^`OJQJ^Jo(hHohpp^p`OJQJo(hHh@ @ ^@ `OJQJo(hHh^`OJQJ^Jo(hHoh^`OJQJo(hHh^`OJQJo(hHh^`OJQJ^Jo(hHohPP^P`OJQJo(hHh^`OJQJo(hHvh^`OJQJ^Jo(hHohpp^p`OJQJo(hHh@ @ ^@ `OJQJo(hHh^`OJQJ^Jo(hHoh^`OJQJo(hHh^`OJQJo(hHh^`OJQJ^Jo(hHohPP^P`OJQJo(hH ^`OJQJo("  ^`OJQJo("  pp^p`OJQJo("  @ @ ^@ `OJQJo("  ^`OJQJo("  ^`OJQJo("  ^`OJQJo("  ^`OJQJo("  PP^P`OJQJo(" h^`OJQJ^Jo(hHoh^`OJQJ^Jo(hHohpp^p`OJQJo(hHh@ @ ^@ `OJQJo(hHh^`OJQJ^Jo(hHoh^`OJQJo(hHh^`OJQJo(hHh^`OJQJ^Jo(hHohPP^P`OJQJo(hH88^8`o(. ^`hH.  L ^ `LhH.   ^ `hH. xx^x`hH. HLH^H`LhH. ^`hH. ^`hH. L^`LhH.@OJQJ^Jo(.hpp^p`OJQJo(hHoh@ @ ^@ `OJQJ^Jo(hHoh^`OJQJo(hHh^`OJQJo(hHh^`OJQJ^Jo(hHoh^`OJQJo(hHhPP^P`OJQJo(hHh  ^ `OJQJ^Jo(hHoh^`OJQJo(hHh88^8`OJQJo(hHoh^`OJQJ^Jo(hHoh  ^ `OJQJo(hHh  ^ `OJQJo(hHhxx^x`OJQJ^Jo(hHohHH^H`OJQJo(hHh^`OJQJo(hHh^`OJQJ^Jo(hHoh^`OJQJo(hHh^`.h^`.hpLp^p`L.h@ @ ^@ `.h^`.hL^`L.h^`.h^`.hPLP^P`L.@lOJQJ^Jo(.h^`OJQJo(hHh^`OJQJ^Jo(hHohpp^p`OJQJo(hHh@ @ ^@ `OJQJo(hHh^`OJQJ^Jo(hHoh^`OJQJo(hHh^`OJQJo(hHh^`OJQJ^Jo(hHohPP^P`OJQJo(hHh^`OJQJo(hHh^`OJQJ^Jo(hHohpp^p`OJQJo(hHh@ @ ^@ `OJQJo(hHh^`OJQJ^Jo(hHoh^`OJQJo(hHh^`OJQJo(hHh^`OJQJ^Jo(hHohPP^P`OJQJo(hHh^`OJQJo(hHvh^`OJQJ^Jo(hHohpp^p`OJQJo(hHh@ @ ^@ `OJQJo(hHh^`OJQJ^Jo(hHoh^`OJQJo(hHh^`OJQJo(hHh^`OJQJ^Jo(hHohPP^P`OJQJo(hH@OJQJ^Jo(.h^`OJQJo(hHh^`OJQJ^Jo(hHohpp^p`OJQJo(hHh@ @ ^@ `OJQJo(hHh^`OJQJ^Jo(hHoh^`OJQJo(hHh^`OJQJo(hHh^`OJQJ^Jo(hHohPP^P`OJQJo(hHh^`OJQJo(hHh^`OJQJ^Jo(hHohpp^p`OJQJo(hHh@ @ ^@ `OJQJo(hHh^`OJQJ^Jo(hHoh^`OJQJo(hHh^`OJQJo(hHh^`OJQJ^Jo(hHohPP^P`OJQJo(hHh88^8`OJQJo(hHoh^`OJQJ^Jo(hHoh  ^ `OJQJo(hHh  ^ `OJQJo(hHohxx^x`OJQJ^Jo(hHohHH^H`OJQJo(hHh^`OJQJo(hHh^`OJQJ^Jo(hHoh^`OJQJo(hHh^`OJQJo(hHh^`OJQJ^Jo(hHohpp^p`OJQJo(hHh@ @ ^@ `OJQJo(hHh^`OJQJo(hHh^`OJQJo(hHh^`OJQJo(hHh^`OJQJ^Jo(hHohPP^P`OJQJo(hH@OJQJ^Jo(.h^`OJQJo(hHh^`OJQJ^Jo(hHohpp^p`OJQJo(hHh@ @ ^@ `OJQJo(hHh^`OJQJ^Jo(hHoh^`OJQJo(hHh^`OJQJo(hHh^`OJQJ^Jo(hHohPP^P`OJQJo(hHh^`OJQJo(hHh^`OJQJ^Jo(hHohpp^p`OJQJo(hHh@ @ ^@ `OJQJo(hHh^`OJQJ^Jo(hHoh^`OJQJo(hHh^`OJQJo(hHh^`OJQJ^Jo(hHohPP^P`OJQJo(hHh^`OJQJo(hHvh^`OJQJ^Jo(hHohpp^p`OJQJo(hHh@ @ ^@ `OJQJo(hHh^`OJQJ^Jo(hHoh^`OJQJo(hHh^`OJQJo(hHh^`OJQJ^Jo(hHohPP^P`OJQJo(hHhpp^p`OJQJo(hHoh@ @ ^@ `OJQJ^Jo(hHoh^`OJQJo(hHh^`OJQJo(hHoh^`OJQJ^Jo(hHoh^`OJQJo(hHhPP^P`OJQJo(hHh  ^ `OJQJ^Jo(hHoh^`OJQJo(hHh^`OJQJo(hHoh^`OJQJ^Jo(hHohpp^p`OJQJo(hHh@ @ ^@ `OJQJo(hHoh^`OJQJ^Jo(hHoh^`OJQJo(hHh^`OJQJo(hHoh^`OJQJ^Jo(hHohPP^P`OJQJo(hHh88^8`.h ^`hH.h  L ^ `LhH.h   ^ `hH.h xx^x`hH.h HLH^H`LhH.h ^`hH.h ^`hH.h L^`LhH.hpp^p`OJQJo(hHvh@ @ ^@ `OJQJ^Jo(hHoh^`OJQJo(hHh^`OJQJo(hHh^`OJQJ^Jo(hHoh^`OJQJo(hHhPP^P`OJQJo(hHh  ^ `OJQJ^Jo(hHoh^`OJQJo(hH@OJQJ^Jo(.^`o(. ^`hH. pLp^p`LhH. @ @ ^@ `hH. ^`hH. L^`LhH. ^`hH. ^`hH. PLP^P`LhH.@OJQJ^Jo(.h^`OJQJo(hHvh^`OJQJ^Jo(hHohpp^p`OJQJo(hHh@ @ ^@ `OJQJo(hHh^`OJQJ^Jo(hHoh^`OJQJo(hHh^`OJQJo(hHh^`OJQJ^Jo(hHohPP^P`OJQJo(hH88^8`o(. ^`hH.  L ^ `LhH.   ^ `hH. xx^x`hH. HLH^H`LhH. ^`hH. ^`hH. L^`LhH.h^`OJQJo(hHh^`OJQJ^Jo(hHohpp^p`OJQJo(hHh@ @ ^@ `OJQJo(hHh^`OJQJ^Jo(hHoh^`OJQJo(hHh^`OJQJo(hHh^`OJQJ^Jo(hHohPP^P`OJQJo(hHhh^h`o(.@OJQJ^Jo(.@rOJQJ^Jo(.LL^L`o(.^`o(.^`o(.^`.  ^ `.\ L\ ^\ `L.,,^,`.^`.L^`L.h^`OJQJo(hHvh^`OJQJ^Jo(hHohpp^p`OJQJo(hHh@ @ ^@ `OJQJo(hHh^`OJQJ^Jo(hHoh^`OJQJo(hHh^`OJQJo(hHh^`OJQJ^Jo(hHohPP^P`OJQJo(hH @OJQJ^Jo(.h88^8`OJQJo(hHoh ^`o(hH.h  ^ `OJQJo(hHh  ^ `OJQJo(hHhxx^x`OJQJ^Jo(hHohHH^H`OJQJo(hHh^`OJQJo(hHh^`OJQJ^Jo(hHoh^`OJQJo(hH88^8`o() ^`OJQJo("  ^`OJQJo("  pp^p`OJQJo("  @ @ ^@ `OJQJo("  ^`OJQJo("  ^`OJQJo("  ^`OJQJo("  ^`OJQJo("  PP^P`OJQJo(" hh^h`o(. @OJQJ^Jo(.h^`OJQJo(hHh^`OJQJ^Jo(hHohpp^p`OJQJo(hHh@ @ ^@ `OJQJo(hHh^`OJQJ^Jo(hHoh^`OJQJo(hHh^`OJQJo(hHh^`OJQJ^Jo(hHohPP^P`OJQJo(hH.^`OJPJQJ^Jo(-^`OJQJ^Jo(hHopp^p`OJQJo(hH@ @ ^@ `OJQJo(hH^`OJQJ^Jo(hHo^`OJQJo(hH^`OJQJo(hH^`OJQJ^Jo(hHoPP^P`OJQJo(hHh^`OJQJo(hHvh^`OJQJ^Jo(hHohpp^p`OJQJo(hHh@ @ ^@ `OJQJo(hHh^`OJQJ^Jo(hHoh^`OJQJo(hHh^`OJQJo(hHh^`OJQJ^Jo(hHohPP^P`OJQJo(hHh^`OJQJo(hHh^`OJQJ^Jo(hHohpp^p`OJQJo(hHh@ @ ^@ `OJQJo(hHh^`OJQJ^Jo(hHoh^`OJQJo(hHh^`OJQJo(hHh^`OJQJ^Jo(hHohPP^P`OJQJo(hH88^8`OJPJQJ^J.h^`OJQJo(hH^`OJQJ^Jo(hHopp^p`OJQJo(hH@ @ ^@ `OJQJo(hHh ^`o(hH.^`OJQJo(hH^`OJQJo(hH^`OJQJ^Jo(hHoPP^P`OJQJo(hH@OJQJ^Jo(.^`o()h^`OJQJo(hHvh^`OJQJ^Jo(hHohpp^p`OJQJo(hHh@ @ ^@ `OJQJo(hHh^`OJQJ^Jo(hHoh^`OJQJo(hHh^`OJQJo(hHh^`OJQJ^Jo(hHohPP^P`OJQJo(hHh^`OJQJo(hHvh^`OJQJ^Jo(hHohpp^p`OJQJo(hHh@ @ ^@ `OJQJo(hHh^`OJQJ^Jo(hHoh^`OJQJo(hHh^`OJQJo(hHh^`OJQJ^Jo(hHohPP^P`OJQJo(hH^`o(.0^`0o(.$ $ ^$ `o(.@ @ ^@ `o()^`.L^`L.^`.^`.PLP^P`L.88^8`OJPJQJ^J.h^`.h ^`hH.h pLp^p`LhH.h @ @ ^@ `hH.h ^`hH.h L^`LhH.h ^`hH.h ^`hH.h PLP^P`LhH.hpp^p`OJQJo(hHoh@ @ ^@ `OJQJ^Jo(hHoh^`OJQJo(hHh^`OJQJo(hHh^`OJQJ^Jo(hHoh^`OJQJo(hHhPP^P`OJQJo(hHh  ^ `OJQJ^Jo(hHoh^`OJQJo(hH@OJQJ^Jo(.^`o(.pp^p`.@ L@ ^@ `L.^`.^`.L^`L.^`.PP^P`. L ^ `L.88^8`o(.^`o(.  L ^ `LhH.   ^ `hH. xx^x`hH. HLH^H`LhH. ^`hH. ^`hH. L^`LhH.@vOJQJ^Jo(.@OJQJ^Jo(.h^`..h^`OJPJQJ^Jo(-h pLp^p`LhH.h @ @ ^@ `hH.h ^`hH.h L^`LhH.h ^`hH.h ^`hH.h PLP^P`LhH.^`o(. ^`hH. pLp^p`LhH. @ @ ^@ `hH. ^`hH. L^`LhH. ^`hH. ^`hH. PLP^P`LhH.h^`OJQJo(hHh^`OJQJ^Jo(hHohpp^p`OJQJo(hHh@ @ ^@ `OJQJo(hHh^`OJQJ^Jo(hHoh^`OJQJo(hHh^`OJQJo(hHh^`OJQJ^Jo(hHohPP^P`OJQJo(hHh^`OJQJo(hHvh^`OJQJ^Jo(hHohpp^p`OJQJo(hHh@ @ ^@ `OJQJo(hHh^`OJQJ^Jo(hHoh^`OJQJo(hHh^`OJQJo(hHh^`OJQJ^Jo(hHohPP^P`OJQJo(hH^`o(. ^`hH. pLp^p`LhH. @ @ ^@ `hH. ^`hH. L^`LhH. ^`hH. ^`hH. PLP^P`LhH.^`o(.^`o(.pp^p`o()  ^ `.\ \ ^\ `.,L,^,`L.^`.^`.L^`L.h^`OJQJo(hHvh^`OJQJ^Jo(hHohpp^p`OJQJo(hHh@ @ ^@ `OJQJo(hHh^`OJQJ^Jo(hHoh^`OJQJo(hHh^`OJQJo(hHh^`OJQJ^Jo(hHohPP^P`OJQJo(hHh^`OJQJo(hHvh^`OJQJ^Jo(hHohpp^p`OJQJo(hHh@ @ ^@ `OJQJo(hHh^`OJQJ^Jo(hHoh^`OJQJo(hHh^`OJQJo(hHh^`OJQJ^Jo(hHohPP^P`OJQJo(hHhhh^h`OJQJo(hHh88^8`OJQJ^Jo(hHoh^`OJQJo(hHh  ^ `OJQJo(hHh  ^ `OJQJ^Jo(hHohxx^x`OJQJo(hHhHH^H`OJQJo(hHh^`OJQJ^Jo(hHoh^`OJQJo(hH2@OJQJ^Jo(.h88^8`OJQJo(hHvh^`OJQJ^Jo(hHoh  ^ `OJQJo(hHh  ^ `OJQJo(hHhxx^x`OJQJ^Jo(hHohHH^H`OJQJo(hHh^`OJQJo(hHh^`OJQJ^Jo(hHoh^`OJQJo(hH^`OJQJo(hH^`OJQJ^Jo(hHohpp^p`OJQJo(hH@ @ ^@ `OJQJo(hH^`OJQJ^Jo(hHo^`OJQJo(hH^`OJQJo(hH^`OJQJ^Jo(hHoPP^P`OJQJo(hHh^`OJQJo(hHh^`OJQJ^Jo(hHohpp^p`OJQJo(hHh@ @ ^@ `OJQJo(hHh^`OJQJ^Jo(hHoh^`OJQJo(hHh^`OJQJo(hHh^`OJQJ^Jo(hHohPP^P`OJQJo(hH88^8`o(. ^`hH.  L ^ `LhH.   ^ `hH. xx^x`hH. HLH^H`LhH. ^`hH. ^`hH. L^`LhH.h^`OJQJ^Jo(hHohpp^p`OJQJ^Jo(hHoh@ @ ^@ `OJQJo(hHh^`OJQJo(hHh^`OJQJ^Jo(hHoh^`OJQJo(hHh^`OJQJo(hHhPP^P`OJQJ^Jo(hHoh  ^ `OJQJo(hHh88^8`OJQJo(hHvh^`OJQJ^Jo(hHoh  ^ `OJQJo(hHh  ^ `OJQJo(hHhxx^x`OJQJ^Jo(hHohHH^H`OJQJo(hHh^`OJQJo(hHh^`OJQJ^Jo(hHoh^`OJQJo(hHh^`OJQJo(hHvh^`OJQJ^Jo(hHohpp^p`OJQJo(hHh@ @ ^@ `OJQJo(hHh^`OJQJ^Jo(hHoh^`OJQJo(hHh^`OJQJo(hHh^`OJQJ^Jo(hHohPP^P`OJQJo(hHh^`OJQJo(hHh^`OJQJ^Jo(hHohpp^p`OJQJo(hHh@ @ ^@ `OJQJo(hHh^`OJQJ^Jo(hHoh^`OJQJo(hHh^`OJQJo(hHh^`OJQJ^Jo(hHohPP^P`OJQJo(hHh^`.h ^`hH.h pLp^p`LhH.h @ @ ^@ `hH.h ^`hH.h L^`LhH.h ^`hH.h ^`hH.h PLP^P`LhH.h^`OJQJo(hHvh^`OJQJ^Jo(hHohpp^p`OJQJo(hHh@ @ ^@ `OJQJo(hHh^`OJQJ^Jo(hHoh^`OJQJo(hHh^`OJQJo(hHh^`OJQJ^Jo(hHohPP^P`OJQJo(hHaa^a`o()1 1 ^1 `.L^`L.^`.^`.qLq^q`L.AA^A`.^`.L^`L.hhh^h`OJQJo(hHh88^8`OJQJ^Jo(hHoh^`OJQJo(hHh  ^ `OJQJo(hHh  ^ `OJQJ^Jo(hHohxx^x`OJQJo(hHhHH^H`OJQJo(hHh^`OJQJ^Jo(hHoh^`OJQJo(hHh^`.h ^`hH.h pLp^p`LhH.h @ @ ^@ `hH.h ^`hH.h L^`LhH.h ^`hH.h ^`hH.h PLP^P`LhH.@OJQJ^Jo(.h^`OJQJo(hHvh^`OJQJ^Jo(hHohpp^p`OJQJo(hHh@ @ ^@ `OJQJo(hHh^`OJQJ^Jo(hHoh^`OJQJo(hHh^`OJQJo(hHh^`OJQJ^Jo(hHohPP^P`OJQJo(hH88^8`o(. ^`hH.  L ^ `LhH.   ^ `hH. xx^x`hH. HLH^H`LhH. ^`hH. ^`hH. L^`LhH.h^`OJQJo(hHvh^`OJQJ^Jo(hHohpp^p`OJQJo(hHh@ @ ^@ `OJQJo(hHh^`OJQJ^Jo(hHoh^`OJQJo(hHh^`OJQJo(hHh^`OJQJ^Jo(hHohPP^P`OJQJo(hH88^8`o(.^`o(.  ^ `o(.   ^ `hH. xx^x`hH. HLH^H`LhH. ^`hH. ^`hH. L^`LhH. @hOJQJ^Jo(.h^`OJQJo(hHvh  ^ `OJQJ^Jo(hHoh  ^ `OJQJo(hHhxx^x`OJQJo(hHhHH^H`OJQJ^Jo(hHoh^`OJQJo(hHh^`OJQJo(hHh^`OJQJ^Jo(hHoh^`OJQJo(hHh^`OJQJo(hHh^`OJQJ^Jo(hHoh^`OJQJo(hHh^`OJQJo(hHhPP^P`OJQJ^Jo(hHoh  ^ `OJQJo(hHh^`OJQJo(hHh!!^!`OJQJ^Jo(hHoh$$^$`OJQJo(hH@OJQJ^Jo(.h^`OJQJo(hHh^`OJQJ^Jo(hHohpp^p`OJQJo(hHh@ @ ^@ `OJQJo(hHh^`OJQJ^Jo(hHoh^`OJQJo(hHh^`OJQJo(hHh^`OJQJ^Jo(hHohPP^P`OJQJo(hH 7 ^ `7o( M 7M ^M `7o(-7^`7o(-.U7U^U`7o(-.. 7^`7o( -... ]7]^]`7o( -.... 7^`7o( -..... e"7e"^e"`7o(-...... %7%^%`7o(-.......hh^h`o(.h  ^ `OJQJo(hHh  ^ `OJQJ^Jo(hHohxx^x`OJQJo(hHhHH^H`OJQJo(hHh^`OJQJ^Jo(hHoh^`OJQJo(hHh^`OJQJo(hHh^`OJQJ^Jo(hHohX X ^X `OJQJo(hHh^`OJQJo(hHvh^`OJQJ^Jo(hHohpp^p`OJQJo(hHh@ @ ^@ `OJQJo(hHh^`OJQJ^Jo(hHoh^`OJQJo(hHh^`OJQJo(hHh^`OJQJ^Jo(hHohPP^P`OJQJo(hH88^8`o(. ^`hH.  L ^ `LhH.   ^ `hH. xx^x`hH. HLH^H`LhH. ^`hH. ^`hH. L^`LhH.h^`OJQJo(hHvh^`OJQJ^Jo(hHohpp^p`OJQJo(hHh@ @ ^@ `OJQJo(hHh^`OJQJ^Jo(hHoh^`OJQJo(hHh^`OJQJo(hHh^`OJQJ^Jo(hHohPP^P`OJQJo(hH @OJQJ^Jo(.h^`OJQJo(hHvh^`OJQJ^Jo(hHohpp^p`OJQJo(hHh@ @ ^@ `OJQJo(hHh^`OJQJ^Jo(hHoh^`OJQJo(hHh^`OJQJo(hHh^`OJQJ^Jo(hHohPP^P`OJQJo(hHLL^L`o(.^`o(.^`o(.^`.  ^ `.\ L\ ^\ `L.,,^,`.^`.L^`L.f@OJQJ^Jo(.h^`OJQJo(hHh^`OJQJ^Jo(hHohpp^p`OJQJo(hHh@ @ ^@ `OJQJo(hHh^`OJQJ^Jo(hHoh^`OJQJo(hHh^`OJQJo(hHh^`OJQJ^Jo(hHohPP^P`OJQJo(hH88^8`o(. ^`hH.  L ^ `LhH.   ^ `hH. xx^x`hH. HLH^H`LhH. ^`hH. ^`hH. L^`LhH.h^`OJQJo(hHvh^`OJQJ^Jo(hHohpp^p`OJQJo(hHh@ @ ^@ `OJQJo(hHh^`OJQJ^Jo(hHoh^`OJQJo(hHh^`OJQJo(hHh^`OJQJ^Jo(hHohPP^P`OJQJo(hH^`5o(. ^`hH. pLp^p`LhH. @ @ ^@ `hH. ^`hH. L^`LhH. ^`hH. ^`hH. PLP^P`LhH.88^8`o(. ^`hH.  L ^ `LhH.   ^ `hH. xx^x`hH. HLH^H`LhH. ^`hH. ^`hH. L^`LhH.h^`OJQJo(hHh^`OJQJ^Jo(hHohpp^p`OJQJo(hHh@ @ ^@ `OJQJo(hHh^`OJQJ^Jo(hHoh^`OJQJo(hHh^`OJQJo(hHh^`OJQJ^Jo(hHohPP^P`OJQJo(hHh^`OJQJo(hHh^`OJQJ^Jo(hHohpp^p`OJQJo(hHh@ @ ^@ `OJQJo(hHh^`OJQJ^Jo(hHoh^`OJQJo(hHh^`OJQJo(hHh^`OJQJ^Jo(hHohPP^P`OJQJo(hH^`5o(.   ^ `hH.  L ^ `LhH. xx^x`hH. HH^H`hH. L^`LhH. ^`hH. ^`hH. L^`LhH.h^`OJQJo(hHvh^`OJQJ^Jo(hHohpp^p`OJQJo(hHh@ @ ^@ `OJQJo(hHh^`OJQJ^Jo(hHoh^`OJQJo(hHh^`OJQJo(hHh^`OJQJ^Jo(hHohPP^P`OJQJo(hH000^0`0o(.^`o(.^`o(.88^8`o(.^`OJPJQJ^Jo(- L ^ `L.  ^ `.xx^x`.HLH^H`L.h^`OJQJo(hHoh^`OJQJ^Jo(hHohpp^p`OJQJo(hHh@ @ ^@ `OJQJo(hHh^`OJQJ^Jo(hHoh^`OJQJo(hHh^`OJQJo(hHh^`OJQJ^Jo(hHohPP^P`OJQJo(hHh^`OJQJo(hHh^`OJQJ^Jo(hHohpp^p`OJQJo(hHh@ @ ^@ `OJQJo(hHh^`OJQJ^Jo(hHoh^`OJQJo(hHh^`OJQJo(hHh^`OJQJ^Jo(hHohPP^P`OJQJo(hH@rOJQJ^Jo(.h^`OJQJo(hHh^`OJQJ^Jo(hHohpp^p`OJQJo(hHh@ @ ^@ `OJQJo(hHh^`OJQJ^Jo(hHoh^`OJQJo(hHh^`OJQJo(hHh^`OJQJ^Jo(hHohPP^P`OJQJo(hHh^`OJQJo(hHvh^`OJQJ^Jo(hHohpp^p`OJQJo(hHh@ @ ^@ `OJQJo(hHh^`OJQJ^Jo(hHoh^`OJQJo(hHh^`OJQJo(hHh^`OJQJ^Jo(hHohPP^P`OJQJo(hH @OJQJ^Jo(.hhh^h`.h 88^8`hH.h L^`LhH.h   ^ `hH.h   ^ `hH.h xLx^x`LhH.h HH^H`hH.h ^`hH.h L^`LhH.h^`OJQJo(hHvh^`OJQJ^Jo(hHohpp^p`OJQJo(hHh@ @ ^@ `OJQJo(hHh^`OJQJ^Jo(hHoh^`OJQJo(hHh^`OJQJo(hHh^`OJQJ^Jo(hHohPP^P`OJQJo(hHh^`OJQJo(hHh^`OJQJ^Jo(hHohpp^p`OJQJo(hHh@ @ ^@ `OJQJo(hHh^`OJQJ^Jo(hHoh^`OJQJo(hHh^`OJQJo(hHh^`OJQJ^Jo(hHohPP^P`OJQJo(hHh88^8`OJQJo(hHh^`OJQJ^Jo(hHoh  ^ `OJQJo(hHh  ^ `OJQJo(hHhxx^x`OJQJ^Jo(hHohHH^H`OJQJo(hHh^`OJQJo(hHh^`OJQJ^Jo(hHoh^`OJQJo(hHh^`OJQJo(hHvh^`OJQJ^Jo(hHohpp^p`OJQJo(hHh@ @ ^@ `OJQJo(hHh^`OJQJ^Jo(hHoh^`OJQJo(hHh^`OJQJo(hHh^`OJQJ^Jo(hHohPP^P`OJQJo(hHh^`OJQJo(hHh^`OJQJ^Jo(hHohpp^p`OJQJo(hHh@ @ ^@ `OJQJo(hHh^`OJQJ^Jo(hHoh^`OJQJo(hHh^`OJQJo(hHh^`OJQJ^Jo(hHohPP^P`OJQJo(hHhhh^h`OJQJo(hHh88^8`OJQJ^Jo(hHoh^`OJQJo(hHh  ^ `OJQJo(hHh  ^ `OJQJ^Jo(hHohxx^x`OJQJo(hHhHH^H`OJQJo(hHh^`OJQJ^Jo(hHoh^`OJQJo(hHh^`OJQJo(hHh^`OJQJ^Jo(hHohpp^p`OJQJo(hHh@ @ ^@ `OJQJo(hHh^`OJQJ^Jo(hHoh^`OJQJo(hHh^`OJQJo(hHh^`OJQJ^Jo(hHohPP^P`OJQJo(hHh^`OJQJo(hHh^`OJQJ^Jo(hHohpp^p`OJQJo(hHh@ @ ^@ `OJQJo(hHh^`OJQJ^Jo(hHoh^`OJQJo(hHh^`OJQJo(hHh^`OJQJ^Jo(hHohPP^P`OJQJo(hHh^`OJQJo(hHvh ^`o(hH.hpp^p`OJQJo(hHvh@ @ ^@ `OJQJo(hHh^`OJQJ^Jo(hHoh^`OJQJo(hHh^`OJQJo(hHh^`OJQJ^Jo(hHohPP^P`OJQJo(hH88^8`o(.^`o(.  ^ `o()  ^ `.xx^x`.HLH^H`L.^`.^`.L^`L.h^`OJQJo(hHh^`OJQJ^Jo(hHohpp^p`OJQJo(hHh@ @ ^@ `OJQJo(hHh^`OJQJ^Jo(hHoh^`OJQJo(hHh^`OJQJo(hHh^`OJQJ^Jo(hHohPP^P`OJQJo(hHh^`OJQJo(hHvh  ^ `OJQJ^Jo(hHoh  ^ `OJQJo(hHhxx^x`OJQJo(hHhHH^H`OJQJ^Jo(hHoh^`OJQJo(hHh^`OJQJo(hHh^`OJQJ^Jo(hHoh^`OJQJo(hHh^`OJQJo(hHvh^`OJQJ^Jo(hHohpp^p`OJQJo(hHh@ @ ^@ `OJQJo(hHh^`OJQJ^Jo(hHoh^`OJQJo(hHh^`OJQJo(hHh^`OJQJ^Jo(hHohPP^P`OJQJo(hH@OJQJ^Jo(.88^8`o(. ^`hH.  L ^ `LhH.   ^ `hH. xx^x`hH. HLH^H`LhH. ^`hH. ^`hH. L^`LhH.h  ^ `OJQJo(hHoh  ^ `OJQJ^Jo(hHohxx^x`OJQJo(hHhHH^H`OJQJo(hHh^`OJQJ^Jo(hHoh^`OJQJo(hHh^`OJQJo(hHh^`OJQJ^Jo(hHohX X ^X `OJQJo(hHh^`OJQJo(hHohpp^p`OJQJ^Jo(hHoh@ @ ^@ `OJQJo(hHh^`OJQJo(hHh^`OJQJ^Jo(hHoh^`OJQJo(hHh^`OJQJo(hHhPP^P`OJQJ^Jo(hHoh  ^ `OJQJo(hH.^`OJPJQJ^Jo(-^`OJQJ^Jo(hHopp^p`OJQJo(hH@ @ ^@ `OJQJo(hH^`OJQJ^Jo(hHo^`OJQJo(hH^`OJQJo(hH^`OJQJ^Jo(hHoPP^P`OJQJo(hHh^`OJQJo(hHvh^`OJQJ^Jo(hHohpp^p`OJQJo(hHh@ @ ^@ `OJQJo(hHh^`OJQJ^Jo(hHoh^`OJQJo(hHh^`OJQJo(hHh^`OJQJ^Jo(hHohPP^P`OJQJo(hHh^`OJQJo(hHvh  ^ `OJQJ^Jo(hHoh  ^ `OJQJo(hHhxx^x`OJQJo(hHhHH^H`OJQJ^Jo(hHoh^`OJQJo(hHh^`OJQJo(hHh^`OJQJ^Jo(hHoh^`OJQJo(hHh^`OJQJo(hHh^`OJQJ^Jo(hHohpp^p`OJQJo(hHh@ @ ^@ `OJQJo(hHh^`OJQJ^Jo(hHoh^`OJQJo(hHh^`OJQJo(hHh^`OJQJ^Jo(hHohPP^P`OJQJo(hHh^`OJQJo(hHvh^`OJQJ^Jo(hHohpp^p`OJQJo(hHh@ @ ^@ `OJQJo(hHh^`OJQJ^Jo(hHoh^`OJQJo(hHh^`OJQJo(hHh^`OJQJ^Jo(hHohPP^P`OJQJo(hHh^`OJQJo(hHh^`OJQJ^Jo(hHoh^`OJQJo(hHh^`OJQJo(hHhPP^P`OJQJ^Jo(hHoh  ^ `OJQJo(hHh^`OJQJo(hHh!!^!`OJQJ^Jo(hHoh$$^$`OJQJo(hH @OJQJ^Jo(.@OJQJ^Jo(.h^`OJQJo(hHoh^`OJQJ^Jo(hHohpp^p`OJQJo(hHh@ @ ^@ `OJQJo(hHh^`OJQJ^Jo(hHoh^`OJQJo(hHh^`OJQJo(hHh^`OJQJ^Jo(hHohPP^P`OJQJo(hHh^`OJQJo(hHvh^`OJQJ^Jo(hHohpp^p`OJQJo(hHh@ @ ^@ `OJQJo(hHh^`OJQJ^Jo(hHoh^`OJQJo(hHh^`OJQJo(hHh^`OJQJ^Jo(hHohPP^P`OJQJo(hHh^`OJQJo(hHvh^`OJQJ^Jo(hHohpp^p`OJQJo(hHh@ @ ^@ `OJQJo(hHh^`OJQJ^Jo(hHoh^`OJQJo(hHh^`OJQJo(hHh^`OJQJ^Jo(hHohPP^P`OJQJo(hHh^`OJQJo(hHvh^`OJQJ^Jo(hHohpp^p`OJQJo(hHh@ @ ^@ `OJQJo(hHh^`OJQJ^Jo(hHoh^`OJQJo(hHh^`OJQJo(hHh^`OJQJ^Jo(hHohPP^P`OJQJo(hHh^`OJQJo(hHvh^`OJQJ^Jo(hHohpp^p`OJQJo(hHh@ @ ^@ `OJQJo(hHh^`OJQJ^Jo(hHoh^`OJQJo(hHh^`OJQJo(hHh^`OJQJ^Jo(hHohPP^P`OJQJo(hHh^`OJQJo(hHvh^`OJQJ^Jo(hHohpp^p`OJQJo(hHh@ @ ^@ `OJQJo(hHh^`OJQJ^Jo(hHoh^`OJQJo(hHh^`OJQJo(hHh^`OJQJ^Jo(hHohPP^P`OJQJo(hH@@OJQJ^Jo(.h88^8`OJQJo(hHoh^`OJQJ^Jo(hHoh  ^ `OJQJo(hHh  ^ `OJQJo(hHhxx^x`OJQJ^Jo(hHohHH^H`OJQJo(hHh^`OJQJo(hHh^`OJQJ^Jo(hHoh^`OJQJo(hH^`o(.^`.hLh^h`L.88^8`.^`. L ^ `L.  ^ `.xx^x`.HLH^H`L.f@OJQJ^Jo(.h^`OJQJo(hHh^`OJQJ^Jo(hHohpp^p`OJQJo(hHh@ @ ^@ `OJQJo(hHh^`OJQJ^Jo(hHoh^`OJQJo(hHh^`OJQJo(hHh^`OJQJ^Jo(hHohPP^P`OJQJo(hH88^8`o(. ^`hH.  L ^ `LhH.   ^ `hH. xx^x`hH. HLH^H`LhH. ^`hH. ^`hH. L^`LhH.h^`OJQJo(hHh^`OJQJ^Jo(hHohpp^p`OJQJo(hHh@ @ ^@ `OJQJo(hHh^`OJQJ^Jo(hHoh^`OJQJo(hHh^`OJQJo(hHh^`OJQJ^Jo(hHohPP^P`OJQJo(hHh^`OJQJo(hHvh^`OJQJ^Jo(hHohpp^p`OJQJo(hHh@ @ ^@ `OJQJo(hHh^`OJQJ^Jo(hHoh^`OJQJo(hHh^`OJQJo(hHh^`OJQJ^Jo(hHohPP^P`OJQJo(hHh$ $ ^$ `OJQJo(hHh   ^ `hH.hxx^x`OJQJo(hH^`o(.h dd^d`hH.h 4L4^4`LhH.h ^`hH.h ^`hH.h L^`LhH.`@OJQJ^Jo(.h^`OJQJo(hHvh^`OJQJ^Jo(hHohpp^p`OJQJo(hHh@ @ ^@ `OJQJo(hHh^`OJQJ^Jo(hHoh^`OJQJo(hHh^`OJQJo(hHh^`OJQJ^Jo(hHohPP^P`OJQJo(hHh^`OJQJo(hHvh^`OJQJ^Jo(hHohpp^p`OJQJo(hHh@ @ ^@ `OJQJo(hHh^`OJQJ^Jo(hHoh^`OJQJo(hHh^`OJQJo(hHh^`OJQJ^Jo(hHohPP^P`OJQJo(hHh88^8`OJQJo(hHoh^`OJQJ^Jo(hHoh  ^ `OJQJo(hHh  ^ `OJQJo(hHhxx^x`OJQJ^Jo(hHohHH^H`OJQJo(hHh^`OJQJo(hHh^`OJQJ^Jo(hHoh^`OJQJo(hH ^`OJQJo("  ^`OJQJo("  pp^p`OJQJo("  @ @ ^@ `OJQJo("  ^`OJQJo("  ^`OJQJo("  ^`OJQJo("  ^`OJQJo("  PP^P`OJQJo(" h^`.h ^`hH.h pLp^p`LhH.h @ @ ^@ `hH.h ^`hH.h L^`LhH.h ^`hH.h ^`hH.h PLP^P`LhH.h^`OJQJo(hHvh^`OJQJo(hHohpp^p`OJQJo(hHvh@ @ ^@ `OJQJo(hHh^`OJQJ^Jo(hHoh^`OJQJo(hHh^`OJQJo(hHh^`OJQJ^Jo(hHohPP^P`OJQJo(hHXX^X`o(.h^`OJQJo(hHh^`OJQJ^Jo(hHohpp^p`OJQJo(hHh@ @ ^@ `OJQJo(hHh^`OJQJ^Jo(hHoh^`OJQJo(hHh^`OJQJo(hHh^`OJQJ^Jo(hHohPP^P`OJQJo(hHh^`OJQJo(hHvh^`OJQJ^Jo(hHohpp^p`OJQJo(hHh@ @ ^@ `OJQJo(hHh^`OJQJ^Jo(hHoh^`OJQJo(hHh^`OJQJo(hHh^`OJQJ^Jo(hHohPP^P`OJQJo(hHh^`OJQJo(hHvh^`OJQJ^Jo(hHohpp^p`OJQJo(hHh@ @ ^@ `OJQJo(hHh^`OJQJ^Jo(hHoh^`OJQJo(hHh^`OJQJo(hHh^`OJQJ^Jo(hHohPP^P`OJQJo(hH@OJQJ^Jo(.h^`OJQJo(hHvh^`OJQJ^Jo(hHohpp^p`OJQJo(hHvh@ @ ^@ `OJQJo(hHh^`OJQJ^Jo(hHoh^`OJQJo(hHh^`OJQJo(hHh^`OJQJ^Jo(hHohPP^P`OJQJo(hHh^`.hpp^p`OJQJo(hHvh @ L@ ^@ `LhH.h ^`hH.h ^`hH.h L^`LhH.h ^`hH.h PP^P`hH.h  L ^ `LhH.h^`OJQJo(hHvh^`OJQJo(hHohpp^p`OJQJo(hHh@ @ ^@ `OJQJo(hHh^`OJQJ^Jo(hHoh^`OJQJo(hHh^`OJQJo(hHh^`OJQJ^Jo(hHohPP^P`OJQJo(hH^`o(.^`.pLp^p`L.@ @ ^@ `.^`.L^`L.^`.^`.PLP^P`L.88^8`o(. ^`hH.  L ^ `LhH.   ^ `hH. xx^x`hH. HLH^H`LhH. ^`hH. ^`hH. L^`LhH.^`o(.^`o(.pLp^p`LOJPJQJ^J. @ @ ^@ `hH. ^`hH. L^`LhH. ^`hH. ^`hH. PLP^P`LhH. @OJQJ^Jo(.h^`OJQJo(hHvh^`OJQJ^Jo(hHohpp^p`OJQJo(hHh@ @ ^@ `OJQJo(hHh^`OJQJ^Jo(hHoh^`OJQJo(hHh^`OJQJo(hHh^`OJQJ^Jo(hHohPP^P`OJQJo(hHhhh^h`OJQJo(hHvh88^8`OJQJ^Jo(hHoh^`OJQJo(hHh  ^ `OJQJo(hHh  ^ `OJQJ^Jo(hHohxx^x`OJQJo(hHhHH^H`OJQJo(hHh^`OJQJ^Jo(hHoh^`OJQJo(hH@OJQJ^Jo(.h^`OJQJo(hHvh ^`o(hH.hpp^p`OJQJo(hHh@ @ ^@ `OJQJo(hHh^`OJQJ^Jo(hHoh^`OJQJo(hHh^`OJQJo(hHh^`OJQJ^Jo(hHohPP^P`OJQJo(hH@lOJQJ^Jo(.hpp^p`OJQJo(hHoh@ @ ^@ `OJQJ^Jo(hHoh^`OJQJo(hHh^`OJQJo(hHh^`OJQJ^Jo(hHoh^`OJQJo(hHhPP^P`OJQJo(hHh  ^ `OJQJ^Jo(hHoh^`OJQJo(hHh^`OJQJo(hHvh^`OJQJ^Jo(hHohpp^p`OJQJo(hHh@ @ ^@ `OJQJo(hHh^`OJQJ^Jo(hHoh^`OJQJo(hHh^`OJQJo(hHh^`OJQJ^Jo(hHohPP^P`OJQJo(hHh^`OJQJo(hHh^`OJQJ^Jo(hHohpp^p`OJQJo(hHh@ @ ^@ `OJQJo(hHh^`OJQJ^Jo(hHoh^`OJQJo(hHh^`OJQJo(hHh^`OJQJ^Jo(hHohPP^P`OJQJo(hHh^`.h ^`hH.h pLp^p`LhH.h @ @ ^@ `hH.h ^`hH.h L^`LhH.h ^`hH.h ^`hH.h PLP^P`LhH.h^`B*OJQJo(phhHh^`OJQJ^Jo(hHohpp^p`OJQJo(hHh@ @ ^@ `OJQJo(hHh^`OJQJ^Jo(hHoh^`OJQJo(hHh^`OJQJo(hHh^`OJQJ^Jo(hHohPP^P`OJQJo(hHh^`OJQJo(hHh^`OJQJ^Jo(hHohpp^p`OJQJo(hHh@ @ ^@ `OJQJo(hHh^`OJQJ^Jo(hHoh^`OJQJo(hHh^`OJQJo(hHh^`OJQJ^Jo(hHohPP^P`OJQJo(hHh88^8`OJQJo(hHoh^`OJQJ^Jo(hHoh  ^ `OJQJo(hHh  ^ `OJQJo(hHhxx^x`OJQJ^Jo(hHohHH^H`OJQJo(hHh^`OJQJo(hHh^`OJQJ^Jo(hHoh^`OJQJo(hHhhh^h`.h 88^8`hH.h L^`LhH.h   ^ `hH.h   ^ `hH.h xLx^x`LhH.h HH^H`hH.h ^`hH.h L^`LhH.h^`OJQJo(hHvh^`OJQJ^Jo(hHohpp^p`OJQJo(hHh@ @ ^@ `OJQJo(hHh^`OJQJ^Jo(hHoh^`OJQJo(hHh^`OJQJo(hHh^`OJQJ^Jo(hHohPP^P`OJQJo(hHh^`OJQJo(hHh^`OJQJ^Jo(hHohpp^p`OJQJo(hHh@ @ ^@ `OJQJo(hHh^`OJQJ^Jo(hHoh^`OJQJo(hHh^`OJQJo(hHh^`OJQJ^Jo(hHohPP^P`OJQJo(hH@OJQJ^Jo(.h^`OJQJo(hHh^`OJQJ^Jo(hHohpp^p`OJQJo(hHh@ @ ^@ `OJQJo(hHh^`OJQJ^Jo(hHoh^`OJQJo(hHh^`OJQJo(hHh^`OJQJ^Jo(hHohPP^P`OJQJo(hH^`o(.pp^p`o(. @ L@ ^@ `LhH. ^`hH. ^`hH. L^`LhH. ^`hH. PP^P`hH.  L ^ `LhH.@vOJQJ^Jo(.h^`OJQJo(hHvh^`OJQJ^Jo(hHohpp^p`OJQJo(hHh@ @ ^@ `OJQJo(hHh^`OJQJ^Jo(hHoh^`OJQJo(hHh^`OJQJo(hHh^`OJQJ^Jo(hHohPP^P`OJQJo(hH88^8`o()%@OJQJ^Jo(.@OJQJ^Jo(.h^`OJQJo(hHh^`OJQJ^Jo(hHohpp^p`OJQJo(hHh@ @ ^@ `OJQJo(hHh^`OJQJ^Jo(hHoh^`OJQJo(hHh^`OJQJo(hHh^`OJQJ^Jo(hHohPP^P`OJQJo(hHh^`OJQJo(hHvh^`OJQJ^Jo(hHohpp^p`OJQJo(hHh@ @ ^@ `OJQJo(hHh^`OJQJ^Jo(hHoh^`OJQJo(hHh^`OJQJo(hHh^`OJQJ^Jo(hHohPP^P`OJQJo(hHhpp^p`OJQJo(hHoh@ @ ^@ `OJQJ^Jo(hHoh^`OJQJo(hHh^`OJQJo(hHh^`OJQJ^Jo(hHoh^`OJQJo(hHhPP^P`OJQJo(hHh  ^ `OJQJ^Jo(hHoh^`OJQJo(hHhhh^h`OJQJo(hHh88^8`OJQJo(hHoh^`OJQJo(hHh  ^ `OJQJo(hHh  ^ `OJQJ^Jo(hHohxx^x`OJQJo(hHhHH^H`OJQJo(hHh^`OJQJ^Jo(hHoh^`OJQJo(hH@OJQJ^Jo(.^`OJQJ^Jo(hHo^`OJQJ^Jo(hHopp^p`OJQJo(hH@ @ ^@ `OJQJo(hH^`OJQJ^Jo(hHo^`OJQJo(hH^`OJQJo(hH^`OJQJ^Jo(hHoPP^P`OJQJo(hHh^`OJQJ^Jo(hHoh^`OJQJ^Jo(hHohpp^p`OJQJo(hHh@ @ ^@ `OJQJo(hHh^`OJQJ^Jo(hHoh^`OJQJo(hHh^`OJQJo(hHh^`OJQJ^Jo(hHohPP^P`OJQJo(hH^`o(.^`. L ^ `L.\ \ ^\ `.,,^,`.L^`L.^`.^`.lLl^l`L.%@OJQJ^Jo(.h^`B*OJQJo(phhHvh^`OJQJ^Jo(hHohpp^p`OJQJo(hHh@ @ ^@ `OJQJo(hHh^`OJQJ^Jo(hHoh^`OJQJo(hHh^`OJQJo(hHh^`OJQJ^Jo(hHohPP^P`OJQJo(hHh^`.h pp^p`hH.h @ L@ ^@ `LhH.h ^`hH.h ^`hH.h L^`LhH.h ^`hH.h PP^P`hH.h  L ^ `LhH.88^8`o()h^`OJQJo(hHvh^`OJQJ^Jo(hHohpp^p`OJQJo(hHh@ @ ^@ `OJQJo(hHh^`OJQJ^Jo(hHoh^`OJQJo(hHh^`OJQJo(hHh^`OJQJ^Jo(hHohPP^P`OJQJo(hHC@OJQJ^Jo(.h^`OJQJo(hHh^`OJQJ^Jo(hHohpp^p`OJQJo(hHh@ @ ^@ `OJQJo(hHh^`OJQJ^Jo(hHoh^`OJQJo(hHh^`OJQJo(hHh^`OJQJ^Jo(hHohPP^P`OJQJo(hH@OJQJ^Jo(.b b Dc c c Ld d d Te e f \f f  g dg g k l dl l  m `m m n \n n o Xo o p Xp p q Tq q q Pr r r Ls s s Ht t t Du u u @v v v kaCl2TKV8q)IkQUzX1$v TR3X,dXF$1[)G'u^JPrDEUR>yg-+,z?ZLchzT' \$?h}h\rm?IG['T(0<|.S.I(oeD%A>#al\RRN@k!^}%Aeo:e '[`v9(`;S*! *6S9O tSCf./&&>o$GibqPIW7 H$XbnW]O<}qNWcsnT_CWWd*sF{WD96gXQ%#e$ v{ IGWwkm}O?_MBGh@"y[>::%_|_?: -^..-<*Kp2.2l[;Tn8bw 8x x x 4y *4[sUc0 z iul%[X >FLrM1BF7T~F(U<9ZLH`A\|s11[B<\N3pC|ZS9enr1[V&# Y($$gxwg~Bd<+bTgoi1o;A chXnLT:?Sj"g^ aRG|'q`Fm w$|~VnlrP1<0wM !>y`L M {,@g@HpZ0XJ&dW+y4Wm.6 @ I_l?/} gNOtX{+g eB1SuMLcO&t]IYF ,6$EHfEOkJ7v5?+qAa@lsF=J,Ho0WbY0 t7GaI}I.$.*~Kh0Qy$}M+GA:[1n9|*KAReDBjea[cl{UOr Bpgx[>{[0f|eey sZ hsZ hy sZ h8z &LuH{Gn. pNF4/y|5\JYj~0( v/sz-oD,DJ a)$`?h9Bn S^ 2eP iI<beH S(qUL%E2 }Tz~-3-3z \o!\o!z \o!@{ Tb"|TD&k?6*@ *>DVs}<09v+a:96$)e6OPb @CJ OJQJ^Jo(" b @CJOJQJ^Jo(" Pc @CJOJQJ^Jo( c @CJOJQJ^Jo(" d @CJOJQJ^Jo( Xd @CJOJQJ^Jo( d @CJOJQJ^Jo(" e @CJOJQJ^Jo(" `e @CJ$OJQJ^Jo(" e @CJOJQJ^Jo(" f @CJ OJQJ^Jo(" hf @CJ(OJQJ^Jo(" f @CJ$OJQJ^Jo(" g @CJ,OJQJ^Jo(" pg V @CJ$OJQJ^Jo(-h @CJ OJQJo(" Ph !^`OJQJ^Jo(hHoh "pp^p`OJQJo(hH,i #@ @ ^@ `OJQJo(hHi $^`OJQJ^Jo(hHoj %^`OJQJo(hHtj &^`OJQJo(hHj '^`OJQJ^Jo(hHoPk (PP^P`OJQJo(hHk @CJOJQJo( l @CJOJQJo( pl @CJ OJQJo(l @CJOJQJo(0m @CJOJQJo(hlm @CJ OJQJo(hm @CJOJQJo(" n @CJ$OJQJo(" hn @CJ$OJQJo( n @CJ OJQJo( o @CJOJQJo(" do @CJOJQJo(" o @CJ OJQJ^Jo( p @CJOJQJo( dp @CJOJQJo( p @CJOJQJo( q `p@CJ OJQJo(v`q `a@CJOJQJo(q `D@CJOJQJo(r `M@CJOJQJo(v\r `d@CJOJQJo(r @&@CJOJQJo(s @CJOJQJo(Xs @CJOJQJo(s `d@CJOJQJo(t C@CJ OJQJo(Tt @CJ OJQJo(t @CJOJQJo(t @CJ*OJQJo(Pu @CJOJQJo(u `a@CJOJQJo(u @CJOJQJo(Lv @CJOJQJo(v `l@CJOJQJo(v @CJOJQJo(Hw @CJOJQJo(w `@CJ!OJQJo(w @CJ OJQJo(" Dx @CJOJQJo( x @CJOJQJo(" x @CJOJQJo( @y @CJOJQJo(" y Z(@^OJQJ^Jo(.y B@hOJQJ^Jo(.Dz B@hOJQJ^Jo(.z (@^OJQJ^Jo(.z B@hOJQJ^Jo(.L{ iB@hOJQJ^Jo(.^^                                    $2G֔4Dd!~alexso         FZ(g                                                            6a                                   &h                                            i                                   Gbi                                                                                                                                                                                             6a                                                                                                                                                       ha4i                                                              tNE(o:J^T >èv&Dt/-                  :                                            BXd~L~ZȨHptTM4                                                                                                                                                                                    ۾l"b&e                        ht?0'ٮdeδwCIv!&M         6a                                                      EZ~                       ~z<        f 6a                                  $pd\                                                                                                                  %^                                             6@                                                               n,F-VGiX                                                   .~:_TJ+,fC                                                                                                                                  (z}7                                                   6a                                                                                                                    K                                      ?fA                                0D<)^G>>9BoFة6q                                                                                                                                                "                                                                                                                             F                                            Mp*JO,U8<@$~`p@Unknown Gz Times New Roman5Symbol3& z Arial?5 z Courier New;Wingdings?& Arial BlackCFComic Sans MS7Marlett3z Times"qh䚦䚦4+4+"'24__ 3qH ?O Practice Test Betty Rowe Betty Rowe^                           ! " # $ % & ' ( ) * + , - . / 0 1 2 3 4 5 6 7 8 9 : ; < = > ? @ A B C D E F G H I J K L M N O P Q R S T U V W X Y Z [ \ ] ^ _ ` a b c d e f g h i j k l m n o p q r s t u v w x y z { | } ~                                   ! " # $ % & ' ( ) * + , - . / 0 1 2 3 4 5 6 7 8 9 : ; < = > ? @ A B C D E F G H I J K L M N O P Q R S T U V W X Y Z [ \ ] Oh+'0|  8 D P\dltPractice Testorac Betty Rowesettett Normal.dots Betty Rowes3ttMicrosoft Word 10.0@Ik@bG@4+՜.+,0  hp  LSU HEALTH SCIENCES CENTERe_: Practice Test Title  !"#$%&'()*+,-./0123456789:;<=>?@ABCDEFGHIJKLMNOPQRSTUVWXYZ[\]^_`abcdefghijklmnopqrstuvwxyz{|}~      !"#$%&'()*+,-./0123456789:;<=>?@ABCDEFGHIJKLMNOPQRSTUVWXYZ[\]^_`abcdefghijklmnopqrstuvwxyz{|}~      !"#$%&'()*+,-./0123456789:;<=>?@ABCDEFGHIJKLMNOPQRSTUVWXYZ[\]^_`abcdefghijklmnopqrstuvwxyz{|}~      !"#$%&'()*+,-./0123456789:;<=>?@ABCDEFGHIJKLMNOPQRSTUVWXYZ[\]^_`abcdefghijklmnopqrstuvwxyz{|}~      !"#$%&'()*+,-./0123456789:;<=>?@ABCDEFGHIJKLMNOPQRSTUVWXYZ[\]^_`abcdefghijklmnopqrstuvwxyz{|}~      !"#$%&'()*+,-./0123456789:;<=>?@ABCDEFGHIJKLMNOPQRSTUVWXYZ[\]^_`abcdefghijklmnopqrstuvwxyz{|}~Root Entry FData 1Table;WordDocumenttSummaryInformation(DocumentSummaryInformation8CompObjj  FMicrosoft Word Document MSWordDocWord.Document.89q